SlideShare uma empresa Scribd logo
1 de 76
Baixar para ler offline
Μπάμπης Στεργίου
Διαγωνισμός
Αρχιμήδης
Juniors-Μικροί
Ασκήσεις στη Γεωμετρία
*** Αφιερωμένο στους μαθητές και τους συναδέλφους
2017
ΘΕΩΡΙΑ ΚΑΙ ΕΦΑΡΜΟΓΕΣ
Α. Τρίγωνα – Τετράπλευρα
1Α . Τρίγωνα
Θεωρήματα
α) Οι μεσοκάθετοι των πλευρών κάθε τριγώνου διέρχονται από το ίδιο σημείο
(συντρέχουν). Το σημείο αυτό λέγεται περίκεντρο του τριγώνου αυτού και είναι
το κέντρο του περιγεγραμμένου κύκλου.
β) Οι διχοτόμοι των γωνιών κάθε τριγώνου διέρχονται από το ίδιο σημείο
(συντρέχουν). Το σημείο αυτό λέγεται έγκεντρο του τριγώνου και είναι το
κέντρο του εγγεγραμμένου κύκλου.
γ) Οι διχοτόμοι δύο εξωτερικών γωνιών και της τρίτης εσωτερικής γωνίας
διέρχονται από το ίδιο σημείο. Το σημείο αυτό λέγεται παράκεντρο.
Υπάρχουν τρία παράκεντρα τα οποία είναι κέντρα των παρεγγεγραμμένων
κύκλων. Οι κύκλοι αυτοί εφάπτονται στη μια πλευρά του τριγώνου και στις
προεκτάσεις των δύο άλλων. Τα παράκεντρα και οι παρεγγεγραμμένοι κύκλοι
ενός τριγώνου φαίνονται στο διπλανό σχήμα.
2. Άθροισμα γωνιών τριγώνου – τετραπλεύρου
Θεωρήματα
α) Το άθροισμα των γωνιών κάθε τριγώνου είναι ο
180 . Δηλαδή σε κάθε
τρίγωνο ΑΒΓ ισχύει:
οˆ ˆ ˆΑ Β Γ 180  
β) Η εξωτερική γωνία τριγώνου είναι ίση με το άθροισμα των δύο απέναντι
εσωτερικών του. Έτσι:
εξ
ˆˆ ˆΓ Α Β  ή ω x y 
γ)  Δύο τρίγωνα που έχουν δύο γωνίες ίσες, έχουν και τις τρίτες ίσες.
 Οι οξείες γωνίες ενός ορθογωνίου τριγώνου είναι συμπληρωματικές.
 Κάθε γωνία ισοπλεύρου τριγώνου είναι ίση με o
60 .
 Ένα ισοσκελές τρίγωνο με μία γωνία o
60 είναι ισόπλευρο.
Θεώρημα
Δύο γωνίες που έχουν τις πλευρές τους κάθετες μία προς μία είναι:
 ίσες, αν είναι και οι δύο οξείες ή αμβλείες (έτσι, θα είναι x y) ,
 παραπληρωματικές, αν είναι η μία οξεία και η άλλη αμβλεία (έτσι, θα
είναι ο
x ω 180  ).
Θεώρημα
α) Το άθροισμα των γωνιών κάθε κυρτού ν-γώνου είναι (2ν 4) ορθές ή ο
(180ν 360) .
β) Το άθροισμα των εξωτερικών γωνιών ενός κυρτού ν-γώνου είναι 4 ορθές ή ο
360 .
Θεώρημα
α) Οι διχοτόμοι των γωνιών ˆΒ και ˆΓ ενός τριγώνου σχηματίζουν γωνία ίση
με ο
ˆΑ
90
2
 , δηλαδή:
ο
ˆΑˆΒΙΓ 90
2
 
β) Οι διχοτόμοι των ˆΒ και εξ
ˆΓ σχηματίζουν γωνία ίση με
ˆΑ
2
. Έτσι:
β
ˆΑˆΙ
2

γ) Οι διχοτόμοι των εξ
ˆΒ και εξ
ˆΓ σχηματίζουν γωνία ίση με ο
ˆΑ
90
2
 . Έτσι:
ο
α
ˆΑˆΙ 90
2
 
1.1 Δίνεται ισοσκελές τρίγωνο ΑΒΓ, με ΑΒ = ΑΓ και οˆΑ = 20 . Στις πλευρές ΑΓ και ΑΒ παίρνουμε
σημεία Δ και Ε αντίστοιχα, ώστε οˆΔΒΓ = 50 και οˆΕΓΒ = 60 . Να αποδειχθεί ότι οˆΓΕΔ = 30 .
Λύση
Παίρνουμε στην ΑΒ σημείο Ζ, ώστε οˆΖΓΒ 20 . Τότε:
 ΓΒ ΓΖ , αφού το τρίγωνο ΓΒΖ είναι ισοσκελές,
 ΓΒ ΓΔ , αφού το τρίγωνο ΓΒΔ είναι ισοσκελές,
 ΖΓ ΖΕ , αφού το τρίγωνο ΖΕΓ είναι ισοσκελές.
Αλλά το τρίγωνο ΓΔΖ θα είναι τότε ισόπλευρο, διότι οˆΖΓΔ 60 . Άρα
ΖΔ ΖΓ ΖΕ  , οπότε:
ο ο ο
ο
ˆ180 ΕΖΔ 180 40ˆΖΕΔ 70
2 2
 
  
Τελικά έχουμε:
ο ο οˆ ˆ ˆΓΕΔ ΖΕΔ ΖΕΓ 70 40 30    
1.2 Δίνεται τρίγωνο ΑΒΓ και η διχοτόμος ΑΔ. Αν ισχύει ότι ΑΒ + ΓΔ = ΑΓ + ΒΔ , να αποδειχθεί ότι
ΑΒ = ΑΓ .
Λύση
Θα χρησιμοποιήσουμε βοηθητικές γραμμές. Στις προεκτάσεις των πλευ-
ρών ΑΒ και ΑΓ παίρνουμε τμήμα ΒΕ ΓΔ y  και ΓΖ ΒΔ x 
αντίστοιχα. Έχουμε ότι:
AB ΓΔ ΑΓ ΒΔ   
ΑΒ ΒΕ ΑΓ ΓΖ ΑΕ ΑΖ     
Το τρίγωνο ΑΕΖ είναι οπότε ισοσκελές και έτσι η διχοτόμος ΑΔ της
γωνίας ˆΑ είναι μεσοκάθετος της ΕΖ. Άρα ΔΕ ΔΖ . Έτσι τα τρίγωνα
ΒΔΕ και ΓΔΖ είναι ίσα (έχουν τις πλευρές τους μία προς μία ίσες), οπότε:
ˆ ˆΕΒΔ ΖΓΔ
Επομένως ˆ ˆΒ Γ , ως παραπληρωματικές των ίσων γωνιών ˆΕΒΔ και ˆΖΓΔ . Άρα το τρίγωνο ΑΒΓ είναι
ισοσκελές, οπότε ΑΒ ΑΓ .
1.3 Σε ένα τρίγωνο ΑΒΓ, οι διχοτόμοι των εξωτερικών γωνιών ˆΒ και ˆΓ τέμνονται στο σημείο Ο. Στην
προέκταση της ΑΒ, προς το Β, παίρνουμε τμήμα ΒΔ = ΒΓ + ΑΓ . Να αποδειχθεί ότι ΟΔ = ΟΑ .
Λύση
Στην προέκταση της ΑΓ παίρνουμε τμήμα ΓΕ ΓΒ α  . Φέρνουμε τις ΟΑ και ΒΕ. Το τρίγωνο ΓΒΕ είναι
ισοσκελές, αφού ΓΒ ΓΕ α  . Επομένως η διχοτόμος ΓΟ της εξ
ˆΓ είναι μεσοκάθετος της ΒΕ. Άρα:
ΟΒ ΟΕ (1)
Από την ισότητα των τριγώνων ΒΟΓ και ΕΟΓ παίρνουμε ότι
ˆ ˆΓΒΟ Ε . Έτσι:
ˆ ˆΕ ΔΒΟ ω  (2)
Τα τρίγωνα τώρα ΟΒΔ και ΟΑΕ είναι ίσα, διότι:
 ΟΒ ΟΕ , λόγω της σχέσης (1),
 ΒΔ ΑΕ α β   και
 ˆ ˆΔΒΟ ΟΕΑ ω  , λόγω της σχέσης (2).
Επομένως θα είναι και ΟΔ ΟΑ .
2Α . Παραλληλόγραμμα
Ορισμός
α) Παραλληλόγραμμο ονομάζεται το τετράπλευρο που έχει τις
απέναντι πλευρές του παράλληλες.
β) Αν το ΑΒΓΔ είναι παραλληλόγραμμο, τότε ΑΒ // ΓΔ και ΑΔ // ΒΓ.
Ιδιότητες
Στο παραλληλόγραμμο ισχύουν οι εξής ιδιότητες:
 Οι απέναντι πλευρές είναι ίσες. Έτσι ΑΒ ΓΔ και ΑΔ ΒΓ .
 Οι απέναντι γωνίες είναι ίσες. Έτσι ˆ ˆΑ Γ και ˆˆΒ Δ .
 Οι διαγώνιοι διχοτομούνται. Αυτό σημαίνει ότι ΟΑ ΟΓ και
ΟΒ ΟΔ .
 Το κέντρο του παραλληλογράμμου, δηλαδή το σημείο τομής των διαγωνίων του, είναι κέντρο
συμμετρίας του παραλληλογράμμου.
Κριτήρια
Για να αποδείξουμε ότι ένα τετράπλευρο είναι παραλληλόγραμμο, αρκεί να αποδείξουμε ότι ισχύει ένα από
τα παρακάτω (κριτήρια):
 Οι απέναντι πλευρές είναι παράλληλες.
 Οι απέναντι πλευρές είναι ίσες.
 Οι απέναντι γωνίες είναι ίσες.
 Οι διαγώνιοι διχοτομούνται.
 Δύο απέναντι πλευρές είναι ίσες και παράλληλες.
Σημείωση
Έστω 1(ε ) και 2(ε ) δύο παράλληλες ευθείες. Πάνω σ' αυτές θεωρούμε δύο
τυχαία, αλλά ίσα τμήματα ΑΒ και ΓΔ, όπως φαίνεται στο διπλανό σχήμα.
Τότε το ΑΒΔΓ, δηλαδή το κυρτό τετράπλευρο που σχηματίζεται από τα άκρα
των τμημάτων αυτών, είναι παραλληλόγραμμο. Πραγματικά, επειδή τα ΑΒ,
ΓΔ είναι ίσα και παράλληλα, το ΑΒΔΓ είναι παραλληλόγραμμο.
Η προηγούμενη παρατήρηση βρίσκει συχνή εφαρμογή σε πολλά θέματα με παραλληλόγραμμα.
1.4 Δίνεται τρίγωνο ΑΒΓ, το ισόπλευρο τρίγωνο ΑΒΔ εκτός αυτού, καθώς και το ισοσκελές τρίγωνο ΑΓΕ
στο εξωτερικό του
Δ
ΑΒΓ , με οˆ ˆΕΑΓ = ΕΓΑ = 30 . Αν Μ είναι το μέσο του ΒΓ, να αποδειχθεί ότι
οˆΔΜΕ = 90 .
Λύση
Έστω Ζ το συμμετρικό του Δ ως προς το Μ. Το ΒΔΓΖ είναι παραλληλόγραμμο, διότι οι διαγώνιες
διχοτομούνται. Επομένως είναι:
ΓΖ ΒΔ ΑΔ  και οˆ ˆ ˆΒΓΖ ΓΒΔ 60 Β  
Τα τρίγωνα ΑΔΕ και ΓΕΖ είναι ίσα, διότι:
 ΑΔ ΓΖ γ  και ΑΕ ΓΕ
 ο ο ο ο ο ο ο ˆˆ ˆ ˆ ˆ ˆ ˆ ˆ ˆΕΓΖ 360 ΕΓΑ ΑΓΒ ΒΓΖ 360 30 Γ (60 Β) 270 (Β Γ) 270 (180 Α)               
ο ˆ ˆ90 Α ΔΑΕ  
Άρα είναι και ΕΔ ΕΖ και αφού στο ισοσκελές τρίγωνο ΕΔΖ η ΕΜ είναι διάμεσος, είναι και ύψος. Είναι
δηλαδή οˆΔΜΕ 90 .
3Α . Είδη Παραλληλογράμμων
Ορισμός
Ορθογώνιο ονομάζεται το παραλληλόγραμμο που έχει μία
(τουλάχιστον) ορθή γωνία. Προφανώς, στο ορθογώνιο όλες οι γωνίες
είναι ορθές.
Ιδιότητες
Στο ορθογώνιο, εκτός από τις ιδιότητες που ισχύουν σε κάθε
παραλληλόγραμμο, ισχύουν και οι εξής:
 όλες οι γωνίες είναι ορθές,
 οι διαγώνιοι είναι ίσες.
Κριτήρια
Για να αποδείξουμε ότι ένα τετράπλευρο είναι ορθογώνιο, αποδεικνύουμε ότι:
 είναι παραλληλόγραμμο και έχει μία ορθή γωνία ή
 είναι παραλληλόγραμμο και έχει ίσες διαγωνίους.
Τονίζουμε ότι:
 Αν σε ένα τετράπλευρο τρεις γωνίες είναι ορθές, τότε αυτό είναι ορθογώνιο.
 Αν οι γωνίες ενός τετραπλεύρου είναι ίσες, τότε αυτό είναι ορθογώνιο.
Ορισμός
Ρόμβος ονομάζεται το παραλληλόγραμμο που έχει δύο διαδοχικές πλευρές ίσες.
Προφανώς, στον ρόμβο όλες οι πλευρές είναι ίσες.
Ιδιότητες
Στον ρόμβο, εκτός από τις ιδιότητες που ισχύουν σε κάθε παραλληλόγραμμο, ισχύουν
και οι εξής:
 όλες οι πλευρές είναι ίσες,
 οι διαγώνιοι τέμνονται κάθετα,
 οι διαγώνιοι διχοτομούν τις γωνίες.
Κριτήρια
Για να αποδείξουμε ότι ένα τετράπλευρο είναι ρόμβος, αποδεικνύουμε ότι:
 είναι παραλληλόγραμμο και έχει δύο διαδοχικές πλευρές ίσες ή
 είναι παραλληλόγραμμο και οι διαγώνιοί του τέμνονται κάθετα ή
 είναι παραλληλόγραμμο και μία διαγώνιός του διχοτομεί μία γωνία του.
Τονίζουμε ότι αν όλες οι πλευρές ενός τετραπλεύρου είναι ίσες, τότε είναι ρόμβος.
Ορισμός
Τετράγωνο ονομάζεται το παραλληλόγραμμο που είναι συγχρόνως και ορθογώνιο και
ρόμβος. Προφανώς, στο τετράγωνο όλες οι πλευρές είναι ίσες και όλες οι γωνίες είναι
ορθές.
Ιδιότητες
Το τετράγωνο, εκτός από τις ιδιότητες του τυχαίου παραλληλογράμμου, έχει επιπλέον τις ιδιότητες του
ορθογωνίου και του ρόμβου. Οι ιδιότητες λοιπόν του τετραγώνου διατυπώνονται ως εξής:
 Οι απέναντι πλευρές είναι ίσες και παράλληλες.
 Όλες οι γωνίες του είναι ορθές.
 Οι διαγώνιοί του είναι ίσες, διχοτομούνται, τέμνονται κάθετα και διχοτομούν τις γωνίες του.
Κριτήρια
Για να αποδείξουμε ότι ένα τετράπλευρο είναι τετράγωνο, αποδεικνύουμε ότι είναι παραλληλόγραμμο, έχει
μία χαρακτηριστική ιδιότητα του ορθογωνίου (μία γωνία ορθή ή ίσες διαγωνίους) και μία χαρακτηριστική
ιδιότητα του ρόμβου (δύο διαδοχικές πλευρές ίσες ή οι διαγώνιοι τέμνονται κάθετα ή μία διαγώνιος διχοτομεί
μία γωνία). Είναι προφανές ότι:
 Ένα ορθογώνιο με ίσες πλευρές ή κάθετες διαγωνίους είναι τετράγωνο.
 Ένας ρόμβος με μια ορθή γωνία ή με ίσες διαγωνίους είναι τετράγωνο.
1.5 Στις πλευρές ΑΒ, ΒΓ ενός τετραγώνου ΑΒΓΔ θεωρούμε αντίστοιχα τα σημεία Ε και Ζ έτσι, ώστε
οˆΕΔΖ = 45 . Να αποδειχθεί ότι:
α) ο κύκλος με κέντρο Δ και ακτίνα ΑΒ εφάπτεται με την ευθεία ΕΖ,
β) ΕΖ = ΑΕ + ΓΖ.
Γερμανία – 2009
Λύση
α) Θεωρούμε το σημείο Η έτσι, ώστε οˆ ˆΖΔΗ ΕΔΖ 45  .
Επειδή oˆΖΔΗ x y 45 x ω     , είναι y ω , δηλαδή:
ˆ ˆΑΔΕ ΓΔΗ και
Δ Δ
ΔΑΕ ΔΓΗ
αφού ΔΑ ΔΓ και ˆ ˆΑΔΕ ΓΔΗ . Άρα ΔΕ ΔΗ και ΑΕ ΓΗ . Έτσι τα
τρίγωνα ΔΖΕ και ΔΖΗ είναι ίσα, αφού:
ΔΖ κοινή, ΔΕ ΔΗ , οˆ ˆΖΔΕ ΖΔΗ 45 
Αν λοιπόν ΔΡ ΕΖ , τότε ΔΡ ΔΓ , αφού στα ίσα τρίγωνα ΔΖΕ, ΔΖΗ τα
ΔΡ, ΔΓ είναι ομόλογα ύψη. Επομένως, αφού ΔΡ α , ο κύκλος (Δ,α)
εφάπτεται με την ΕΖ στο Ρ.
β) Επειδή ΖΗ ΖΕ και ΓΗ ΑΕ , έχουμε:
ΕΖ ΖΗ ΖΓ ΓΗ ΖΓ ΑΕ    
1.6 Στις πλευρές ΒΓ, ΓΔ ενός τετραγώνου ΑΒΓΔ με ΑΒ = α , παίρνουμε τα σημεία Ε, Ζ αντίστοιχα. Αν η
περίμετρος του τριγώνου ΓΕΖ είναι ίση με 2α, να αποδειχθεί ότι οˆΕΑΖ = 45 .
Λύση
Στο ορθογώνιο τρίγωνο ΓΕΖ είναι:
 ΓΖ ΖΕ ΕΓ 2α  

ΓΖ ΖΕ ΕΓ
ΑΔ ΑΒ α
2
 
  
Άρα το Α είναι παράκεντρο της γωνίας ˆΓ του
Δ
ΓΕΖ . Αν λοιπόν
ΑΗ ΖΕ , τότε:
ο
ˆ ˆ ˆΔΑΗ ΗΑΒ ΔΑΒˆ ˆ ˆΖΑΕ ΖΑΗ ΗΑΕ 45
2 2 2
     
Μπορούμε επίσης απευθείας να πούμε ότι:
ο ο
ˆΓˆΖΑΕ 90 45
2
  
με βάση γνωστή ιδιότητα των διχοτόμων των γωνιών τριγώνου.
Σχόλιο
Αν αΙ είναι το παράκεντρο ορθογωνίου τριγώνου ΑΒΓ οˆ(Α 90 ) , τότε
είναι:
α
ΑΒ ΒΓ ΓΑ
ρ ΑΖ τ
2
 
  
Αντίστροφα, αν ισχύει η παραπάνω σχέση, τότε το αΙ είναι παράκεντρο του
Δ
ΑΒΓ .
Άλλος τρόπος
Στην προέκταση του ΓΒ παίρνουμε τμήμα ΒΗ ΔΖ . Είναι τότε
Δ Δ
ΑΒΗ ΑΔΖ , οπότε:
ΑΗ ΑΖ και οˆΖΑΗ 90
Είναι οπότε:
 ΕΖ 2α ΕΓ ΖΓ (α ΕΓ) (α ΖΓ) ΒΕ ΔΖ BE BH           
ΕΗ

Δ Δ
ΑΕΗ ΑΕΖ , διότι ΑΗ ΑΖ , ΕΗ ΕΖ και ΑΕ κοινή.
Άρα παίρνουμε:
ο
ο
ˆΖΑΗ 90ˆΖΑΕ 45
2 2
  
Άλλος τρόπος
Έστω ΓΕ x και ΓZ y . Είναι τότε:
 2 2
ΓΕ ΓΖ ΕΖ 2α x y x y 2α        
2 2 2
x y [2α (x y)]     
2 2 2 2
x y (α x) (α y) 2(α x)(α y)         
2 2 2 2 2 2
x y (α 2αx x ) (α 2αy y ) 2(α x)(α y)           
2
(α x)(α y) αx αy α      (1)

(1)
2 2 2
2
α x α y 2α x y
εφω εφφ α(2α x y)α α αεφ(ω φ) 1
α x α y1 εφω εφφ α (α x)(α y) α αx αy α1
α α α
   

  
     
         
.
Άρα ο
ω φ 45  , οπότε οˆΕΑΖ 45 .
1.7 Δίνεται τετράγωνο ΑΒΓΔ, το μέσο Μ του ΑΔ, το μέσο Ν του ΑΒ και σημείο Ρ στην προέκταση της
ΜΓ τέτοιο, ώστε ΓΡ = ΓΜ . Αν η ΡΝ τέμνει τη ΒΓ στο Κ, να αποδειχθεί ότι ΚΒ = ΚΓ .
Λύση
Έστω ότι η ΜΝ τέμνει τη ΒΓ στο Σ. Επειδή
Δ Δ
ΝΑΜ ΝΒΣ , είναι ΝΜ ΝΣ . Αλλά ΓΜ ΓΡ , οπότε στο
τρίγωνο ΡΜΣ το Κ είναι βαρύκεντρο. Αφού ΓΣ ΓΒ ΒΣ 3α   (έχουμε θέσει ΑΔ ΓΔ 2α  ), είναι:
1 1
ΓΚ ΓΣ 3α α
3 3
   
Άρα
ΒΓ
ΓΚ α
2
  , οπότε το Κ είναι μέσο του ΒΓ.
1.8 Στη διαγώνιο ΑΓ ενός τετραγώνου ΑΒΓΔ παίρνουμε σημείο Ε έτσι, ώστε οˆΓΒΕ = 30 . Στην προέκταση
της διαγωνίου ΑΓ, προς το Γ, παίρνουμε σημείο Ζ, ώστε ΓΖ = ΓΕ . Να αποδειχθεί ότι το τρίγωνο ΖΒΔ
είναι ισόπλευρο.
Λύση
Θεωρούμε το ισόπλευρο τρίγωνο ΒΓΗ. Αν η ευθεία ΔΗ
τέμνει την ΑΒ στο σημείο Θ, τότε:
 Το σημείο Η βρίσκεται στη μεσοκάθετη του ΒΓ, άρα
και του ΑΔ, και έτσι το Η είναι μέσο του ΔΘ, αφού
ΗΔ ΗΑ και οˆΑ 90 .
 Επειδή ΓΗ ΓΔ και η γωνία ˆΔΓΗ είναι ίση με ο
30 , η
γωνία ΑΔΘ είναι ίση με ο ο ο
90 75 15  . Άρα η γωνία
ΘΗΒ είναι ίση με ο
45 .
 Τα τρίγωνα ΒΓΕ και ΘΒΗ είναι ίσα, διότι ΒΓ ΒΗ και οι προσκείμενες σε αυτές γωνίες είναι ίσες (οι
γωνίες αυτές είναι ο
45 και ο
30 αντίστοιχα). Επομένως ΒΕ ΒΘ , ΕΓ ΗΘ και έτσι:
ΖΕ 2ΕΓ 2ΗΘ ΔΘ  
δηλαδή ΖΕ ΔΘ .
 Τα τρίγωνα ΘΒΔ και ΕΒΖ είναι τώρα ίσα, διότι ΘΔ ΕΖ , ΘΒ ΕΒ και οι γωνίες ˆΒΘΔ , ˆΒΕΖ είναι
ίσες με ο
105 η καθεμιά. Άρα ΒΔ ΒΖ . Αλλά ΒΖ ΖΔ , οπότε ΒΔ ΒΖ ΖΔ  και η απόδειξη
ολοκληρώθηκε.
Τη λύση οφείλω στον Ρώσο Μαθηματικό Mikhail Leptchinsky.
4Α . Βαρύκεντρο – Ορθόκεντρο
Θεωρήματα
α) Το ευθύγραμμο τμήμα που ενώνει τα μέσα δύο πλευρών ενός τριγώνου είναι
παράλληλο προς την τρίτη πλευρά και ισούται με το μισό της. Έτσι, αν στο διπλανό
σχήμα Μ και Ν είναι τα μέσα των πλευρών ΑΒ και ΑΓ αντίστοιχα, ισχύει ΜΝ // ΒΓ
και
ΒΓ
ΜΝ
2
 .
β) Αν μια ευθεία διέρχεται από το μέσο μιας πλευράς τριγώνου και είναι παράλληλη
προς κάποια πλευρά του, τότε η ευθεία αυτή διέρχεται και από το μέσο της τρίτης
πλευράς. Έτσι, αν στο διπλανό σχήμα το Μ είναι μέσο της ΑΒ και ε // ΒΓ, τότε το Ν
θα είναι μέσο και της πλευράς ΑΓ.
γ) Το τετράπλευρο που έχει κορυφές τα μέσα των πλευρών ενός τετραπλεύρου
(κυρτού ή μη κυρτού) είναι παραλληλόγραμμο.
Έτσι, με βάση το διπλανό σχήμα, το ΚΛΜΝ είναι παραλληλόγραμμο.
Θεώρημα
α) Οι διάμεσοι κάθε τριγώνου διέρχονται από το ίδιο σημείο το οποίο λέγεται
βαρύκεντρο (ή κέντρο βάρους).
β) Το βαρύκεντρο έχει μια πολύ σπουδαία ιδιότητα: χωρίζει την κάθε διάμεσο
σε δύο τμήματα από τα οποία το ένα είναι διπλάσιο του άλλου. Έτσι έχουμε:
 ΑΘ 2ΘΜ , ΓΘ 2ΘΚ και ΒΘ 2ΘΝ
 α
2
ΘΑ μ
3
 και α
1
ΘΜ μ
3

 β
2
ΘΒ μ
3
 και β
1
ΘΝ μ
3

 γ
2
ΘΓ μ
3
 και γ
1
ΘΚ μ
3

Τονίζουμε ότι αν Θ είναι σημείο της διαμέσου ΑΜ ενός τριγώνου ΑΒΓ και ΘΑ 2ΘΜ , τότε το Θ είναι το
βαρύκεντρο του τριγώνου αυτού.
Θεώρημα
α) Οι φορείς των υψών κάθε τριγώνου διέρχονται από το ίδιο σημείο το οποίο λέγεται ορθόκεντρο.
β) Για το ορθόκεντρο σημειώνουμε τα εξής:
 Αν το τρίγωνο είναι οξυγώνιο, τότε το ορθόκεντρο είναι εσωτερικό σημείο του τριγώνου.
 Αν το τρίγωνο είναι ορθογώνιο, τότε το ορθόκεντρο συμπίπτει με την κορυφή της ορθής γωνίας.
 Αν το τρίγωνο είναι αμβλυγώνιο, τότε το ορθόκεντρο είναι εξωτερικό σημείο του τριγώνου.
1.9 Δίνεται τρίγωνο ΑΒΓ, με οˆΑ < 90 , και εκτός αυτού τα ισοσκελή τρίγωνα ΑΒΕ, ΑΓΖ, με βάσεις ΑΒ,
ΑΓ αντίστοιχα. Αν Δ είναι το μέσο του ΒΓ και οˆΕΔΖ = 90 , να αποδειχθεί ότι οι γωνίες ˆΑΕΒ και
ˆΑΖΓ είναι παραπληρωματικές.
(JBMO – 2008, short list)
Λύση
Θεωρούμε το συμμετρικό Η του Ζ ως προς το μέσο Δ του ΒΓ. Το
ΒΗΓΖ είναι παραλληλόγραμμο, διότι οι διαγώνιες διχοτομούνται.
Επομένως:
ΒΗ ΖΓ ΖΑ 
Στο τρίγωνο ΕΗΖ η ΕΔ είναι διάμεσος και ύψος, οπότε αυτό είναι
ισοσκελές. Άρα ΕΗ ΕΖ και επειδή ΒΕ ΕΑ , είναι
Δ Δ
ΑΕΖ ΒΕΗ .
Είναι λοιπόν:
ˆˆΕΒΗ ΕΑΖ 
ο ˆˆ ˆ360 φ Β ΓΒΗ φ Α ω       
ο ˆˆ ˆ360 φ Β ΒΓΖ φ Α ω       
ο οˆ ˆˆ ˆ ˆ ˆ2φ ω (Α Β) (Γ ω) 360 2φ 2ω (Α Β Γ) 360             
ο ο
ο ο
ˆ ˆ180 ΑΕΒ 180 ΑΖΓ
φ ω 90 90
2 2
 
      
οˆ ˆΑΕΒ ΑΖΓ 180  
1.10 Δίνεται ισοσκελές τρίγωνο ΑΒΓ (ΑΒ = ΑΓ) και δύο εσωτερικά σημεία Δ, Ε της πλευράς ΑΓ τέτοια,
ώστε ΔΒ = ΔΕ και ˆ ˆΓΒΔ = ΑΒΕ . Αν Ο είναι το έγκεντρο του τριγώνου ΕΒΓ, να αποδειχθεί ότι
οˆΓΟΕ =120 .
(JBMO – 2006)
Λύση
Έστω ˆΔΕΒ 2ω . Επειδή ΔΕ ΔΒ , είναι ˆΔΒΕ 2ω . Αλλά:
ο ο
ˆΕΒΓ 2ω xˆΕΟΓ 90 90
2 2

    (1)
Στο τρίγωνο ΒΕΑ είναι:
ˆ ˆˆ ˆBEΓ ΒΑΕ ΕΒΑ 2ω Α x     
ο ˆ2ω (180 2Β) x    
o
2ω 180 2(2x 2ω) x     
ο o
6ω 180 3x 3x 6ω 180      
ο
x 2ω 60   (2)
Έτσι η (1) δίνει:
ο(2)
ο ο ο ο
ˆΕΒΓ x 2ω 60ˆΕΟΓ 90 90 90 120
2 2 2

      
Θεωρήματα
α) Η διάμεσος ορθογωνίου τριγώνου που αντιστοιχεί στην υποτείνουσα
είναι ίση με το μισό της υποτείνουσας. Έτσι, με βάση το διπλανό σχήμα,
ισχύει ότι:
ΒΓ
ΑΜ ΜΒ ΜΓ
2
  
β) Αν η διάμεσος ενός τριγώνου ισούται με το μισό της αντίστοιχης πλευράς, τότε το τρίγωνο αυτό είναι
ορθογώνιο με υποτείνουσα την πλευρά αυτή.
Έτσι, αν για το τρίγωνο ΑΒΓ του διπλανού σχήματος ισχύει:
ΑΜ ΜΒ ΜΓ 
τότε οˆΑ 90 .
Θεωρήματα
α) Αν σε ένα ορθογώνιο τρίγωνο μια γωνία του είναι ίση με ο
30 , τότε η απέναντι
πλευρά του είναι ίση με το μισό της υποτείνουσας.
β) Αν σε ένα ορθογώνιο τρίγωνο μία κάθετη πλευρά του είναι ίση με το μισό της
υποτείνουσας, τότε η γωνία που βρίσκεται απέναντι από την πλευρά αυτή είναι ίση με
ο
30 .
Πρόταση
Αν Μ είναι σημείο στην υποτείνουσα ΒΓ ενός ορθογωνίου τριγώνου ΑΒΓ, ώστε ΜΑ ΜΓ , τότε το Μ είναι
μέσο του ΒΓ.
Απόδειξη
Αφού ΜΑ ΜΓ , είναι ˆ ˆΜΑΓ ΜΓΑ φ  . Άρα:
 ο οˆ ˆΒ 90 Γ 90 φ   
 οˆ ˆ ˆΜΑΒ Α ΜΑΓ 90 φ   
Άρα οˆˆΒ ΜΑΒ 90 φ   , οπότε το τρίγωνο ΜΑΒ είναι ισοσκελές. Άρα:
ΜΒ ΜΑ ΜΓ 
1.11 Δίνεται τετράγωνο ΑΒΓΔ και Ρ εσωτερικό σημείο αυτού, ώστε οˆˆΡΓΔ = ΡΔΓ =15 . Να αποδειχθεί ότι
το τρίγωνο ΡΑΒ είναι ισόπλευρο.
Λύση
Έστω ΑΜ, ΓΝ ΡΔ . Είναι
Δ Δ
ΑΔΜ ΓΔΝ , οπότε:
ΓΝ ΔΜ
Αλλά οˆΓΡΝ 30 , οπότε:
ΡΓ ΡΔ
ΓΝ ΔΜ
2 2
  
Το Μ είναι λοιπόν μέσο του ΡΔ, οπότε ΑΔ ΑΡ . Έτσι:
ΡΒ ΡΑ ΑΔ ΑΒ  
1.12 Δίνεται ορθογώνιο τρίγωνο ΑΒΓ οˆ(Α = 90 ), το ύψος ΑΗ, η διχοτόμος ΒΜ της γωνίας ˆAΒΓ , που
τέμνει την ΑΗ στο Ρ, και η διχοτόμος ΓΝ της ˆΑΓΒ , που τέμνει την ΑΗ στο Ζ. Αν Κ, Λ είναι τα μέσα
των ΡΜ, ΖΝ, να αποδειχθεί ότι ΚΛ // ΒΓ.
(Κίνα – 2005)
Λύση
Προεκτείνουμε τις ΑΛ, ΑΚ μέχρι να συναντήσουν τη
ΒΓ. Τα τρίγωνα
Δ
ΑΡΜ και
Δ
ΑΖΝ είναι ισοσκελή, διότι:
ˆΒˆˆ ˆ ˆ ˆΑΡΜ ΡΑΒ ΡΒΑ Γ ΑΜΡ
2
    
και:
ˆΓˆˆ ˆ ˆ ˆΑΖΝ ΑΓΡ ΗΑΓ Β ΑΝΖ
2
    
Άρα ΑΛ ΓΝ και ΑΚ ΒΜ . Αλλά τότε τα Λ, Κ είναι μέσα των ΑΣ, ΑΤ, οπότε ΚΛ // ΣΤ ΚΛ // ΒΓ.
5Α . Τραπέζια
Ορισμός
α) Τραπέζιο λέγεται το κυρτό τετράπλευρο που έχει μόνο δύο πλευρές
παράλληλες.
Οι παράλληλες πλευρές λέγονται βάσεις του τραπεζίου.
β) Το ευθύγραμμο τμήμα που ενώνει τα μέσα των μη παράλληλων πλευρών του τραπεζίου λέγεται διάμεσος
του τραπεζίου.
γ) Η απόσταση των παράλληλων πλευρών του τραπεζίου λέγεται ύψος του τραπεζίου.
Θεωρήματα
α) Η διάμεσος του τραπεζίου είναι παράλληλη προς τις βάσεις του και ισούται
με το ημιάθροισμά τους. Ισχύει δηλαδή ότι:
 MN // AB // ΓΔ 
AB ΓΔ
MN
2


β) i) Το ευθύγραμμο τμήμα που ενώνει τα μέσα των διαγωνίων ενός τραπεζίου είναι παράλληλο προς τις
βάσεις του και ισούται με την ημιδιαφορά τους. Ισχύει δηλαδή ότι:
 ΚΛ // AB // ΓΔ 
ΓΔ AB
ΚΛ
2


ii) Η διάμεσος του τραπεζίου διέρχεται από τα μέσα των διαγωνίων του.
Ορισμός
Το τραπέζιο που έχει τις μη παράλληλες πλευρές ίσες, λέγεται
ισοσκελές.
Θεώρημα
Στο ισοσκελές τραπέζιο ισχύουν οι εξής ιδιότητες:
 οι γωνίες που πρόσκεινται στις βάσεις είναι ίσες, δηλαδή ˆˆΓ Δ και ˆ ˆA B ,
 οι διαγώνιες είναι ίσες, δηλαδή ΑΓ ΒΔ .
Κριτήρια
Για να είναι ένα τραπέζιο ισοσκελές, αρκεί να αποδείξουμε ότι:
 οι μη παράλληλες πλευρές είναι ίσες,
 οι γωνίες μιας βάσης είναι ίσες,
 οι διαγώνιοι είναι ίσες.
Μια χρήσιμη βοηθητική γραμμή
Ας υποθέσουμε ότι λύνοντας μια άσκηση εντοπίζουμε στο σχήμα ένα τραπέζιο
ΑΒΓΔ, με ΑΒ // ΓΔ, του οποίου μάλιστα υπάρχει και το μέσο Μ μιας μη
παράλληλης πλευράς (εδώ της ΑΔ). Αν με τα υπάρχοντα στοιχεία δεν
μπορούμε να επιτύχουμε λύση, τότε κάνουμε μία από τις επόμενες ενέργειες:
i) Θεωρούμε το μέσο Ν και της ΒΓ, οπότε ισχύουν:
 ΜΝ // ΑΒ και ΜΝ // ΓΔ

ΑΒ + ΓΔ
ΜΝ =
2
ii) Φέρνουμε ΜΝ // ΑΒ (ή ΜΝ // ΓΔ), οπότε το ΜΝ θα είναι διάμεσος του τραπεζίου και θα ισχύουν προφανώς
οι προηγούμενες σχέσεις.
iii) Θεωρούμε το συμμετρικό Σ του Β ως προς το Μ, οπότε το ΑΒΔΣ είναι παραλληλόγραμμο και τα σημεία Σ,
Δ, Γ είναι συνευθειακά.
1.13 Δίνεται ορθογώνιο τρίγωνο ΑΒΓ, το μέσο Μ της ΒΓ και τα σημεία Δ, Ε των πλευρών ΑΒ, ΑΓ
αντίστοιχα έτσι, ώστε οˆΔΜΕ = 90 . Από το Γ φέρουμε τμήμα ΓΝ ίσο και παράλληλο με το ΔΕ. Να
αποδειχθεί ότι ΝΜ ΒΓ .
Λύση
Έστω Κ το μέσο του ΔΕ και Λ το μέσο του ΝΓ. Επειδή το ΔΕΓΝ είναι
παραλληλόγραμμο, το ΚΛΓΕ είναι επίσης παραλληλόγραμμο. Έτσι
ΚΛ // ΑΓ. Έχουμε λοιπόν:
ΔΕ ΝΓ
ΑΚ ΛΓ
2 2
  
Επομένως το τραπέζιο ΚΑΓΛ είναι ισοσκελές. Είναι όμως ΜΑ ΜΓ ,
οπότε το Μ βρίσκεται στη μεσοκάθετο του ΑΓ, συνεπώς και στη
μεσοκάθετο του ΚΛ (αφού το ΚΛΓΑ είναι ισοσκελές τραπέζιο). Είναι
λοιπόν ΜΛ ΜΚ , οπότε:
ΔΕ ΝΓ
ΜΛ ΜΚ
2 2
  
Αφού στο τρίγωνο ΜΝΓ η ΜΛ είναι διάμεσος και
ΝΓ
ΜΛ
2
 , το τρίγωνο αυτό είναι ορθογώνιο. Άρα
ΝΜ ΒΓ .
1.14 Δίνεται ισοσκελές τρίγωνο ΑΒΓ (ΑΒ = ΑΓ) και σημείο Δ της διαμέσου ΑΜ. Σχηματίζουμε το
παραλληλόγραμμο ΑΒΔΕ. Να αποδειχθεί ότι ΕΓ ΒΓ .
(GM – 1997)
Λύση
Έστω Ζ η τέταρτη κορυφή του παραλληλογράμμου ΑΒΜΖ.
Επειδή:
ΔΕ ΑΒ ΜΖ  και ΔΕ // ΑΒ // ΜΖ
το ΔΕΖΜ είναι παραλληλόγραμμο. Άρα:
ΕΖ // ΔΜ
και επειδή ΔΜ ΒΓ , θα είναι και ΕΖ ΒΓ . Επίσης είναι:
ΑΖ / / ΒΜ / / ΜΓ 
οπότε και το τετράπλευρο ΑΖΓΜ είναι παραλληλόγραμμο.
Συνεπώς:
ΖΓ ΒΓ
Τα σημεία λοιπόν Ε, Ζ, Γ είναι συνευθειακά (αφού ΕΖ, ΖΓ // ΑΜ) και έτσι:
ΕΓ ΒΓ
1.15 Σε ένα ορθογώνιο τρίγωνο ΑΒΓ οˆ(Α = 90 ) είναι ΑΒ > ΑΓ . Στην πλευρά ΑΒ παίρνουμε σημείο Μ,
ώστε ΒΜ = ΑΓ . Στην πλευρά ΑΓ παίρνουμε σημείο Ν, ώστε ΓΝ = ΑΜ . Να αποδειχθεί ότι η γωνία
που σχηματίζουν οι ευθείες ΒΝ, ΓΜ είναι ίση με ο
45 .
(Βουλγαρία)
Λύση
Θεωρούμε το ορθογώνιο ΓΑΒΔ. Φέρνουμε ΒΕ // ΓΜ. Έστω ΒΜ α και ΜΑ β . Είναι:
 ΓΕ ΜΒ ΑΓ ΒΔ α   
 ΔΕ ΓΔ ΓΕ ΑΒ α ΑΜ ΓΝ β      
 Τα ορθογώνια τρίγωνα ΓΕΝ και ΔΕΒ είναι ίσα, οπότε:
ΕΒ ΕΝ και ˆ ˆΓΕΝ ΔΒΕ φ 
Επομένως:
οˆ ˆ ˆ ˆΓΕΝ ΔΕΒ ΔΒΕ ΔΕΒ 90   
Συνεπώς το τρίγωνο ΕΒΝ είναι ορθογώνιο και ισοσκελές,
οπότε:
οˆ ˆΕΝΒ ΕΒΝ 45 
Άρα οˆ ˆΜΙΒ ΙΒΕ 45  , διότι ΙΜ // ΕΒ.
1.16 Σε ένα τρίγωνο ΑΒΓ, με οˆΑ = 60 , φέρουμε τις διχοτόμους ΑΔ και ΒΕ. Αν ισχύει
ΑΒ + ΒΔ = ΑΕ + ΕΒ , να αποδειχθεί ότι οˆΓ = 40 .
(ΙΜΟ – 2001)
Λύση
Στις προεκτάσεις των ΑΒ, ΑΓ παίρνουμε τμήματα ΒΗ ΒΔ και ΕΖ ΕΒ . Επειδή ΒΗ ΒΔ , είναι:
ˆΑΒΔˆΒΗΔ φ
2
 
Είναι επίσης:
ΑΒ ΒΔ ΑΕ ΕΒ   
ΑΒ ΒΗ ΑΕ ΕΖ    
ΑΗ ΑΖ 
Επομένως το τρίγωνο ΑΗΖ είναι ισόπλευρο και
επειδή η ΑΔ είναι διχοτόμος της γωνίας ˆΑ , είναι:
ΔΗ ΔΖ και
ˆ ˆΔΖΑ ΔΗΑ φ 
Θα αποδείξουμε ότι το Ζ ταυτίζεται με το Γ. Αν το Ζ δεν συμπίπτει με το Γ, τότε στο τρίγωνο ΔΒΖ είναι:
ˆ ˆ ˆ ˆ ˆ ˆ ˆ ˆΔΒΖ ΕΒΖ ΕΒΔ ΕΖΒ ΒΗΔ ΕΖΒ ΔΖΑ ΔΖΒ      
οπότε ΔΒ ΔΖ , άρα και το τρίγωνο ΔΒΖ είναι ισοσκελές. Είναι λοιπόν ΔΗ ΔΖ ΔΒ  και επειδή
ΒΗ ΒΔ , το τρίγωνο ΒΔΗ είναι ισόπλευρο. Άρα:
ο ο οˆˆ ˆΑΒΔ ΒΗΔ ΒΔΗ 60 60 120    
άτοπο, διότι οˆΑ 60 . Άρα το Ζ συμπίπτει με το Γ (ανάλογα εργαζόμαστε αν το Ζ είναι ανάμεσα στα Ε, Γ).
Επειδή λοιπόν τα Γ, Ζ συμπίπτουν, είναι:

ˆΒ ˆΕΖ ΕΒ ΕΓ ΕΒ Γ
2
    
 ο οˆˆ ˆ ˆ ˆΒ Γ Α 180 Β Γ 120     
Έχουμε επομένως:
ο ο οˆ ˆ ˆ ˆ ˆΒ Γ 120 2Γ Γ 120 Γ 40      
B. Κύκλος και εγγράψιμα τετράπλευρα
1Α . Εγγεγραμμένες Γωνίες
Ορισμός
Μια γωνία που έχει την κορυφή της πάνω σε έναν κύκλο και οι πλευρές της
γωνίας τέμνουν τον κύκλο, λέγεται εγγεγραμμένη γωνία.
Έτσι, στο διπλανό σχήμα έχουμε τα εξής:
 Η γωνία ˆΣ είναι εγγεγραμμένη και βαίνει στο τόξο ΑΒ .
 Η γωνία ˆΑΟΒ είναι η αντίστοιχη επίκεντρη της γωνίας ˆΣ .
Θεωρήματα
α) Κάθε εγγεγραμμένη γωνία είναι ίση με το μισό της αντίστοιχης επίκεντρης γωνίας. Έτσι στο παραπάνω
σχήμα έχουμε:

1 ˆˆΣ ΑΟΒ
2
 ή ˆ ˆΑΟΒ 2Σ ,
 1ˆΣ ΑΒ
2
 , δηλαδή η εγγεγραμμένη γωνία ισούται με το μισό του αντίστοιχου τόξου.
β) Η εγγεγραμμένη γωνία που βαίνει σε ημικύκλιο (βλέπει διάμετρο) είναι ορθή.
Έτσι στο διπλανό σχήμα είναι:
οˆ ˆΜ Σ 90 
γ) Οι εγγεγραμμένες γωνίες του ίδιου ή ίσων κύκλων που βαίνουν στο ίδιο ή
σε ίσα τόξα είναι ίσες. Έτσι στο διπλανό σχήμα είναι:
ˆ ˆ ˆ ˆΜ Ν Σ P  
Θεώρημα
Η γωνία που σχηματίζεται από μια χορδή και την εφαπτομένη σε ένα άκρο της
ισούται με κάθε εγγεγραμμένη γωνία που βαίνει στο (κυρτό) τόξο της χορδής
αυτής. Έτσι στο διπλανό σχήμα ισχύουν οι σχέσεις:
 ˆ ˆM ABΔ φ  ,
 ˆ ˆΝ ΑΒΔ φ  ,
 ˆ ˆ ˆΜ Ν ΑΒΔ φ   .
Θεώρημα
Όταν δύο χορδές ενός κύκλου ή οι προεκτάσεις τους τέμνονται σε ένα σημείο,
τότε ισχύουν:

 ΑΓ ΒΔ
φ
2

 ,

 ΑΓ ΒΔ
ω
2

 .
Σημείωση
Ο γεωμετρικός τόπος των σημείων του επιπέδου από τα οποία ένα τμήμα ΑΒ
φαίνεται υπό γωνία φ είναι δύο τόξα κύκλων, χορδής ΑΒ, χωρίς τα άκρα τους,
συμμετρικά ως προς την ευθεία ΑΒ. Τα τόξα αυτά είναι:
 μείζονα (μη κυρτά), αν η φ είναι οξεία,
 ελάσσονα (κυρτά), αν η φ είναι αμβλεία.
1.1 Δίνεται ισόπλευρο τρίγωνο ΑΒΓ και τα σημεία Δ, Ε των ΑΒ, ΑΓ αντίστοιχα, ώστε ΔΕ // ΒΓ. Αν
Μ είναι το μέσο του τμήματος ΒΕ και Κ το περίκεντρο του τριγώνου ΑΔΕ, να αποδειχθεί ότι
οˆΚΜΓ = 90 .
(Ρωσία)
Λύση
Θεωρούμε το παραλληλόγραμμο ΕΓΒΖ. Τα Ζ, Δ, Ε
βρίσκονται στην ίδια ευθεία και το τρίγωνο ΖΔΒ είναι
ισόπλευρο, διότι:
ο ˆˆΖΒΔ 60 ΖΔΒ 
Τα τρίγωνα ΚΔΖ και ΚΕΓ είναι ίσα, διότι:
 ΚΔ ΚΕ , αφού το Κ είναι περίκεντρο,
 ΖΔ ΕΓ , διότι ΖΔ ZΒ ΕΓ  ,
 οˆ ˆΚΔΖ ΚΕΓ 150  , διότι οˆ ˆΚΔΕ ΚΕΑ 30  .
Είναι επομένως ΚΖ ΚΓ και αφού το Μ είναι μέσο του ΓΖ, είναι ΚΜ ΓΖ . Άρα οˆΚΜΓ 90 .
2Α . Εγγεγραμμένα – Εγγράψιμα Τετράπλευρα
Ορισμός
Ένα τετράπλευρο λέγεται εγγεγραμμένο σε κύκλο, αν οι κορυφές του είναι σημεία ενός κύκλου. Ο κύκλος
αυτός λέγεται περιγεγραμμένος κύκλος του τετραπλεύρου.
Θεώρημα
Σε κάθε εγγεγραμμένο τετράπλευρο ισχύουν οι παρακάτω ιδιότητες:
α) Οι απέναντι γωνίες είναι παραπληρωματικές. Έτσι:
οˆ ˆΑ Γ 180  , οˆˆΒ Δ 180 
β) Κάθε πλευρά του φαίνεται από τις απέναντι κορυφές υπό ίσες γωνίες, δηλαδή:
ˆ ˆΔΑΓ ΔΒΓ
γ) Κάθε εσωτερική γωνία ισούται με την απέναντι εξωτερική. Έτσι εξ
ˆ ˆΑ Γ , εξ
ˆˆΒ Δ .
Ορισμός
Ένα τετράπλευρο λέγεται εγγράψιμο όταν μπορεί να γραφεί κύκλος που να
διέρχεται από τις τέσσερις κορυφές του.
Κριτήρια
εγγραψιμότητας
Ένα τετράπλευρο είναι εγγράψιμο σε κύκλο, αν ισχύει μία από τις παρακάτω προτάσεις:
α) Δύο απέναντι γωνίες του είναι παραπληρωματικές. Έτσι, αν οˆ ˆΑ Γ 180  ,
τότε τα σημεία Α, Β, Γ και Δ είναι ομοκυκλικά (το ΑΒΓΔ είναι εγγράψιμο).
β) Μία πλευρά φαίνεται από τις απέναντι κορυφές υπό ίσες γωνίες. Έτσι, αν
ˆ ˆΔΑΓ ΔΒΓ , τότε το ΑΒΓΔ είναι εγγράψιμο.
γ) Μια εξωτερική του γωνία είναι ίση με την απέναντι εσωτερική. Έτσι, αν
εξ
ˆ ˆΑ Γ , τότε το ΑΒΓΔ είναι εγγράψιμο.
Ορισμός
Ένα τετράπλευρο του οποίου οι πλευρές εφάπτονται στον ίδιο κύκλο λέγεται περιγεγραμμένο στον κύκλο
αυτό, ενώ ο κύκλος λέγεται εγγεγραμμένος.
Θεώρημα
Σε κάθε περιγεγραμμένο τετράπλευρο ισχύουν οι εξής ιδιότητες:
 Οι διχοτόμοι των γωνιών του διέρχονται από το ίδιο σημείο, που είναι το
κέντρο του εγγεγραμμένου κύκλου.
 Τα αθροίσματα των απέναντι πλευρών είναι ίσα, δηλαδή:
ΑΒ ΓΔ ΑΔ ΒΓ  
Ορισμός –
Κριτήρια
Αν ένα τετράπλευρο μπορεί να περιγραφεί σε κύκλο, τότε λέγεται περιγράψιμο σε κύκλο. Για να είναι ένα
τετράπλευρο περιγράψιμο, αρκεί να ισχύει μία από τις ακόλουθες προτάσεις:
 Οι διχοτόμοι των γωνιών του διέρχονται από το ίδιο σημείο.
 Τα αθροίσματα των απέναντι πλευρών του είναι ίσα.
1.2 Θεωρούμε ορθογώνιο και ισοσκελές τρίγωνο ΑΒΓ, με οˆΑ = 90 , και το ύψος του ΑΔ. Η διχοτόμος
ΒΕ του τριγώνου ΑΒΓ τέμνει την ΑΔ στο Κ. Αν Ι είναι το έγκεντρο του τριγώνου ΑΔΓ, να
αποδειχθεί ότι ΙΚ = ΙΕ .
Λύση
Το Κ είναι έγκεντρο του τριγώνου ΑΒΓ, οπότε το Ι βρίσκεται
πάνω στην ΚΓ. Αν ΚΛ ΑΓ , τότε:
ΚΔ ΚΛ και
Δ Δ
ΚΔΙ ΚΛΙ
αφού ˆ ˆΔΚΓ ΛΚΓ . Επομένως:
οˆ ˆΚΛΙ ΚΔΙ 45 
δηλαδή οˆ ˆΙΛΕ 45 ΙΚΕ  . Άρα το ΙΚΛΕ είναι εγγράψιμο και
έτσι:
οˆˆ ˆΙΕΚ ΙΛΚ 45 ΙΚΕ  
Άρα το τρίγωνο ΙΚΕ είναι ισοσκελές, οπότε ΙΚ ΙΕ .
Σχόλιο
Ας παρατηρήσουμε ακόμα ότι οˆˆΚΙΕ = ΚΛΑ = 90 , δηλαδή ΙΚ ΙΕ και οˆΙΕΒ = 45 .
Άλλος τρόπος
Αν η ΓΚ τέμνει την ΑΒ στο Ζ, τότε το ΒΖΕΓ είναι ισοσκελές τραπέζιο. Έτσι:
ο
45ˆ ˆ ˆΕΑΙ ΖΓΒ ΕΖΙ φ
2
   
Άρα το ΑΖΙΕ είναι εγγράψιμο, οπότε:
οˆˆΕΙΓ Α 90 
Όμως:
οˆ ˆ ˆΕΚΙ ΚΒΓ ΚΓΒ 45  
οπότε το ορθογώνιο τρίγωνο ΙΚΕ είναι και ισοσκελές. Επομένως ΙΚ ΙΕ και οˆΙΕΚ 45 .
1.3 Δίνεται τρίγωνο ΑΒΓ, το ύψος ΑΔ και η διάμεσος ΑΜ. Αν ˆ ˆΒΑΔ = ΜΑΓ , να αποδειχθεί ότι το
τρίγωνο ΑΒΓ είναι ορθογώνιο.
Λύση
Έστω Ε, Ζ τα συμμετρικά του Α ως προς τα σημεία Δ, Μ αντίστοιχα. Τότε:
ˆ ˆΒΑΔ ΒΕΑ και ˆ ˆΜΑΓ ΑΖΒ
διότι το τετράπλευρο ΑΒΖΓ είναι παραλληλόγραμμο, μια και οι
διαγώνιες διχοτομούνται. Επειδή από την υπόθεση είναι
ˆ ˆΒΑΔ ΜΑΓ , θα είναι και ˆ ˆΒΕΑ ΑΖΒ . Αυτό εξασφαλίζει ότι
το τετράπλευρο ΑΒΕΖ είναι εγγράψιμο, οπότε:
ˆ ˆΑΒΖ ΑΕΖ
Στο τρίγωνο ΑΕΖ το τμήμα ΔΜ ενώνει μέσα, οπότε ΔΜ // ΕΖ.
Επειδή ΜΔ ΑΕ , θα είναι και ΕΖ ΑΕ . Άρα οˆΑΕΖ 90 , δηλαδή οˆΑΒΖ 90 . Το παραλληλόγραμμο
λοιπόν ΑΒΖΓ είναι ορθογώνιο, οπότε οˆΑ 90 .
1.4 Στο εσωτερικό ενός παραλληλογράμμου ΑΒΓΔ υπάρχει σημείο Κ τέτοιο, ώστε οˆΑΚΔ =120 και
οˆΒΚΓ = 60 . Να αποδειχθεί ότι οι περιγεγραμμένοι κύκλοι των τριγώνων ΚΑΔ και ΚΒΓ είναι
ίσοι.
Λύση
Θεωρούμε το σημείο Ε, ώστε ΔΕ // ΑΚ και ΔΕ ΑΚ , όπως στο σχήμα. Είναι τότε:
 ΑΔ ΚΕ και
Δ Δ
ΚΑΔ ΚΔΕ
 ΚΕ ΑΔ ΒΓ  και
Δ Δ
ΚΒΓ ΚΓΕ , διότι το τετράπλευρο ΚΕΓΒ
είναι παραλληλόγραμμο.
 οˆ ˆΚΔΕ ΔΚΑ 120  και οˆ ˆΚΓΕ ΓΚΒ 60 
Το τετράπλευρο ΚΔΕΓ είναι εγγράψιμο, διότι:
ο ο οˆ ˆΚΔΕ ΚΓΕ 120 60 180   
Αφού λοιπόν τα τρίγωνα ΚΔΕ και ΚΕΓ έχουν τον ίδιο περιγεγραμμένο κύκλο, συμπεραίνουμε ότι τα τρίγωνα
ΚΑΔ, ΚΒΓ έχουν ίσους περιγεγραμμένους κύκλους.
Σχόλιο
Επειδή
Δ Δ
ΚΑΒ = ΕΔΓ , προκύπτει ότι και τα τρίγωνα ΚΑΒ, ΚΓΔ έχουν ίσους περιγεγραμμένους κύκλους με τα
τρίγωνα ΚΑΔ, ΚΒΓ.
1.5 Στο εσωτερικό ενός τριγώνου ΑΒΓ υπάρχει σημείο Δ, ώστε οˆ ˆΔΑΓ = ΔΓΑ = 30 και οˆΔΒΑ = 60 .
Στο τμήμα ΑΓ παίρνουμε σημείο Ζ τέτοιο, ώστε ΑΖ = 2ΖΓ . Αν Ε είναι το μέσο του ΒΓ, να
αποδειχθεί ότι ΔΕ ΕΖ .
(Κίνα – 2007, Girls Olympiad)
Λύση
Έστω ΒΗ η διχοτόμος της γωνίας ˆΑΒΔ . Επειδή οˆ ˆΔΑΗ 30 ΔΒΗ  , το τετράπλευρο ΑΒΔΗ είναι
εγγράψιμο. Επομένως:
 οˆ ˆˆΗΔΑ ΗΒΑ ΗΑΔ 30   , οπότε ΗΔ ΗΑ
 οˆ ˆΔΗΖ ΑΒΔ 60  , οπότε το τρίγωνο ΗΔΓ είναι ορθογώνιο
 2ΑΗ 2ΔΗ ΗΓ  και επειδή ΑΖ 2ΖΓ , είναι ΑΗ ΗΖ ΖΓ 
Πραγματικά, έχουμε:
ΗΓ 2ΑΗ , ΑΓ 3ΑΗ , ΑΓ 3ΖΓ
οπότε
ΑΓ
ΑΗ ΖΓ
3
  και έτσι:
ΑΓ
ΗΖ ΑΗ ΖΓ
3
   .
Αν Θ είναι το μέσο του ΑΓ, τότε ΕΘ // ΑΒ και ΕΖ // ΒΗ. Άρα:
ο ˆˆ ˆΘΕΖ ΑΒΗ 30 ΘΔΖ  
Επομένως το ΔΘΖΕ είναι εγγράψιμο και αφού οˆΔΘΖ 90 , είναι οˆΔΕΖ 90 .
Τονίζουμε ότι δεν γνωρίζουμε εξαρχής ότι τα Β, Δ, Ζ είναι συνευθειακά, ούτε χρησιμοποιούμε πουθενά στη
λύση αυτόν τον ισχυρισμό. Τελικά όμως, επειδή ΗΖ ΖΓ ΗΔ  και οˆΗΔΖ 60 , τα σημεία Β, Δ, Ζ είναι
όντως συνευθειακά.
Σημείωση
Δύο άλλες λύσεις υπάρχουν στο βιβλίο Mathematical Olympiad in China, 2007 – 2008, σελ. 116.
3Α . Θεωρήματα και Εφαρμογές στον Κύκλο
Θεώρημα 1ο
Αν η διχοτόμος της γωνίας ˆΑ τριγώνου ΑΒΓ και η μεσοκάθετη της πλευράς ΒΓ τέμνονται στο
σημείο Ε, τότε το Ε είναι σημείο του περιγεγραμμένου κύκλου του τριγώνου ΑΒΓ.
(Θεώρημα του Νότιου Πόλου)
Απόδειξη
Έστω ότι η μεσοκάθετος της ΒΓ τέμνει τον περιγεγραμμένο κύκλο C του
τριγώνου ΑΒΓ στο Ε. Τότε το Ε είναι μέσο του τόξου ΒΓ , οπότε η ΑΕ είναι
διχοτόμος της γωνίας ˆΑ του
Δ
ΑΒΓ . Επειδή το Ε είναι το μοναδικό μέσο του
τόξου ΒΓ , η απόδειξη έχει ολοκληρωθεί.
1.6 Σε ένα μη ισοσκελές τρίγωνο ΑΒΓ φέρουμε τις διχοτόμους ΑΛ και ΒΚ. Η μεσοκάθετος του ΒΚ
τέμνει την ΑΛ στο σημείο Μ. Η παράλληλη από το Λ προς τη ΜΚ τέμνει τη ΒΚ στο σημείο Ν. Να
αποδειχθεί ότι NA = NΛ .
(JBMO – 2010)
Λύση
Στο τρίγωνο ΑΒΚ η ΑΜ είναι διχοτόμος της γωνίας ˆA . Επειδή η ΜΡ είναι μεσοκάθετος του ΒΚ, σύμφωνα
με το θεώρημα του Νότιου Πόλου, το Μ βρίσκεται στον
περιγεγραμμένο κύκλο του τριγώνου
Δ
ΑΒΚ . Το
τετράπλευρο λοιπόν ΑΒΜΚ είναι εγγράψιμο, οπότε:
ˆ ˆ ˆ ˆΑΛN AΜΚ ΑΒΚ ΑΒN  
Επειδή ˆ ˆΑΛN ΑΒN , το τετράπλευρο ΑΒΛΝ είναι
επίσης εγγράψιμο και επειδή η ΒΝ είναι διχοτόμος της
γωνίας ˆΒ , θα είναι  ΝΑ ΝΛ (στον κύκλο (Α, Β, Λ, Ν)).
Άρα NA NΛ .
Μπορούμε βέβαια, να εργαστούμε και ως εξής:
ˆ ˆ ˆˆ ˆΛΑΝ ΛΒΝ ΝΒΑ ΝΛΑ ΑΛΝ   
Άρα ˆ ˆΛΑN ΑΛN , οπότε το τρίγωνο ΝΑΛ είναι ισοσκελές. Επομένως NA NΛ .
Θεώρημα 2ο
Έστω Ι το έγκεντρο ενός τριγώνου ΑΒΓ. Αν η ευθεία ΑΙ τέμνει τον περιγεγραμμένο κύκλο του
τριγώνου ΑΒΓ στο σημείο Μ, τότε ΜΒ = ΜΙ = ΜΓ .
Απόδειξη
Η ΑΙ είναι διχοτόμος της γωνίας ˆΑ του τριγώνου ΑΒΓ, οπότε το Μ είναι το
μέσο του τόξου ΒΓ . Επομένως ΜΒ ΜΓ . Θα αποδείξουμε λοιπόν ότι
ΜΒ ΜΙ . Είναι:

ˆΑˆ ˆΙΑΒ ΙΑΓ φ
2
   ,

ˆΒˆ ˆΙΒΑ ΙΒΓ ω
2
   ,
 ˆˆ ˆΒΙΜ ΙΑΒ ΙΒΑ φ ω    , ˆ ˆ ˆΙΒΜ ΙΒΓ ΓΒΜ ω φ    .
Είναι λοιπόν ˆ ˆΒΙΜ ΙΒΜ φ ω   , οπότε ΜΒ ΜΙ . Άρα ΜΒ ΜΙ ΜΓ  .
Σχόλιο
Αν Μ είναι το μέσο του τόξου ΒΓ και Ι σημείο του τμήματος ΑΜ, ώστε ΜΙ = ΜΒ , τότε το Ι είναι το έγκεντρο
του τριγώνου ΑΒΓ.
Πρόταση 3η
Έστω Ι το έγκεντρο ενός τριγώνου ΑΒΓ και σημείο Ε στην προέκταση του ΑΙ, προς το Ι, ώστε
ΕΙ = ΕΒ. Να αποδειχθεί ότι το Ε είναι σημείο του περιγεγραμμένου κύκλου του τριγώνου ΑΒΓ.
Απόδειξη
Έχουμε ΕΙ ΕΒ , οπότε το τρίγωνο ΕΒΙ είναι ισοσκελές. Άρα:
φ ω ρ ω
ˆˆ ˆ ˆ ˆ ˆΕΙΒ ΕΒΙ Α Β Β Β     
ˆ ˆφ ω ρ ω φ ρ ΕΑΓ ΕΒΓ       
Άρα το τετράπλευρο ΑΒΕΓ είναι εγγράψιμο, δηλαδή το Ε είναι σημείο του
περιγεγραμμένου κύκλου του τριγώνου ΑΒΓ.
Θεώρημα
4ο
Ο εγγεγραμμένος κύκλος ενός τριγώνου ΑΒΓ και ο παρεγγεγραμμένος κύκλος, που αντιστοιχεί
στη γωνία ˆA , εφάπτονται με την πλευρά ΒΓ στα σημεία Δ και Ε. Να αποδειχθεί ότι BΔ = ΓΕ .
Απόδειξη
Γνωρίζουμε ότι:
ΒΔ τ β  και ΓΕ ΓΖ τ β  
Άρα:
ΒΔ ΓΕ τ β  
Σχόλιο
Το μέσο Μ της πλευράς ΒΓ είναι και μέσο του τμήματος ΔΕ, διότι ΜΒ = ΜΓ και ΒΔ = ΓΕ .
ΒΑΣΙΚΟ ΣΧΗΜΑ
Στο παρακάτω σχήμα βλέπουμε το έγκεντρο και τα παράκεντρα ενός τριγώνου ΑΒΓ, καθώς και τα σημεία
επαφής των παρεγγεγραμμένων κύκλων (Κ), (Λ), (Μ) με τις πλευρές του τριγώνου ΑΒΓ. Στο σχήμα αυτό
ισχύουν οι παρακάτω βασικές ιδιότητες:
α) ΑΝ ΑΗ τ α   , ΒΝ ΒΔ τ β   , ΓΔ ΓΗ τ γ   .
β)  1ΑΑ ΑΘ τ β   , 2ΑΑ ΑΖ τ γ  
 1ΒΒ ΒΘ τ α   , 2ΒΒ ΒΕ τ γ  
 1ΓΓ ΓΖ τ α   , 2ΓΓ ΓΕ τ β  
γ) ΑΘ ΒΝ , ΒΔ ΓΕ , ΓΖ ΑΗ .
δ) ΡΛ ΡΜ , ΣΛ ΣΚ , TK TM (κύκλος Euler).
ε) Ο περιγεγραμμένος κύκλος του τριγώνου ΑΒΓ διχοτομεί τα τμήματα ΙΚ, ΙΛ, ΙΜ.
1.7 Σε ένα τετράπλευρο ΑΒΓΔ είναι ΑΒ = ΒΓ και οˆˆΑΒΓ + 2ΑΔΓ =180 . Αν Ε είναι το μέσο του ΑΓ, να
αποδειχθεί ότι ˆ ˆΓΔΕ = ΒΔΑ .
(Ρουμανία, IMAR – 2009)
Λύση
Στην προέκταση της ΒΕ παίρνουμε σημείο Ζ τέτοιο, ώστε:
ˆˆΓΖΕ ΓΔΕ x 
Το τετράπλευρο ΕΓΖΔ είναι εγγράψιμο, οπότε:
ˆ ˆΔZΕ ΔΓΕ (1)
Είναι όμως:
 ο ο1ˆ ˆˆ ˆΑΒΓ 2ΑΔΓ 180 ΑΔΓ (180 ΑΒΓ)
2
     
ο
ˆΑΒΓˆΑΔΓ 90
2
   (2)
 οˆˆ ˆΑΒΓ ΒΑΓ ΒΓΑ 180   
ο
ˆΑΒΓˆΒΑΓ 90
2
   (3)
Επομένως παίρνουμε:
(1)
ο ˆ ˆˆ ˆΔΖΕ ΔΓΕ 180 ΓΔΑ ΓΑΔ    
(3)
ο ο οˆ ˆ ˆ ˆ ˆ180 ΓΑΒ ΓΑΔ 180 (ΓΑΒ ΓΑΔ) 180 ΒΑΔ       
Είναι λοιπόν οˆˆΔΖΕ ΒΑΔ 180  , δηλαδή οˆˆΔΖΒ ΔΑΒ 180  , οπότε το τετράπλευρο ΑΒΖΔ είναι εγγράψιμο.
Έτσι:
ˆ ˆˆ ˆΒΔΑ ΒΖΑ ΓΖΕ ΓΔΕ  
διότι η ΖΕ είναι μεσοκάθετος του ΑΓ και έτσι ˆ ˆ ˆΒΖΑ ΒΖΓ ΕΖΓ  .
Άλλος τρόπος
Έστω 1C , 2C , οι περιγεγραμμένοι κύκλοι των τριγώνων ΑΒΓ και
ΑΓΔ αντίστοιχα. Επειδή:
οˆ ˆ ˆAΟΓ 2ΑΔΓ 180 ΑΒΓ  
το ΑΒΓΟ είναι εγγράψιμο. Επειδή ΟΑ ΟΓ και ΑΒ ΑΓ , το Ο
βρίσκεται στη ΒΕ και η ΒΟ είναι διάμετρος του κύκλου 1C . Άρα:
οˆ ˆOAB OΓΒ 90 
Αυτό σημαίνει ότι οι ΑΒ, ΓΒ είναι εφαπτομένες του 2C στα Α, Γ και
έτσι η ΔΒ είναι συμμετροδιάμεσος του
Δ
ΔAΓ , οπότε θα είναι ˆ ˆΑΔΒ ΓΔΕ .
1.8 Δίνεται τετράπλευρο ΑΒΓΔ, με ΑΒ = ΑΔ και οˆˆΒ = Δ = 90 . Στην πλευρά ΓΔ παίρνουμε σημείο Ε
και στην πλευρά ΒΓ σημείο Ζ τέτοια, ώστε ΑΕ ΔΖ . Να αποδειχθεί ότι ΑΖ ΒΕ .
(Ρωσία – 1995)
Λύση
Έστω Η το συμμετρικό του Ε ως προς τη διαγώνιο ΑΓ. Προφανώς
τα τρίγωνα ΑΒΓ και ΑΔΓ είναι ίσα και έτσι το Η ανήκει στη ΒΓ.
Ας είναι Κ το σημείο τομής των ΑΕ, ΔΖ και Λ το σημείο τομής
των ΑΖ και ΒΕ. Φέρνουμε τις ΚΛ, ΑΗ και ΔΗ. Είναι:
x y o ˆ( 90 ΔΑΕ)  και y ω
(λόγω συμμετρίας)
δηλαδή x ω . Επομένως το ΑΔΗΖ είναι εγγράψιμο, οπότε φ ρ
(βλέπουν το ΑΔ) και ρ ν (λόγω συμμετρίας ως προς την ΑΓ).
Επειδή φ ν , το ΚΛΖΕ είναι επίσης εγγράψιμο, οπότε οˆ ˆΕΛΖ ΕΚΖ 90  . Άρα ΒΕ ΑΖ .
Θεώρημα 5ο
Αν ρ, αρ , βρ , γρ είναι οι ακτίνες του εγγεγραμμένου και των παρεγγεγραμμένων κύκλων ενός
τριγώνου ΑΒΓ, τότε α β γρ + ρ + ρ = ρ + 4R , όπου R είναι η ακτίνα του περιγεγραμμένου κύκλου
του τριγώνου ΑΒΓ.
Απόδειξη
Έστω Δ, Ε, Ζ, Η οι προβολές των Ι, Μ, Κ, Λ στην ευθεία ΒΓ.
Σύμφωνα με βασικό θεώρημα είναι:
BΔ ΓΖ τ β   , ΒΕ ΓΗ τ α  
Αν Ρ είναι το μέσο της ΒΓ, τότε το Ρ είναι και μέσο του ΔΖ.
Η μεσοκάθετος της ΒΓ τέμνει την ΑΙ στο Τ, οπότε το Τ είναι
σημείο του περιγεγραμμένου κύκλου του τριγώνου ΑΒΓ
(θεώρημα Νότιου Πόλου). Είναι επίσης ΡΕ ΡΗ , οπότε η
μεσοκάθετος του ΕΗ, δηλαδή του ΒΓ, διέρχεται από το μέσο
Σ του τμήματος ΜΛ (λόγω του τραπεζίου ΜΕΗΛ). Όμως από
το μέσο Σ του ΜΛ διέρχεται ο περιγεγραμμένος κύκλος του
Δ
ΑΒΓ μια και αυτός είναι ο κύκλος Euler για το
τρίγωνο ΚΛΜ (καθόσον το Ι είναι ορθόκεντρο στο
Δ
ΚΛΜ ).
 Στο τραπέζιο ΙΔΚΖ είναι:
αρ ρΚΖ ΙΔ
ΤΡ
2 2

 
 Στο τραπέζιο ΜΕΗΛ είναι:
γ βρ ρΜΕ ΛΗ
ΡΣ
2 2

 
Με πρόσθεση κατά μέλη παίρνουμε:
α γ βρ ρ ρ ρ
ΤΡ ΡΣ
2
  
 
Αλλά ΤΡ ΡΣ 2R  , αφού η μεσοκάθετος μιας χορδής ορίζει με τον κύκλο διάμετρο. Άρα:
α γ β
α β γ
ρ ρ ρ ρ
2R ρ ρ ρ ρ 4R
2
  
     
Σχόλιο
Το γεγονός ότι το μέσο Σ του ΜΛ ανήκει στον περιγεγραμμένο κύκλο του
Δ
ΑΒΓ προκύπτει και ως εξής:
Φέρουμε τη ΓΣ. Στο ορθογώνιο τρίγωνο ΓΜΛ η ΓΣ είναι διάμεσος, οπότε ΓΣ = ΣΛ και έτσι ˆ ˆΣΛΓ = ΣΓΛ .
Άρα:
ο οˆ ˆ ˆˆΓΣΛ =180 2ΣΛΓ =180 2(ΣΛΒ + ΒΛΓ) = 
ο
ο ο
ˆˆ ˆΓ Α 180 Β ˆ=180 2 + =180 2 = Β
2 2 2
 
  
 

  
Επομένως το τετράπλευρο ΑΒΓΣ είναι εγγράψιμο. Θυμίζουμε ότι από βασική πρόταση στις διχοτόμους
τριγώνου, είναι
ˆΑˆΒΛΓ =
2
και
ˆΓˆΒΛΑ =
2
.
1.9 Σε ένα τρίγωνο ΑΒΓ (ΑΒ < ΑΓ) φέρουμε το ύψος ΒΔ και τη διάμεσο ΒΜ. Έστω Κ, Λ οι
προβολές των σημείων Α, Γ πάνω στη διχοτόμο της γωνίας ˆΒ . Να αποδειχθεί ότι:
α) τα σημεία Κ, Μ, Λ, Δ ανήκουν σε κύκλο C,
β) το κέντρο Σ του C ανήκει στον περιγεγραμμένο κύκλο του τριγώνου ΡΔΜ, όπου Ρ είναι το
μέσο του τμήματος ΑΒ.
Λύση
α) Αν η ΑΚ τέμνει τη ΒΓ στο Ν, τότε το τρίγωνο ΒΑΝ είναι
ισοσκελές, διότι η διχοτόμος ΒΚ είναι και ύψος. Έτσι το Κ είναι
μέσο του ΑΝ, οπότε ΚΜ // ΓΝ (από το τρίγωνο ΑΝΓ). Επομένως:
ˆ ˆΒΓΑ ΡΜΑ x y  
Αλλά το τετράπλευρο ΒΔΛΓ είναι εγγράψιμο, διότι:
οˆ ˆBΔΓ ΒΛΓ 90 
Έτσι x ω , που σημαίνει τελικά ότι y ω . Άρα τα σημεία Δ, Κ, Μ, Λ είναι ομοκυκλικά.
β) Έστω Σ το κέντρο του κύκλου (Δ, Κ, Μ, Λ). Τότε:
ˆ ˆˆ ˆΔΣΜ 2ΔΛΜ 2(ω φ) 2ω 2φ 2Γ 2ΚΔΜ       
ˆ ˆ2Γ 2ΑΒΚ  (το ΒΑΔΚ είναι εγγράψιμο, οπότε ˆ ˆΚΔΜ ΑΒΚ ) ˆ ˆ2Γ Β 
αφού η ΒΚ είναι διχοτόμος της ˆΒ . Επίσης είναι:
ο ˆˆ ˆ ˆ ˆΔΡΜ ΑΡΜ ΑΡΔ Β (180 2Α)     (διότι ΡΜ // ΒΓ και ΡΔ ΡΑ)
ο ο o οˆ ˆ ˆ ˆ ˆˆ ˆ ˆ ˆΒ 2Α 180 Α (Α Β) 180 A (180 Γ) 180 A Γ            
Είναι λοιπόν:
οˆ ˆˆ ˆ ˆ ˆ ˆ ˆ ˆΔΣΜ ΔΡΜ (2Γ Β) (Α Γ) Α Β Γ 180        
οπότε το τετράπλευρο ΔΡΜΣ είναι εγγράψιμο. Αυτό σημαίνει ότι το Σ ανήκει στον περιγεγραμμένο κύκλο
του τριγώνου ΡΔΜ (δηλαδή στον κύκλο Euler του τριγώνου ΑΒΓ).
Θεώρημα 6ο
Έστω Ο το περίκεντρο ενός οξυγώνιου τριγώνου ΑΒΓ και x, y, z οι αποστάσεις του Ο από τις
πλευρές ΒΓ, ΑΓ, ΑΒ αντίστοιχα. Να αποδειχθεί ότι:
x + y + z = R + ρ
όπου R, ρ είναι οι ακτίνες του περιγεγραμμένου και του εγγεγραμμένου κύκλου αντίστοιχα στο
τρίγωνο ΑΒΓ.
Απόδειξη
Θεωρούμε το παράκεντρο Κ του τριγώνου ΑΒΓ. Έστω Ι το έγκεντρο, ΙΔ, ΚΕ ΒΓ και Ρ το μέσο της ΒΓ.
Είναι γνωστό ότι:
ΒΔ ΓΕ τ β  
οπότε ΡΔ ΡΕ . Αφού ΟΡ ΒΓ , η ΟΡ θα περάσει από το μέσο
Ν του ΙΚ, το οποίο είναι (από βασική πρόταση) σημείο του
περιγεγραμμένου κύκλου του τριγώνου ΑΒΓ. Είναι λοιπόν:
KE IΔ
x OP ON PN R
2

     
α α
1 1
R (ρ ρ) (2R ρ ρ)
2 2
     
Όμοια είναι:
β
1
y (2R ρ ρ)
2
   και γ
1
z (2R ρ ρ)
2
  
Άρα, με πρόσθεση κατά μέλη, παίρνουμε:
α β γ
1
x y z (6R ρ ρ ρ 3ρ)
2
      
Γνωρίζουμε όμως ότι α β γρ ρ ρ ρ 4R    , οπότε:
 
1 1
x y z 6R (ρ 4R) 3ρ (2R 2ρ) R ρ
2 2
         
Σημείωση
Πρέπει να τονίσουμε με έμφαση ότι ο τύπος αυτός ισχύει μόνο στα οξυγώνια τρίγωνα. Αν π.χ. οι γωνίες ˆΑ , ˆΒ ,
ˆΓ είναι αντίστοιχα αμβλείες, τότε ισχύει:
-x + y + z = R + ρ , x - y + z = R + ρ , x + y - z = R + ρ
Θεώρημα 7ο
Θεωρούμε τετράπλευρο ΑΒΓΔ εγγεγραμμένο σε κύκλο. Αν 1ρ , 2ρ είναι οι ακτίνες των
εγγεγραμμένων κύκλων, αντίστοιχα, στα τρίγωνα ΑΒΓ, ΑΓΔ και 3ρ , 4ρ οι ακτίνες των
εγγεγραμμένων κύκλων στα τρίγωνα ΒΓΔ, ΒΑΔ, τότε 1 2 3 4ρ + ρ = ρ + ρ .
(Θεώρημα των Ιαπώνων)
Απόδειξη
Ας σημειώσουμε με α, β, γ, δ, x, y τις αποστάσεις του Ο από τις πλευρές ΑΒ, ΒΓ, ΓΔ, ΔΑ και από τις διαγώνιες
ΑΓ, ΒΔ αντίστοιχα. Με βάση το προηγούμενο θεώρημα είναι:
1α β x R ρ    (1)
2γ δ x R ρ    (2)
3β γ y R ρ    (3)
4α δ y R ρ    (4)
Προσθέτουμε τις σχέσεις (1), (2) και έχουμε:
1 22R ρ ρ α β γ δ     
Από τις σχέσεις (3), (4) παίρνουμε:
3 42R ρ ρ α β γ δ     
Έχουμε αποδείξει λοιπόν ότι:
1 2 3 4 1 2 3 42R ρ ρ 2R ρ ρ ρ ρ ρ ρ        
Πρόταση 8η
Ο εγγεγραμμένος κύκλος (Ι,ρ) ενός τριγώνου ΑΒΓ εφάπτεται των πλευρών ΑΒ, ΑΓ στα σημεία
Ε, Ζ αντίστοιχα. Αν η ευθεία ΓΙ τέμνει την ευθεία ΕΖ στο σημείο Ρ, τότε η γωνία ˆΒΡΓ είναι
ορθή.
Απόδειξη
 Στο τρίγωνο ΙΒΓ είναι:
ο
ˆˆ ˆΒ Γ Αˆ ˆ ˆΡΙΒ ΙΒΓ ΙΓΒ 90
2 2 2
      (1)
 Στο ισοσκελές τρίγωνο ΑΕΖ (ΑΕ ΑΖ) είναι:
ο οˆ ˆˆ ˆΑΕΖ ΑΖΕ 180 Α 2φ 180 Α      
ο
ˆΑ
φ 90
2
   (2)
Επειδή ο
ˆΑˆ ˆΑΕΡ ΡΙΒ 90
2
   , το τετράπλευρο ΡΕΒΙ είναι εγγράψιμο. Άρα oˆ ˆΒΡΙ BEI 90  .
Θεώρημα 9ο
Δίνεται ισόπλευρο τρίγωνο ΑΒΓ, εγγεγραμμένο σε κύκλο, και Μ τυχαίο σημείο του μικρού τόξου
ΒΓ . Να αποδειχθεί ότι ΜΒ + ΜΓ = ΜΑ .
(Θεώρημα van Schooten)
Απόδειξη
Έστω σημείο Ν στη ΜΑ, ώστε ΜΝ ΜΒ . Το
Δ
MBN είναι ισόπλευρο,
διότι οˆ ˆΒΜΑ ΒΓΑ 60  . Άρα:
BN BM και οˆ ˆ ˆABN MBΓ 60 ΝΒΓ  
Αλλά τώρα είναι
Δ Δ
ΑΝΒ ΜΒΓ , αφού:
ΑΒ ΒΓ , ΒΝ ΒΜ και οˆ ˆ ˆΑΒΝ ΜΒΓ 60 ΝΒΓ  
Άρα ΑΝ ΜΓ , δηλαδή:
ΜΑ ΜΝ ΝΑ ΜΒ ΜΓ   
Άλλος τρόπος
Το θεώρημα Πτολεμαίου στο εγγεγραμμένο τετράπλευρο ΑΒΜΓ δίνει:
ΑΒ ΜΓ ΑΓ ΜΒ ΑΜ ΒΓ α ΜΓ α ΜΒ ΑΜ α           
ΜΓ ΜΒ ΜΑ ΜΑ ΜΒ ΜΓ     
Άλλος τρόπος
Στην προέκταση της ΒΜ παίρνουμε τμήμα ΜΕ ΜΓ . Αλλά τότε τα τρίγωνα ΑΜΓ και ΕΒΓ είναι ίσα, οπότε:
ΒΕ ΑΜ ΒΜ ΜΕ ΑΜ ΑΜ ΜΒ ΜΓ      
Θεώρημα 10ο
Σε τρίγωνο ΑΒΓ, που είναι εγγεγραμμένο σε κύκλο (Κ,R), φέρνουμε το ύψος ΑΔ. Αν ΑΒ < ΑΓ , να
αποδειχθεί ότι:
α) ˆ ˆΔΑΒ = ΚΑΓ ,
β) ˆ ˆ ˆΚΑΔ = Β - Γ .
Απόδειξη
α) Φέρνουμε τη διάμετρο ΑΚΕ και τη ΓΕ. Το τρίγωνο ΑΓΕ είναι
ορθογώνιο, οπότε:
o oˆ ˆy 90 E 90 B x    
διότι και το τρίγωνο ΑΔΒ είναι ορθογώνιο. Άρα x y , δηλαδή
ˆ ˆΔΑΒ ΚΑΓ .
β) Είναι:
οˆ ˆ ˆ ˆΚΑΔ ΓΑΔ ΓΑΕ (90 Γ) y     
o οˆ ˆ ˆ ˆ(90 y) Γ (90 x) Γ Β Γ       
Σχόλια
α) Επειδή ˆ ˆΔΑΒ = ΚΑΓ , η διχοτόμος της γωνίας ˆΒΑΓ διχοτομεί τη γωνία ˆΚΑΔ . Επομένως, αν Μ είναι το
μέσο του τόξου ΒΓ , τότε ˆ ˆΜΑΔ = ΜΑΚ .
β) Από την παραπάνω πρόταση προκύπτει το θεώρημα:
"Το ορθόκεντρο και το περίκεντρο ενός τριγώνου είναι ισογώνια σημεία".
Υπενθυμίζουμε ότι δύο ημιευθείες λέγονται ισογώνιες ως προς τις πλευρές μιας γωνίας, όταν είναι συμμετρικές
ως προς τη διχοτόμο της γωνίας αυτής.
Θεώρημα 11ο
Να αποδειχθεί ότι οι ακτίνες της περιγεγραμμένης περιφέρειας ενός τριγώνου, που αντιστοιχούν
στις κορυφές του είναι αντίστοιχα κάθετες στις πλευρές του ορθικού τριγώνου.
(Θεώρημα Nagel)
Απόδειξη
Έστω O το περίκεντρο, Η το ορθόκεντρο και ΒΔ, ΓΕ ύψη του τριγώνου ΑΒΓ. Θα αποδειχθεί ότι OA ΔΕ .
Έστω ΚΑΛ η κοινή εφαπτομένη στο Α. Τότε:
 ˆˆΑΓΒ ΒΑΚ (x y) , διότι η ˆΓ είναι εγγεγραμμένη και η ˆΒΑΚ
είναι αντίστοιχη γωνία χορδής και εφαπτομένης.
 ˆ ˆΑΓΒ ΔΕΑ (x ω) , διότι το τετράπλευρο ΒΕΔΓ είναι εγγράψιμο
οˆˆ(ΒΕΓ ΒΔΓ 90 )  .
Επομένως y ω , που σημαίνει ότι ΚΛ // ΔΕ. Όμως ΟΑ ΚΛ , οπότε
ΟΑ ΔΕ .
Όμοια, αν ΑΖ είναι το τρίτο ύψος του
Δ
ΑΒΓ , τότε ΟΒ ΕΖ και ΟΓ ΔΖ . Με απλά λόγια, οι ακτίνες ΟΑ,
ΟΒ, ΟΓ είναι αντίστοιχα κάθετες στις πλευρές ΔΕ, ΕΖ, ΖΔ του ορθικού τριγώνου ΔΕΖ, του τριγώνου ΑΒΓ.
Θεώρημα 12ο
Στο εξωτερικό ενός τριγώνου ΑΒΓ θεωρούμε τα ισόπλευρα τρίγωνα ΑΒΔ, ΒΓΕ και ΓΑΖ. Να
αποδειχθεί ότι:
α) ΑΕ = ΒΖ = ΓΔ ,
β) οι ευθείες ΑΕ, ΒΖ, ΓΔ συντρέχουν σε σημείο Σ και ισχύει ΣΑ + ΣΒ + ΣΓ = ΑΕ = ΒΖ = ΓΔ .
γ) τα κέντρα Κ, Λ, Μ των τριγώνων ΑΒΔ, ΒΓΕ, ΓΑΖ σχηματίζουν ισόπλευρο τρίγωνο.
(Θεώρημα Steiner)
Απόδειξη
α) Τα τρίγωνα ΑΔΓ και ΑΒΖ έχουν:
 ΑΔ ΑΒ γ  και ΑΓ ΑΖ β  ,
 οˆ ˆ ˆ ˆ ˆ ˆΔΑΓ ΔΑΒ ΒΑΓ 60 ΒΑΓ ΒΑΓ ΓΑΖ      
ˆΒΑΖ .
Επομένως
Δ Δ
ΑΔΓ ΑΒΖ , οπότε ΔΓ ΒΖ .
Όμοια είναι
Δ Δ
ΒΔΓ ΑΒΕ , οπότε ΔΓ ΑΕ . Άρα
ΑΕ ΒΖ ΓΔ  .
β) Έστω ότι οι ΒΖ, ΓΔ τέμνονται στο Σ. Επειδή
Δ Δ
ΑΔΓ ΑΒΖ , είναι:
ˆ ˆΑΔΓ ΑΒΖ x  και ˆ ˆAZB AΓΔ y 
Επειδή ˆ ˆAΔΣ ΑΒΣ x  και ˆ ˆAZΣ ΑΓΣ y  , τα τετράπλευρα ΑΣΒΔ, ΑΣΓΖ είναι εγγράψιμα. Επομένως:
οˆ ˆΔΣΑ ΔΒΑ 60  και οˆˆΔΣΒ ΔΑΒ 60 
Επειδή οˆ ˆΒΣΔ 60 ΒΕΓ  , το ΒΣΓΕ είναι εγγράψιμο. Άρα οˆ ˆΒΣΕ ΒΓΕ 60  . Είναι λοιπόν:
ο ο ο οˆˆ ˆ ˆ ˆ ˆ ˆΑΣΕ ΑΣΔ ΔΣΒ ΒΣΕ ΑΒΔ ΔΑΒ ΒΓΕ 60 60 60 180         
οπότε η ΑΣΕ είναι ευθεία. Επομένως και η ΑΕ διέρχεται από το Σ. Από το θεώρημα van Schooten είναι
όμως ΣΒ ΣΓ ΣΕ  , οπότε
(α)
ΣΑ ΣΒ ΣΓ ΣΑ ΣΕ ΑΕ ΓΔ ΒΖ       .
γ) Έχουμε αποδείξει ότι:
οˆ ˆ ˆΑΣΒ ΒΣΓ ΓΣΑ 120  
Τα Κ, Λ είναι κέντρα των κύκλων (Σ,Α,Δ,Β) και
(Σ,Β,Ε,Γ). Άρα η ΚΛ είναι κάθετη στην κοινή χορδή
τους ΣΒ. Όμοια, η ΛΜ είναι κάθετη στην κοινή χορδή
ΣΓ των κύκλων (Λ), (Μ) και η ΚΜ είναι κάθετη στην
κοινή χορδή ΑΣ των κύκλων (Κ) και (Μ). Επειδή:
οˆ ˆ ˆΑΣΒ ΒΣΓ ΓΣΑ 120  
είναι:
οˆˆ ˆΚ Λ Μ 60  
και έτσι το τρίγωνο ΚΛΜ είναι ισόπλευρο.
Σχόλια
i) Αν όλες οι γωνίες του τριγώνου ΑΒΓ είναι μικρότερες από ο
120 , τότε το Σ είναι εσωτερικό σημείο του
τριγώνου ΑΒΓ. Στην περίπτωση αυτή το σημείο Σ λέγεται σημείο Fermat – Torricelli και είναι το μοναδικό
σημείο του τριγώνου για το οποίο το άθροισμα
ΣΑ + ΣΒ + ΣΓ γίνεται ελάχιστο.
ii) Στο ερώτημα (γ) είδαμε ότι τα κέντρα των τριγώνων ΔΑΒ, ΕΒΓ, ΖΓΑ σχηματίζουν ισόπλευρο τρίγωνο. Το
συμπέρασμα αυτό είναι γνωστό ως θεώρημα του Ναπολέοντα. Η απόδειξη μπορεί να γίνει επίσης με χρήση
μιγαδικών αριθμών, με τριγωνομετρία, αλλά και με χρήση ενός γεωμετρικού μετασχηματισμού που λέγεται
στροφή.
Θεώρημα 13ο
Αν Η είναι το ορθόκεντρο και Ο το περίκεντρο ενός τριγώνου ΑΒΓ, να αποδειχθεί ότι ΑΗ = 2ΟΜ , όπου
Μ είναι το μέσο του ΒΓ.
Απόδειξη
Θεωρούμε το αντιδιαμετρικό Ν του Β ως προς το Ο, δηλαδή τη
διάμετρο ΒΟΝ. Φέρουμε τις ΝΑ, ΝΓ. Επειδή ΝΓ ΓΒ και
ΑΗ ΒΓ , είναι ΑΗ // ΝΓ.
Είναι ΝΑ ΑΒ και ΓΗ ΑΒ , οπότε ΓΗ // ΝΑ. Το ΝΑΗΓ είναι
λοιπόν παραλληλόγραμμο, οπότε ΑΗ ΝΓ .
Στο τρίγωνο ΒΓΝ το ΟΜ ενώνει τα μέσα δύο πλευρών. Άρα
ΓΝ
ΟΜ
2
 , δηλαδή ΓΝ 2ΟΜ . Επομένως:
ΑΗ ΝΓ 2ΟΜ 
Θεώρημα 14ο
Το συμμετρικό του ορθόκεντρου ενός τριγώνου ΑΒΓ, ως προς τυχαία πλευρά του, είναι σημείο
του περιγεγραμμένου κύκλου του τριγώνου αυτού.
Απόδειξη
Θεωρούμε το ύψος ΑΔ, το ορθόκεντρο Η του
Δ
ΑΒΓ και το συμμετρικό Ν
του Η ως προς το ΒΓ. Για να ανήκει το Ν στον περιγεγραμμένο κύκλο του
Δ
ΑΒΓ , αρκεί να αποδείξουμε ότι το τετράπλευρο ΑΒΝΓ είναι εγγράψιμο.
Επειδή όμως ˆ ˆΒΝΓ ΒΗΓ , έχουμε:
οˆ ˆ ˆˆ ˆ ˆΒΝΓ ΒΑΓ ΒΗΓ ΒΑΓ ΖΗΕ ΖΑΕ 180     
διότι το ΑΖΗΕ είναι εγγράψιμο οˆ ˆ(ΑΖΗ ΑΕΗ 90 )  . Άρα το ΑΒΝΓ είναι
εγγράψιμο, δηλαδή το Ν ανήκει πράγματι στον περιγραμμένο κύκλο του τριγώνου ΑΒΓ.
Θεώρημα 15ο
Το συμμετρικό του ορθόκεντρου Η ενός τριγώνου ως προς το μέσο μιας τυχαίας πλευράς του,
είναι σημείο του περιγεγραμμένου κύκλου του τριγώνου αυτού.
Απόδειξη
Αρκεί να αποδείξουμε ότι το τετράπλευρο ΑΒΝΓ είναι εγγράψιμο σε
κύκλο. Το τετράπλευρο ΗΒΝΓ είναι παραλληλόγραμμο, διότι οι
διαγώνιες ΒΓ και ΗΝ διχοτομούνται. Επομένως ˆ ˆΒΝΓ ΒΗΓ . Άρα:
οˆ ˆ ˆˆ ˆ ˆΒΝΓ ΒΑΓ ΒΗΓ ΒΑΓ ΕΗΖ ΖΑΕ 180     
διότι το τετράπλευρο ΑΖΗΕ είναι εγγράψιμο. Άρα το Ν είναι σημείο του
περιγεγραμμένου κύκλου του τριγώνου ΑΒΓ.
Σημείωση
Επειδή, όπως έχουμε δει στο προηγούμενο θεώρημα, το συμμετρικό του ορθόκεντρου ως προς τυχαία πλευρά
είναι επίσης σημείο του περιγεγραμμένου κύκλου, μπορούμε να διατυπώσουμε το εξής θεώρημα:
«Τα συμμετρικά του ορθόκεντρου ενός τριγώνου ως προς τις πλευρές του και ως προς τα μέσα των
πλευρών του είναι σημεία του περιγεγραμμένου κύκλου του τριγώνου.»
Θεώρημα 16ο
Να αποδειχθεί ότι σε κάθε τρίγωνο, το ορθόκεντρο, το βαρύκεντρο και το περίκεντρο είναι
συνευθειακά σημεία.
(Ευθεία Euler)
Απόδειξη
Έστω Η το ορθόκεντρο και Ο το περίκεντρο του
Δ
ΑΒΓ . Έστω Μ το μέσο της ΒΓ, Θ το σημείο τομής των ΑΜ
και ΟΗ, Ρ το μέσο του ΑΘ και Ν το μέσο του ΘΗ. Θα αποδείξουμε ότι το Θ είναι το βαρύκεντρο του
τριγώνου ΑΒΓ, δηλαδή ότι:
ΘΑ 2ΘΜ
Στο τρίγωνο ΘΗΑ το ΡΝ ενώνει τα μέσα δύο πλευρών, οπότε:
ΑΗ 2ΟΜ
ΡΝ ΟΜ
2 2
  
διότι ΑΗ 2ΟΜ . Είναι επίσης ΡΝ // ΑΗ και αφού ΑΗ ΒΓ , είναι
ΡΝ ΒΓ . Αφού ΟΜ BΓ , είναι ΡΝ // ΟΜ και επειδή ΟΜ ΡΝ , το
ΟΜΝΡ είναι παραλληλόγραμμο. Επομένως οι διαγώνιες ΟΝ και ΡΜ
διχοτομούνται, δηλαδή ΘΜ ΘΡ ΡΑ  . Άρα:
ΑΘ 2ΘΜ
και έτσι το Θ είναι το βαρύκεντρο του τριγώνου ΑΒΓ.
Σημείωση
Επειδή ΘΟ = ΘΝ = ΝΗ , το βαρύκεντρο διαιρεί το τμήμα ΟΗ σε δύο τμήματα με λόγο 2. Πιο συγκεκριμένα
είναι ΘΗ = 2ΘΟ .
Θεώρημα 17ο
Να αποδειχθεί ότι το ορθόκεντρο ενός τριγώνου είναι το έγκεντρο του τριγώνου που έχει κορυφές
τα ίχνη των υψών του.
(Θεώρημα του ορθικού τριγώνου)
Απόδειξη
Φέρνουμε τα ύψη ΑΔ, ΒΕ, ΓΖ του τριγώνου ΑΒΓ και έστω Η το ορθόκεντρο. Θα αποδείξουμε ότι η ΔΑ είναι
διχοτόμος της γωνίας ˆΕΔΖ του ορθικού τριγώνου ΔΕΖ.
 Το τετράπλευρο ΗΔΒΖ είναι εγγράψιμο, διότι:
ο ο οˆ ˆΗΔΒ ΗΖΒ 90 90 180   
Επομένως x φ ˆ ˆ(ΗΔΖ ΗΒΖ) .
 Το τετράπλευρο ΗΔΓΕ είναι ομοίως εγγράψιμο, οπότε y ω ˆ ˆ(ΗΔΕ ΗΓΕ) .
Όμως και το τετράπλευρο ΒΖΕΓ είναι εγγράψιμο, διότι:
οˆ ˆΒΖΓ ΒΕΓ 90 
Επομένως φ ω ˆ ˆ(ΖΒΕ ΖΓΕ) .
Αφού λοιπόν x φ , y ω και φ ω , είναι και x y . Άρα η ΔΑ διχοτομεί τη γωνία ˆEΔΖ . Όμοια, η ΕΒ
διχοτομεί τη γωνία ˆΔΕΖ και η ΖΓ διχοτομεί τη γωνία ˆΕΖΔ .
Βλέπουμε λοιπόν ότι το Η είναι το έγκεντρο του τριγώνου ΔΕΖ.
Σχόλιο
Το τρίγωνο ΔΕΖ λέγεται ορθικό τρίγωνο του τριγώνου ΑΒΓ.
Θεώρημα 18ο
Σε κάθε τρίγωνο, τα μέσα των πλευρών του, τα ίχνη των υψών του και τα μέσα των τμημάτων
που συνδέουν το ορθόκεντρο με τις κορυφές του τριγώνου είναι ομοκυκλικά σημεία.
(Κύκλος του Euler)
Απόδειξη
Θα αποδείξουμε ότι ο περιγεγραμμένος κύκλος του τριγώνου με κορυφές τα μέσα Κ, Λ, Μ των πλευρών ΒΓ,
ΓΑ, ΑΒ του τριγώνου ΑΒΓ διέρχεται:
i) από τα ίχνη των υψών του τριγώνου ΑΒΓ,
ii) από τα μέσα των αποστάσεων του ορθοκέντρου από τις κορυφές του
τριγώνου ΑΒΓ.
i) Φέρνουμε το ύψος ΑΔ. Επειδή ΜΛ // ΒΓ, το ΔΚΛΜ είναι τραπέζιο. Είναι
επίσης:
ΑΒ
ΚΛ ΔΜ
2
 
διότι στο ορθογώνιο τρίγωνο ΑΔΒ το ΔΜ είναι διάμεσος. Το ΔΚΛΜ είναι λοιπόν ισοσκελές τραπέζιο, οπότε:
οˆˆ ˆ ˆΔΜΛ ΔΚΛ ΜΛΚ ΔΚΛ 180   
Άρα το ΔΚΛΜ είναι εγγράψιμο σε κύκλο (να τονίσουμε εδώ ότι κάθε ισοσκελές τραπέζιο είναι εγγράψιμο σε
κύκλο). Επομένως ο κύκλος (Κ,Λ,Μ), που διέρχεται από τις κορυφές του
Δ
ΚΛΜ , διέρχεται επίσης από το Δ.
Με άλλα λόγια, ο κύκλος (Κ, Λ, Μ) διέρχεται από τα ίχνη των υψών του τριγώνου ΑΒΓ.
ii) Θα αποδείξουμε τώρα ότι ο κύκλος (Κ,Λ,Μ) διέρχεται και από το μέσο Ρ
του τμήματος ΗΑ, όπου Η το ορθόκεντρο του τριγώνου ΑΒΓ. Αρκεί λοιπόν
να αποδειχθεί ότι το τετράπλευρο ΚΛΡΜ είναι εγγράψιμο. Επειδή ΜΡ // ΗΒ
και ΡΛ // ΗΓ, είναι:
 ˆ ˆ ˆΜΡΛ ΒΗΓ ΖΗΕ 
 οˆˆ ˆ ˆΜΡΛ ΜΚΛ ΖΗΕ ΖΑΛ 180   
διότι το ΑΖΗΕ είναι εγγράψιμο και το ΑΜΚΛ είναι παραλληλόγραμμο, οπότε ˆ ˆˆΜΚΛ Α ΖΑΕ  . Άρα το
τετράπλευρο ΜΡΛΚ είναι εγγράψιμο, οπότε ο κύκλος (Κ,Λ,Μ) διέρχεται και από το μέσο Ρ του ΗΑ. Όμοια,
ο κύκλος αυτός διέρχεται και από τα μέσα των ΗΒ, ΗΓ.
Άρα τελικά τα εννέα παραπάνω σημεία είναι ομοκυκλικά.
Σχόλια
Από τις παραγράφους (i) και (ii) συμπεραίνουμε ότι και τα 9 σημεία βρίσκονται στον ίδιο κύκλο. Ο κύκλος
αυτός λέγεται κύκλος του Euler.
 Το κέντρο του κύκλου του Euler είναι το μέσο Ι του τμήματος που συνδέει το ορθόκεντρο με το
περίκεντρο του τριγώνου ΑΒΓ. Αυτό συμβαίνει διότι π.χ. η μεσοκάθετος της χορδής ΔΚ του κύκλου του
Euler διέρχεται από το μέσο Ι της μη παράλληλης πλευράς ΟΗ του τραπεζίου ΗΔΚΟ (ΗΔ // ΟΚ, διότι
ΗΔ, ΟΚ ΒΓ) .
 Η ακτίνα του κύκλου του Euler είναι ίση με το μισό της ακτίνας R του περιγεγραμμένου κύκλου του
τριγώνου ΑΒΓ.
Πραγματικά, από το τρίγωνο ΗΟΑ προκύπτει ότι:
9
ΟΑ R
R = ΙΡ = ΙΡ =
2 2

Αλλά η ΙΡ είναι ακτίνα του κύκλου του Euler και έτσι ο ισχυρισμός μας αποδείχθηκε.
Άλλος τρόπος
Γνωρίζουμε από τα θεωρήματα 15 και 16 ότι το συμμετρικό του ορθόκεντρου Η ως προς την πλευρά ΒΓ
καθώς και ως προς το μέσο Μ της πλευράς ΒΓ είναι σημεία του περιγεγραμμένου κύκλου του
Δ
ΑΒΓ . Αν
λοιπόν Θ είναι το μέσο του ΗΒ, Δ το ίχνος του ύψους από το Α και Μ το μέσο του ΒΓ, αρκεί να αποδειχθεί
ότι TΘ ΤΔ ΤΜ  , όπου Τ είναι το μέσο του ΟΗ.
 Φέρουμε ΤΣ ΒΓ . Στο τραπέζιο ΟΜΔΗ το ΤΣ είναι διάμεσος (αφού ΤΣ // ΟΜ και το Τ είναι μέσο
του ΟΗ), οπότε η ΤΣ είναι μεσοκάθετος του ΜΔ. Άρα:
ΟΝ R
ΤΔ ΤΜ
2 2
   (1)
Σημειώνουμε ότι στο τρίγωνο ΗΟΝ το ΤΜ ενώνει τα μέσα των
πλευρών ΗΟ, ΗΝ.
 Στο τρίγωνο ΗΟΒ το ΤΘ ενώνει τα μέσα Τ, Θ των πλευρών ΗΟ,
ΗΒ. Επομένως:
ΟΒ R
TΘ
2 2
  (2)
Από τις (1) και (2) συμπεραίνουμε ότι:
R
TΘ ΤΔ ΤΜ
2
  
Άρα ο κύκλος με κέντρο το μέσο Τ του ΟΗ και ακτίνα
R
2
διέρχεται από το Θ, το Δ και το Μ, δηλαδή τελικά
από τα μέσα των τμημάτων ΗΒ, ΗΓ, ΗΑ, από τα ίχνη των υψών και από τα μέσα των πλευρών του τριγώνου
ΑΒΓ. Το Τ είναι λοιπόν το κέντρο του κύκλου Euler, η δε ακτίνα του κύκλου αυτού είναι
R
2
.
Θεώρημα 19ο
Δίνεται τρίγωνο ΑΒΓ και το τρίγωνο ΚΛΜ που
σχηματίζουν τα παράκεντρα του τριγώνου
αυτού. Τότε ο περιγεγραμμένος κύκλος του
τριγώνου ΑΒΓ διέρχεται από τα μέσα των
τμημάτων ΚΛ, ΛΜ, ΚΜ.
Απόδειξη
Επειδή oˆIAM 90 , είναι:
KA MΛ
Ανάλογα είναι ΛΒ ΚΜ , ΜΓ ΚΛ . Άρα στο τρίγωνο
ΚΛΜ το έγκεντρο Ι του
Δ
ΑΒΓ είναι ορθόκεντρο. Στο
τρίγωνο ΚΛΜ τα Α, Β, Γ είναι ίχνη των υψών, οπότε από
τα Α, Β, Γ περνάει ο κύκλος Euler του τριγώνου ΚΛΜ.
Αλλά ο κύκλος Euler περνάει και από τα μέσα των πλευρών ΚΛ, ΛΜ, ΚΜ. Με άλλα λόγια, ο
περιγεγραμμένος κύκλος του τριγώνου ΑΒΓ δέρχεται από τα μέσα των τμημάτων ΚΛ, ΛΜ, ΚΜ του
τριγώνου ΚΛΜ που σχηματίζουν τα παράκεντρα του
Δ
ΑΒΓ .
Σημείωση
Ο κύκλος (Α,Β,Γ) διέρχεται επίσης από τα μέσα των ΙΜ, ΙΚ, ΙΛ, αφού στο τρίγωνο ΚΛΜ ο κύκλος αυτός είναι ο
κύκλος Euler και το Ι είναι το ορθόκεντρο του
Δ
ΚΛΜ .
Θεώρημα 20ο
Οι προβολές τυχαίου σημείου του περιγεγραμμένου κύκλου ενός τριγώνου πάνω στις πλευρές του
τριγώνου αυτού, είναι συνευθειακά σημεία.
(Ευθεία Simson)
Απόδειξη
Έστω ΜΔ ΒΓ , ΜΕ ΑΓ , ΜΖ ΑΒ . Επειδή η ΑΕΓ είναι ευθεία, για να είναι τα σημεία Δ, Ε, Ζ
συνευθειακά, αρκεί να αποδείξουμε ότι ˆ ˆΑΕΖ ΔΕΓ .
 Το τετράπλευρο ΓΔΕΜ είναι εγγράψιμο, οπότε:
ˆ ˆΔΕΓ ΔΜΓ x 
 Το τετράπλευρο ΑΕΜΖ είναι εγγράψιμο, οπότε:
ˆ ˆAEZ AMZ y 
 Τα τρίγωνα ΜΔΓ, ΜΖΑ είναι ορθογώνια, οπότε:
ο oˆ ˆ ˆΔΜΓ 90 ΜΓΔ x 90 MΓΔ     (1)
ο oˆ ˆˆΑΜΖ 90 ΖΑΜ y 90 ZAM     (2)
 Από το εγγεγραμμένο τετράπλευρο ΑΒΓΜ παίρνουμε όμως ότι:
ˆ ˆˆ ˆMΓΒ ΜΑΖ ΜΓΔ ΖΑΜ   (3)
Οι σχέσεις (1) και (2), λόγω της (3), δίνουν x y . Άρα τα σημεία Δ, Ε, Ζ είναι συνευθειακά.
Σχόλια
α) Είναι σημαντικό να παρατηρήσουμε ότι ισχύει και το αντίστροφο του θεωρήματος αυτού. Έτσι:
"Αν οι προβολές ενός σημείου του επιπέδου εκτός τριγώνου πάνω στις πλευρές του είναι συνευθειακά
σημεία, τότε το σημείο αυτό ανήκει στον περιγεγραμμένο κύκλο του τριγώνου."
β) Η ευθεία που σχηματίζουν οι προβολές των σημείων Μ πάνω στις πλευρές του τριγώνου λέγεται ευθεία
Simson του Μ για το τρίγωνο αυτό ή ακόμα ευθεία Simson – Wallace.
γ) Το γενικευμένο θεώρημα Simson – Wallace έχει την εξής μορφή:
Αν Μ είναι σημείο του περιγεγραμμένου κύκλου ενός τριγώνου ΑΒΓ
και Δ, Ε, Ζ είναι σημεία των ΒΓ, ΓΑ, ΑΒ αντίστοιχα, ώστε οι
προσανατολισμένες γωνίες ˆMΔΓ , ˆΜΕΓ , ˆΜΖΑ να είναι ίσες, τότε τα
σημεία Δ, Ε, Ζ είναι συνευθειακά.
Η απόδειξη προκύπτει, όπως και πριν, από τα εγγράψιμα τετράπλευρα
ΜΕΔΓ, ΜΕΑΖ και το εγγεγραμμένο τετράπλευρο ΑΒΓΜ, οπότε τελικά
είναι ˆ ˆΔΕΓ = ΑΕΖ .
Σελίδα 1 από 5
Μπάμπης Στεργίου – Μαθηματικός 25/02/2017
Οι εκφωνήσεις των ασκήσεων
Σελίδα 2 από 5
Μπάμπης Στεργίου – Μαθηματικός 25/02/2017
A. Τρίγωνα – τετράπλευρα
ArJun.1 Δίνεται ισόπλευρο τρίγωνο ΑΒΓ και σημείο Δ
στην πλευρά ΒΓ. Στην προέκταση της ΑΓ παίρνουμε τμήμα
ΓΕ ΒΔ . Να αποδείξετε ότι ΔΑ ΔΕ .
ArJun.2 Δίνεται τετράγωνο ΑΒΓΔ, σημείο Ε στην
πλευρά ΑΒ και σημείο Ζ στην πλευρά ΒΓ, ώστε ΒΕ ΒΖ . Η
κάθετη ΒΚ από το Β προς την ΓΕ τέμνει την ΑΔ στο σημείο
Η. Να αποδείξετε ότι:
α) ΓΕ ΒΗ και ΓΕ ΒΗ .
β) Το ΔΗΖΓ είναι ορθογώνιο.
γ) Η γωνία ˆΔΚΖ είναι ορθή.
ArJun.3 Δίνεται τρίγωνο ΑΒΓ, το μέσο Μ της ΒΓ, μια
ημιευθεία Αx στο εσωτερικό της γωνίας ˆΑ που δεν διέρχεται
από το Μ και οι προβολές Δ, Ε των σημείων Β και Γ πάνω
στην Αx. Να αποδείξετε ότι:
ΜΔ ΜΕ .
ArJun.4 Δίνεται τετράγωνο ΑΒΓΔ και το μέσο Μ της
πλευράς ΑΒ. Η κάθετη προς την ΑΓ στο σημείο Α τέμνει την
ευθεία ΓΒ στο σημείο Ε. Να αποδείξετε ότι τα σημεία Γ, Β, Ε
βρίσκονται στην ίδια ευθεία.
ArJun.5 Έστω Μ το μέσο της πλευράς ΑΒ ενός
τετραγώνου ΑΒΓΔ. Η ευθεία ΔΜ τέμνει την ευθεία ΓΒ στο
σημείο Ε. Να αποδείξετε ότι το τρίγωνο ΑΓΕ είναι ορθογώνιο
και ισοσκελές.
ArJun.6 Στη βάση ΒΓ ισοσκελούς τριγώνου ΑΒΓ
παίρνουμε σημείο Δ. Στις πλευρές ΑΓ, ΑΒ παίρνουμε
αντίστοιχα τα σημεία Ε και Ζ έτσι, ώστε:
ˆ ˆΔΑΒ 2ΓΔΕ και ˆ ˆΔΑΓ 2ΒΔΖ .
Να αποδείξετε ότι το τρίγωνο ΑΖΕ είναι ισοσκελές.
ArJun.7 Στο εσωτερικό ενός ισοσκελούς τριγώνου ΑΒΓ
(ΑΒ ΑΓ) υπάρχει ένα σημείο Μ, ώστε:
οˆΜΒΓ 30 και
3ˆ ˆΜΑΒ ΒΑΓ
4
 .
Να αποδείξετε ότι οˆΑΜΓ 150 .
ArJun.8 Σε ένα τρίγωνο ΑΒΓ είναι:
οˆΒ 30 και οˆΓ 45
Αν Ε είναι το συμμετρικό του Β ως προς το σημείο Γ, να
αποδειχθεί ότι οˆΕ 15 .
(Vrenceanu – 2006)
ArJun.9 Έστω ΑΒΓ ισοσκελές τρίγωνο με ΑΒ ΑΓ και
οˆΑ 40 . Στο εσωτερικό του ΑΒΓ παίρνουμε σημείο Δ
τέτοιο, ώστε οˆΔΑΓ 10 και ΑΔ ΒΓ . Να αποδείξετε ότι
οˆΔΓΑ 20 .
ArJun.10 Δίνεται ισόπλευρο τρίγωνο ΑΒΓ και σημείο Δ
στην πλευρά ΑΓ. Η διχοτόμος της γωνίας ΑΒΔ και η
παράλληλη από το Α προς τη ΒΓ τέμνονται στο σημείο Ε. Να
αποδείξετε ότι ΒΔ ΑΕ ΓΔ  .
(Sperante Olympia – 2010)
ArJun.11 Στις πλευρές ΑΒ και ΑΓ ενός τριγώνου ΑΒΓ
παίρνουμε τα σημεία Δ, Ε αντίστοιχα έτσι, ώστε ΒΔ ΓΕ .
Αν Μ, Ν είναι τα μέσα των τμημάτων ΒΕ, ΓΔ και η ευθεία
ΜΝ τέμνει τις ευθείες ΑΒ, ΑΓ στα σημεία Ζ και Η, να
αποδείξετε ότι το τρίγωνο ΑΖΗ είναι ισοσκελές.
(Βουλγαρία – 2002)
ArJun.12 Σε ένα ισοσκελές τρίγωνο ΑΒΓ είναι
οˆ ˆΒ Γ 80  . Στην πλευρά ΑΓ παίρνουμε τμήμα ΑΔ ΒΓ .
Να αποδειχθεί ότι οˆΑΒΔ 10 .
ArJun.13 Θεωρούμε το παραλληλόγραμμο ΑΒΓΔ, το
μέσο Μ του ΑΒ, το μέσο Ν του ΑΓ και η προβολή Ρ του Δ
στην ευθεία ΓΜ. Να αποδείξετε ότι ˆ ˆΑΡΝ ΑΒΓ .
ArJun.14 Δίνεται ορθογώνιο τρίγωνο ΑΒΓ
οˆ(Α 90 ) , το
ύψος ΑΔ, τυχαίο σημείο Ρ του τμήματος ΑΔ, η κάθετη προς
την ΒΡ στο Ρ που τέμνει την ευθεία ΓΑ στο Ε και η
παράλληλη από το Ρ προς τη ΒΓ που τέμνει την ΑΓ στο Ζ.
Να αποδείξετε ότι ΑΕ ΓΖ .
ArJun.15 Δίνεται τετράγωνο ΑΒΓΔ και τα σημεία Μ, Ν
των πλευρών ΓΔ, ΓΒ αντίστοιχα έτσι, ώστε ˆ ˆΜΑΔ ΜΑΝ .
Στην προέκταση της ΝΒ παίρνουμε σημείο Ε τέτοιο, ώστε
ΑΕ ΑΜ . Αν η ευθεία ΑΜ τέμνει τη ΒΓ στο Ρ, να
αποδείξετε ότι:
α) Το τρίγωνο ΑΕΡ είναι ορθογώνιο.
β)
ΡΕ
ΔΜ ΒΝ
2
  .
γ) ΑΝ ΜΔ ΝΒ  .
(Arad – 2008)
ArJun.16 Σε ένα τραπέζιο ΑΒΓΔ με ΑΒ // ΓΔ είναι
ΓΔ 2ΑΒ . Έστω Ν το μέσο του ΓΔ και Μ το μέσο του ΒΓ.
Εξωτερικά του ΑΒΓΔ θεωρούμε τα ισόπλευρα τρίγωνα ΑΔΕ
και ΓΝΖ. Να αποδειχθεί ότι:
οˆΕΜΖ 90 .
Διαγωνισμός Αρχιμήδης (μικροί) - 56 Ασκήσεις Γεωμετρίας με λύσεις
Διαγωνισμός Αρχιμήδης (μικροί) - 56 Ασκήσεις Γεωμετρίας με λύσεις
Διαγωνισμός Αρχιμήδης (μικροί) - 56 Ασκήσεις Γεωμετρίας με λύσεις
Διαγωνισμός Αρχιμήδης (μικροί) - 56 Ασκήσεις Γεωμετρίας με λύσεις
Διαγωνισμός Αρχιμήδης (μικροί) - 56 Ασκήσεις Γεωμετρίας με λύσεις
Διαγωνισμός Αρχιμήδης (μικροί) - 56 Ασκήσεις Γεωμετρίας με λύσεις
Διαγωνισμός Αρχιμήδης (μικροί) - 56 Ασκήσεις Γεωμετρίας με λύσεις
Διαγωνισμός Αρχιμήδης (μικροί) - 56 Ασκήσεις Γεωμετρίας με λύσεις
Διαγωνισμός Αρχιμήδης (μικροί) - 56 Ασκήσεις Γεωμετρίας με λύσεις
Διαγωνισμός Αρχιμήδης (μικροί) - 56 Ασκήσεις Γεωμετρίας με λύσεις
Διαγωνισμός Αρχιμήδης (μικροί) - 56 Ασκήσεις Γεωμετρίας με λύσεις
Διαγωνισμός Αρχιμήδης (μικροί) - 56 Ασκήσεις Γεωμετρίας με λύσεις
Διαγωνισμός Αρχιμήδης (μικροί) - 56 Ασκήσεις Γεωμετρίας με λύσεις
Διαγωνισμός Αρχιμήδης (μικροί) - 56 Ασκήσεις Γεωμετρίας με λύσεις
Διαγωνισμός Αρχιμήδης (μικροί) - 56 Ασκήσεις Γεωμετρίας με λύσεις
Διαγωνισμός Αρχιμήδης (μικροί) - 56 Ασκήσεις Γεωμετρίας με λύσεις
Διαγωνισμός Αρχιμήδης (μικροί) - 56 Ασκήσεις Γεωμετρίας με λύσεις
Διαγωνισμός Αρχιμήδης (μικροί) - 56 Ασκήσεις Γεωμετρίας με λύσεις
Διαγωνισμός Αρχιμήδης (μικροί) - 56 Ασκήσεις Γεωμετρίας με λύσεις
Διαγωνισμός Αρχιμήδης (μικροί) - 56 Ασκήσεις Γεωμετρίας με λύσεις
Διαγωνισμός Αρχιμήδης (μικροί) - 56 Ασκήσεις Γεωμετρίας με λύσεις
Διαγωνισμός Αρχιμήδης (μικροί) - 56 Ασκήσεις Γεωμετρίας με λύσεις
Διαγωνισμός Αρχιμήδης (μικροί) - 56 Ασκήσεις Γεωμετρίας με λύσεις
Διαγωνισμός Αρχιμήδης (μικροί) - 56 Ασκήσεις Γεωμετρίας με λύσεις

Mais conteúdo relacionado

Mais procurados

Σημειώσεις στα πολυώνυμα για προχωρημένους!!
Σημειώσεις στα πολυώνυμα για προχωρημένους!! Σημειώσεις στα πολυώνυμα για προχωρημένους!!
Σημειώσεις στα πολυώνυμα για προχωρημένους!! Μάκης Χατζόπουλος
 
Επαναληπτικό διαγώνισμα Β Λυκείου Άλγεβρα - Πολυώνυμα
Επαναληπτικό διαγώνισμα Β Λυκείου Άλγεβρα - ΠολυώνυμαΕπαναληπτικό διαγώνισμα Β Λυκείου Άλγεβρα - Πολυώνυμα
Επαναληπτικό διαγώνισμα Β Λυκείου Άλγεβρα - ΠολυώνυμαΜάκης Χατζόπουλος
 
οριστική Ε.Φ. βαρυτόνων ρημάτων
οριστική Ε.Φ. βαρυτόνων ρημάτωνοριστική Ε.Φ. βαρυτόνων ρημάτων
οριστική Ε.Φ. βαρυτόνων ρημάτωνzazagina
 
α γυμνασιου 35 θεματα εξετασεων 2016
α γυμνασιου 35 θεματα εξετασεων 2016α γυμνασιου 35 θεματα εξετασεων 2016
α γυμνασιου 35 θεματα εξετασεων 2016Christos Loizos
 
συνολο τιμων συναρτησησ
συνολο τιμων συναρτησησσυνολο τιμων συναρτησησ
συνολο τιμων συναρτησησChristos Loizos
 
'Αλγεβρα Β λυκείου,συστήματα
'Αλγεβρα Β λυκείου,συστήματα'Αλγεβρα Β λυκείου,συστήματα
'Αλγεβρα Β λυκείου,συστήματαΘανάσης Δρούγας
 
πως λύνουμε προβλήματα περιεκτικότητας
πως λύνουμε προβλήματα περιεκτικότηταςπως λύνουμε προβλήματα περιεκτικότητας
πως λύνουμε προβλήματα περιεκτικότηταςkanakidi
 
Γ Γυμνασίου παράγρ 1.2 -1.5 σχ. έτος 2014 15
Γ Γυμνασίου παράγρ 1.2 -1.5 σχ. έτος 2014 15Γ Γυμνασίου παράγρ 1.2 -1.5 σχ. έτος 2014 15
Γ Γυμνασίου παράγρ 1.2 -1.5 σχ. έτος 2014 15Μάκης Χατζόπουλος
 
Σημειώσεις Τριγωνομετρίας Β΄ Λυκείου
Σημειώσεις Τριγωνομετρίας Β΄ Λυκείου Σημειώσεις Τριγωνομετρίας Β΄ Λυκείου
Σημειώσεις Τριγωνομετρίας Β΄ Λυκείου Μάκης Χατζόπουλος
 
Τεστ στην ομοιότητα τριγώνων - Γεωμετρία Β Λυκείου
Τεστ στην ομοιότητα τριγώνων - Γεωμετρία Β ΛυκείουΤεστ στην ομοιότητα τριγώνων - Γεωμετρία Β Λυκείου
Τεστ στην ομοιότητα τριγώνων - Γεωμετρία Β ΛυκείουΜάκης Χατζόπουλος
 
Σημειώσεις μαθηματικών θετικής Κατεύθυνσης Β΄ Λυκείου
Σημειώσεις μαθηματικών θετικής Κατεύθυνσης Β΄ ΛυκείουΣημειώσεις μαθηματικών θετικής Κατεύθυνσης Β΄ Λυκείου
Σημειώσεις μαθηματικών θετικής Κατεύθυνσης Β΄ ΛυκείουΜάκης Χατζόπουλος
 
anisotikes
anisotikesanisotikes
anisotikesperi2005
 
Αρχαία Ελληνική Γλώσσα Α΄ Γυμνασίου, ενότητα 3 -Τονισμός
Αρχαία Ελληνική Γλώσσα Α΄ Γυμνασίου, ενότητα 3 -ΤονισμόςΑρχαία Ελληνική Γλώσσα Α΄ Γυμνασίου, ενότητα 3 -Τονισμός
Αρχαία Ελληνική Γλώσσα Α΄ Γυμνασίου, ενότητα 3 -Τονισμόςvserdaki
 
Μαθηματικά προσανατολισμού Β Λυκείου
Μαθηματικά προσανατολισμού Β ΛυκείουΜαθηματικά προσανατολισμού Β Λυκείου
Μαθηματικά προσανατολισμού Β ΛυκείουΜάκης Χατζόπουλος
 
Φυλλάδιο στην Τριγωνική Ανισότητα
Φυλλάδιο στην Τριγωνική ΑνισότηταΦυλλάδιο στην Τριγωνική Ανισότητα
Φυλλάδιο στην Τριγωνική ΑνισότηταΜάκης Χατζόπουλος
 
Μετοχές, είδη μετοχών Αρχαία Ελληνικά
Μετοχές, είδη μετοχών Αρχαία ΕλληνικάΜετοχές, είδη μετοχών Αρχαία Ελληνικά
Μετοχές, είδη μετοχών Αρχαία Ελληνικάmvourtsian
 

Mais procurados (20)

Σημειώσεις στα πολυώνυμα για προχωρημένους!!
Σημειώσεις στα πολυώνυμα για προχωρημένους!! Σημειώσεις στα πολυώνυμα για προχωρημένους!!
Σημειώσεις στα πολυώνυμα για προχωρημένους!!
 
Επαναληπτικό διαγώνισμα Β Λυκείου Άλγεβρα - Πολυώνυμα
Επαναληπτικό διαγώνισμα Β Λυκείου Άλγεβρα - ΠολυώνυμαΕπαναληπτικό διαγώνισμα Β Λυκείου Άλγεβρα - Πολυώνυμα
Επαναληπτικό διαγώνισμα Β Λυκείου Άλγεβρα - Πολυώνυμα
 
οριστική Ε.Φ. βαρυτόνων ρημάτων
οριστική Ε.Φ. βαρυτόνων ρημάτωνοριστική Ε.Φ. βαρυτόνων ρημάτων
οριστική Ε.Φ. βαρυτόνων ρημάτων
 
Άλγεβρα Β Γυμνασίου
Άλγεβρα Β Γυμνασίου Άλγεβρα Β Γυμνασίου
Άλγεβρα Β Γυμνασίου
 
α γυμνασιου 35 θεματα εξετασεων 2016
α γυμνασιου 35 θεματα εξετασεων 2016α γυμνασιου 35 θεματα εξετασεων 2016
α γυμνασιου 35 θεματα εξετασεων 2016
 
συνολο τιμων συναρτησησ
συνολο τιμων συναρτησησσυνολο τιμων συναρτησησ
συνολο τιμων συναρτησησ
 
'Αλγεβρα Β λυκείου,συστήματα
'Αλγεβρα Β λυκείου,συστήματα'Αλγεβρα Β λυκείου,συστήματα
'Αλγεβρα Β λυκείου,συστήματα
 
1η ενότητα νεοελληνικής γλώσσας. Παραγωγή με αχώριστα μόρια (4ο μάθημα)
1η ενότητα νεοελληνικής γλώσσας. Παραγωγή με αχώριστα μόρια (4ο μάθημα)1η ενότητα νεοελληνικής γλώσσας. Παραγωγή με αχώριστα μόρια (4ο μάθημα)
1η ενότητα νεοελληνικής γλώσσας. Παραγωγή με αχώριστα μόρια (4ο μάθημα)
 
πως λύνουμε προβλήματα περιεκτικότητας
πως λύνουμε προβλήματα περιεκτικότηταςπως λύνουμε προβλήματα περιεκτικότητας
πως λύνουμε προβλήματα περιεκτικότητας
 
Γ Γυμνασίου παράγρ 1.2 -1.5 σχ. έτος 2014 15
Γ Γυμνασίου παράγρ 1.2 -1.5 σχ. έτος 2014 15Γ Γυμνασίου παράγρ 1.2 -1.5 σχ. έτος 2014 15
Γ Γυμνασίου παράγρ 1.2 -1.5 σχ. έτος 2014 15
 
Σημειώσεις Τριγωνομετρίας Β΄ Λυκείου
Σημειώσεις Τριγωνομετρίας Β΄ Λυκείου Σημειώσεις Τριγωνομετρίας Β΄ Λυκείου
Σημειώσεις Τριγωνομετρίας Β΄ Λυκείου
 
Τεστ στην ομοιότητα τριγώνων - Γεωμετρία Β Λυκείου
Τεστ στην ομοιότητα τριγώνων - Γεωμετρία Β ΛυκείουΤεστ στην ομοιότητα τριγώνων - Γεωμετρία Β Λυκείου
Τεστ στην ομοιότητα τριγώνων - Γεωμετρία Β Λυκείου
 
Σημειώσεις μαθηματικών θετικής Κατεύθυνσης Β΄ Λυκείου
Σημειώσεις μαθηματικών θετικής Κατεύθυνσης Β΄ ΛυκείουΣημειώσεις μαθηματικών θετικής Κατεύθυνσης Β΄ Λυκείου
Σημειώσεις μαθηματικών θετικής Κατεύθυνσης Β΄ Λυκείου
 
anisotikes
anisotikesanisotikes
anisotikes
 
Αρχαία Ελληνική Γλώσσα Α΄ Γυμνασίου, ενότητα 3 -Τονισμός
Αρχαία Ελληνική Γλώσσα Α΄ Γυμνασίου, ενότητα 3 -ΤονισμόςΑρχαία Ελληνική Γλώσσα Α΄ Γυμνασίου, ενότητα 3 -Τονισμός
Αρχαία Ελληνική Γλώσσα Α΄ Γυμνασίου, ενότητα 3 -Τονισμός
 
Μαθηματικά προσανατολισμού Β Λυκείου
Μαθηματικά προσανατολισμού Β ΛυκείουΜαθηματικά προσανατολισμού Β Λυκείου
Μαθηματικά προσανατολισμού Β Λυκείου
 
Διαγωνίσματα στην Άλγεβρα Α' Γυμνασίου
Διαγωνίσματα στην Άλγεβρα Α' ΓυμνασίουΔιαγωνίσματα στην Άλγεβρα Α' Γυμνασίου
Διαγωνίσματα στην Άλγεβρα Α' Γυμνασίου
 
Φυλλάδιο στην Τριγωνική Ανισότητα
Φυλλάδιο στην Τριγωνική ΑνισότηταΦυλλάδιο στην Τριγωνική Ανισότητα
Φυλλάδιο στην Τριγωνική Ανισότητα
 
Μετοχές, είδη μετοχών Αρχαία Ελληνικά
Μετοχές, είδη μετοχών Αρχαία ΕλληνικάΜετοχές, είδη μετοχών Αρχαία Ελληνικά
Μετοχές, είδη μετοχών Αρχαία Ελληνικά
 
60
6060
60
 

Semelhante a Διαγωνισμός Αρχιμήδης (μικροί) - 56 Ασκήσεις Γεωμετρίας με λύσεις

γεωμετρια α λυκειου
γεωμετρια   α  λυκειουγεωμετρια   α  λυκειου
γεωμετρια α λυκειουChristos Loizos
 
Είδη και στοιχεία τριγώνων
Είδη και στοιχεία τριγώνωνΕίδη και στοιχεία τριγώνων
Είδη και στοιχεία τριγώνωνpstavro
 
Τρίγωνα
ΤρίγωναΤρίγωνα
Τρίγωναpstavro
 
Εργασία τμήματος Α1 - Αποδείξεις Ιδ και Κρ - Ορισμοί
Εργασία τμήματος Α1 - Αποδείξεις Ιδ και Κρ - ΟρισμοίΕργασία τμήματος Α1 - Αποδείξεις Ιδ και Κρ - Ορισμοί
Εργασία τμήματος Α1 - Αποδείξεις Ιδ και Κρ - ΟρισμοίΜάκης Χατζόπουλος
 
ΘΕΜΑΤΑ ΜΑΘΗΜΑΤΙΚΩΝ Α' ΛΥΚΕΙΟΥ (ΤΡΑΠΕΖΑ ΘΕΜΑΤΩΝ)
ΘΕΜΑΤΑ ΜΑΘΗΜΑΤΙΚΩΝ Α' ΛΥΚΕΙΟΥ (ΤΡΑΠΕΖΑ ΘΕΜΑΤΩΝ) ΘΕΜΑΤΑ ΜΑΘΗΜΑΤΙΚΩΝ Α' ΛΥΚΕΙΟΥ (ΤΡΑΠΕΖΑ ΘΕΜΑΤΩΝ)
ΘΕΜΑΤΑ ΜΑΘΗΜΑΤΙΚΩΝ Α' ΛΥΚΕΙΟΥ (ΤΡΑΠΕΖΑ ΘΕΜΑΤΩΝ) lykkarea
 
200 άλυτες ασκήσεις γεωμετρίας κοπάδης
200 άλυτες ασκήσεις γεωμετρίας κοπάδης200 άλυτες ασκήσεις γεωμετρίας κοπάδης
200 άλυτες ασκήσεις γεωμετρίας κοπάδηςΜάκης Χατζόπουλος
 
ΓΕΩΜΕΤΡΙΑ Α ΓΥΜΝ ΘΕΩΡΙΑ.pdf
ΓΕΩΜΕΤΡΙΑ Α ΓΥΜΝ ΘΕΩΡΙΑ.pdfΓΕΩΜΕΤΡΙΑ Α ΓΥΜΝ ΘΕΩΡΙΑ.pdf
ΓΕΩΜΕΤΡΙΑ Α ΓΥΜΝ ΘΕΩΡΙΑ.pdfMaria Economidou
 
ΓΕΩΜΕΤΡΙΑ Α ΓΥΜΝ ΘΕΩΡΙΑ.pdf
ΓΕΩΜΕΤΡΙΑ Α ΓΥΜΝ ΘΕΩΡΙΑ.pdfΓΕΩΜΕΤΡΙΑ Α ΓΥΜΝ ΘΕΩΡΙΑ.pdf
ΓΕΩΜΕΤΡΙΑ Α ΓΥΜΝ ΘΕΩΡΙΑ.pdfMaria Economidou
 
Θεωρία από τη Γεωμετρία Α΄ Λυκείου
Θεωρία από τη Γεωμετρία Α΄ ΛυκείουΘεωρία από τη Γεωμετρία Α΄ Λυκείου
Θεωρία από τη Γεωμετρία Α΄ ΛυκείουΜάκης Χατζόπουλος
 

Semelhante a Διαγωνισμός Αρχιμήδης (μικροί) - 56 Ασκήσεις Γεωμετρίας με λύσεις (20)

Γεωμετρία Α΄ Λυκείου 2017 - 18
Γεωμετρία Α΄ Λυκείου 2017 - 18Γεωμετρία Α΄ Λυκείου 2017 - 18
Γεωμετρία Α΄ Λυκείου 2017 - 18
 
γεωμετρια α λυκειου
γεωμετρια   α  λυκειουγεωμετρια   α  λυκειου
γεωμετρια α λυκειου
 
Γεωμετρία: 3.1- 3.2
Γεωμετρία: 3.1- 3.2Γεωμετρία: 3.1- 3.2
Γεωμετρία: 3.1- 3.2
 
Είδη και στοιχεία τριγώνων
Είδη και στοιχεία τριγώνωνΕίδη και στοιχεία τριγώνων
Είδη και στοιχεία τριγώνων
 
Τρίγωνα
ΤρίγωναΤρίγωνα
Τρίγωνα
 
α΄λ γεωμετρια επαναληψη
α΄λ γεωμετρια επαναληψηα΄λ γεωμετρια επαναληψη
α΄λ γεωμετρια επαναληψη
 
Γεωμετρία: 3.3-3.4
Γεωμετρία: 3.3-3.4Γεωμετρία: 3.3-3.4
Γεωμετρία: 3.3-3.4
 
Εργασία τμήματος Α1 - Αποδείξεις Ιδ και Κρ - Ορισμοί
Εργασία τμήματος Α1 - Αποδείξεις Ιδ και Κρ - ΟρισμοίΕργασία τμήματος Α1 - Αποδείξεις Ιδ και Κρ - Ορισμοί
Εργασία τμήματος Α1 - Αποδείξεις Ιδ και Κρ - Ορισμοί
 
Parallilogramma trapezia
Parallilogramma trapeziaParallilogramma trapezia
Parallilogramma trapezia
 
θεωρια της α΄λυκείου
θεωρια της α΄λυκείουθεωρια της α΄λυκείου
θεωρια της α΄λυκείου
 
ΘΕΜΑΤΑ ΜΑΘΗΜΑΤΙΚΩΝ Α' ΛΥΚΕΙΟΥ (ΤΡΑΠΕΖΑ ΘΕΜΑΤΩΝ)
ΘΕΜΑΤΑ ΜΑΘΗΜΑΤΙΚΩΝ Α' ΛΥΚΕΙΟΥ (ΤΡΑΠΕΖΑ ΘΕΜΑΤΩΝ) ΘΕΜΑΤΑ ΜΑΘΗΜΑΤΙΚΩΝ Α' ΛΥΚΕΙΟΥ (ΤΡΑΠΕΖΑ ΘΕΜΑΤΩΝ)
ΘΕΜΑΤΑ ΜΑΘΗΜΑΤΙΚΩΝ Α' ΛΥΚΕΙΟΥ (ΤΡΑΠΕΖΑ ΘΕΜΑΤΩΝ)
 
ισοτητα τριγωνων
ισοτητα τριγωνωνισοτητα τριγωνων
ισοτητα τριγωνων
 
200 άλυτες ασκήσεις γεωμετρίας κοπάδης
200 άλυτες ασκήσεις γεωμετρίας κοπάδης200 άλυτες ασκήσεις γεωμετρίας κοπάδης
200 άλυτες ασκήσεις γεωμετρίας κοπάδης
 
Geometria a lukeiou theoria askiseis
Geometria a lukeiou theoria askiseisGeometria a lukeiou theoria askiseis
Geometria a lukeiou theoria askiseis
 
ΓΕΩΜΕΤΡΙΑ Α ΓΥΜΝ ΘΕΩΡΙΑ.pdf
ΓΕΩΜΕΤΡΙΑ Α ΓΥΜΝ ΘΕΩΡΙΑ.pdfΓΕΩΜΕΤΡΙΑ Α ΓΥΜΝ ΘΕΩΡΙΑ.pdf
ΓΕΩΜΕΤΡΙΑ Α ΓΥΜΝ ΘΕΩΡΙΑ.pdf
 
ΓΕΩΜΕΤΡΙΑ Α ΓΥΜΝ ΘΕΩΡΙΑ.pdf
ΓΕΩΜΕΤΡΙΑ Α ΓΥΜΝ ΘΕΩΡΙΑ.pdfΓΕΩΜΕΤΡΙΑ Α ΓΥΜΝ ΘΕΩΡΙΑ.pdf
ΓΕΩΜΕΤΡΙΑ Α ΓΥΜΝ ΘΕΩΡΙΑ.pdf
 
ΕΜΒΑΔΟ ΤΡΙΓΩΝΟΥ
ΕΜΒΑΔΟ ΤΡΙΓΩΝΟΥΕΜΒΑΔΟ ΤΡΙΓΩΝΟΥ
ΕΜΒΑΔΟ ΤΡΙΓΩΝΟΥ
 
Θεωρία από τη Γεωμετρία Α΄ Λυκείου
Θεωρία από τη Γεωμετρία Α΄ ΛυκείουΘεωρία από τη Γεωμετρία Α΄ Λυκείου
Θεωρία από τη Γεωμετρία Α΄ Λυκείου
 
C geometria
C geometriaC geometria
C geometria
 
θεωρια της α΄λυκείου
θεωρια της α΄λυκείουθεωρια της α΄λυκείου
θεωρια της α΄λυκείου
 

Mais de Μάκης Χατζόπουλος

Σχόλια, κριτική, εκτιμήσεις και προτάσεις για τις εκλογές της ΕΜΕ
Σχόλια, κριτική, εκτιμήσεις και προτάσεις για τις εκλογές της ΕΜΕΣχόλια, κριτική, εκτιμήσεις και προτάσεις για τις εκλογές της ΕΜΕ
Σχόλια, κριτική, εκτιμήσεις και προτάσεις για τις εκλογές της ΕΜΕΜάκης Χατζόπουλος
 
Τι ΔΕΝ πρέπει να δούμε στις Πανελλαδικές Εξετάσεις;
Τι ΔΕΝ πρέπει να δούμε στις Πανελλαδικές Εξετάσεις; Τι ΔΕΝ πρέπει να δούμε στις Πανελλαδικές Εξετάσεις;
Τι ΔΕΝ πρέπει να δούμε στις Πανελλαδικές Εξετάσεις; Μάκης Χατζόπουλος
 
ΕΜΕ τεύχος 120: Α΄ Γυμνασίου ασκήσεις
ΕΜΕ τεύχος 120: Α΄ Γυμνασίου ασκήσειςΕΜΕ τεύχος 120: Α΄ Γυμνασίου ασκήσεις
ΕΜΕ τεύχος 120: Α΄ Γυμνασίου ασκήσειςΜάκης Χατζόπουλος
 
Μια γνωστή άσκηση του σχολικού βιβλίου με προεκτάσεις
Μια γνωστή άσκηση του σχολικού βιβλίου με προεκτάσειςΜια γνωστή άσκηση του σχολικού βιβλίου με προεκτάσεις
Μια γνωστή άσκηση του σχολικού βιβλίου με προεκτάσειςΜάκης Χατζόπουλος
 
Επαναληπτικό διαγώνισμα Γ Λυκείου [21/5/2021]
Επαναληπτικό διαγώνισμα Γ Λυκείου [21/5/2021]Επαναληπτικό διαγώνισμα Γ Λυκείου [21/5/2021]
Επαναληπτικό διαγώνισμα Γ Λυκείου [21/5/2021]Μάκης Χατζόπουλος
 
Διδακτικά σενάρια στη Γ΄ Λυκείου
Διδακτικά σενάρια στη Γ΄ Λυκείου Διδακτικά σενάρια στη Γ΄ Λυκείου
Διδακτικά σενάρια στη Γ΄ Λυκείου Μάκης Χατζόπουλος
 
2 Κριτήρια Αξιολόγησης από τον Βασίλη Παπαδάκη και Φάνη Μαργαρώνη
2 Κριτήρια Αξιολόγησης από τον Βασίλη Παπαδάκη και Φάνη Μαργαρώνη2 Κριτήρια Αξιολόγησης από τον Βασίλη Παπαδάκη και Φάνη Μαργαρώνη
2 Κριτήρια Αξιολόγησης από τον Βασίλη Παπαδάκη και Φάνη ΜαργαρώνηΜάκης Χατζόπουλος
 
Διαγώνισμα Β Λυκείου επαναληπτικό
Διαγώνισμα Β Λυκείου επαναληπτικόΔιαγώνισμα Β Λυκείου επαναληπτικό
Διαγώνισμα Β Λυκείου επαναληπτικόΜάκης Χατζόπουλος
 
H εισήγηση στο Εκπαιδευτικό σεμινάριο που διεξάχθηκε από τα Φροντιστήρια "Εν ...
H εισήγηση στο Εκπαιδευτικό σεμινάριο που διεξάχθηκε από τα Φροντιστήρια "Εν ...H εισήγηση στο Εκπαιδευτικό σεμινάριο που διεξάχθηκε από τα Φροντιστήρια "Εν ...
H εισήγηση στο Εκπαιδευτικό σεμινάριο που διεξάχθηκε από τα Φροντιστήρια "Εν ...Μάκης Χατζόπουλος
 
Θεωρία - Ορισμοί - Προτάσεις 2021 - Γ Λυκείου
Θεωρία - Ορισμοί - Προτάσεις 2021 - Γ Λυκείου Θεωρία - Ορισμοί - Προτάσεις 2021 - Γ Λυκείου
Θεωρία - Ορισμοί - Προτάσεις 2021 - Γ Λυκείου Μάκης Χατζόπουλος
 
Διδακτικό σενάριο Α΄ Λυκείου [2021]
Διδακτικό σενάριο Α΄ Λυκείου [2021]Διδακτικό σενάριο Α΄ Λυκείου [2021]
Διδακτικό σενάριο Α΄ Λυκείου [2021]Μάκης Χατζόπουλος
 
Διαγώνισμα Γ Λυκείου ( 2.6 έως 2.10) από το Καλαμαρί
Διαγώνισμα Γ Λυκείου ( 2.6 έως 2.10) από το ΚαλαμαρίΔιαγώνισμα Γ Λυκείου ( 2.6 έως 2.10) από το Καλαμαρί
Διαγώνισμα Γ Λυκείου ( 2.6 έως 2.10) από το ΚαλαμαρίΜάκης Χατζόπουλος
 
Διαγώνισμα Γ Λυκείου από Σούρμπη
Διαγώνισμα Γ Λυκείου από ΣούρμπηΔιαγώνισμα Γ Λυκείου από Σούρμπη
Διαγώνισμα Γ Λυκείου από ΣούρμπηΜάκης Χατζόπουλος
 

Mais de Μάκης Χατζόπουλος (20)

Εσείς πώς τα διδάσκετε;
Εσείς πώς τα διδάσκετε;Εσείς πώς τα διδάσκετε;
Εσείς πώς τα διδάσκετε;
 
Σχόλια, κριτική, εκτιμήσεις και προτάσεις για τις εκλογές της ΕΜΕ
Σχόλια, κριτική, εκτιμήσεις και προτάσεις για τις εκλογές της ΕΜΕΣχόλια, κριτική, εκτιμήσεις και προτάσεις για τις εκλογές της ΕΜΕ
Σχόλια, κριτική, εκτιμήσεις και προτάσεις για τις εκλογές της ΕΜΕ
 
Πανελλαδικές Εξετάσεις 2021 ΕΠΑΛ
Πανελλαδικές Εξετάσεις 2021 ΕΠΑΛΠανελλαδικές Εξετάσεις 2021 ΕΠΑΛ
Πανελλαδικές Εξετάσεις 2021 ΕΠΑΛ
 
Τι ΔΕΝ πρέπει να δούμε στις Πανελλαδικές Εξετάσεις;
Τι ΔΕΝ πρέπει να δούμε στις Πανελλαδικές Εξετάσεις; Τι ΔΕΝ πρέπει να δούμε στις Πανελλαδικές Εξετάσεις;
Τι ΔΕΝ πρέπει να δούμε στις Πανελλαδικές Εξετάσεις;
 
ΕΜΕ τεύχος 120: Α΄ Γυμνασίου ασκήσεις
ΕΜΕ τεύχος 120: Α΄ Γυμνασίου ασκήσειςΕΜΕ τεύχος 120: Α΄ Γυμνασίου ασκήσεις
ΕΜΕ τεύχος 120: Α΄ Γυμνασίου ασκήσεις
 
Μια γνωστή άσκηση του σχολικού βιβλίου με προεκτάσεις
Μια γνωστή άσκηση του σχολικού βιβλίου με προεκτάσειςΜια γνωστή άσκηση του σχολικού βιβλίου με προεκτάσεις
Μια γνωστή άσκηση του σχολικού βιβλίου με προεκτάσεις
 
Ξεφτέρης Μαστερίδης σενάριο 3ο
Ξεφτέρης Μαστερίδης σενάριο 3οΞεφτέρης Μαστερίδης σενάριο 3ο
Ξεφτέρης Μαστερίδης σενάριο 3ο
 
Επαναληπτικό διαγώνισμα Γ Λυκείου [21/5/2021]
Επαναληπτικό διαγώνισμα Γ Λυκείου [21/5/2021]Επαναληπτικό διαγώνισμα Γ Λυκείου [21/5/2021]
Επαναληπτικό διαγώνισμα Γ Λυκείου [21/5/2021]
 
45+1 Θέματα Γ Λυκείου
45+1 Θέματα Γ Λυκείου 45+1 Θέματα Γ Λυκείου
45+1 Θέματα Γ Λυκείου
 
Διδακτικά σενάρια στη Γ΄ Λυκείου
Διδακτικά σενάρια στη Γ΄ Λυκείου Διδακτικά σενάρια στη Γ΄ Λυκείου
Διδακτικά σενάρια στη Γ΄ Λυκείου
 
2 Κριτήρια Αξιολόγησης από τον Βασίλη Παπαδάκη και Φάνη Μαργαρώνη
2 Κριτήρια Αξιολόγησης από τον Βασίλη Παπαδάκη και Φάνη Μαργαρώνη2 Κριτήρια Αξιολόγησης από τον Βασίλη Παπαδάκη και Φάνη Μαργαρώνη
2 Κριτήρια Αξιολόγησης από τον Βασίλη Παπαδάκη και Φάνη Μαργαρώνη
 
Σωστό - Λάθος Γ Λυκείου 2021
Σωστό - Λάθος Γ Λυκείου 2021Σωστό - Λάθος Γ Λυκείου 2021
Σωστό - Λάθος Γ Λυκείου 2021
 
Διαγώνισμα Β Λυκείου επαναληπτικό
Διαγώνισμα Β Λυκείου επαναληπτικόΔιαγώνισμα Β Λυκείου επαναληπτικό
Διαγώνισμα Β Λυκείου επαναληπτικό
 
H εισήγηση στο Εκπαιδευτικό σεμινάριο που διεξάχθηκε από τα Φροντιστήρια "Εν ...
H εισήγηση στο Εκπαιδευτικό σεμινάριο που διεξάχθηκε από τα Φροντιστήρια "Εν ...H εισήγηση στο Εκπαιδευτικό σεμινάριο που διεξάχθηκε από τα Φροντιστήρια "Εν ...
H εισήγηση στο Εκπαιδευτικό σεμινάριο που διεξάχθηκε από τα Φροντιστήρια "Εν ...
 
Θεωρία - Ορισμοί - Προτάσεις 2021 - Γ Λυκείου
Θεωρία - Ορισμοί - Προτάσεις 2021 - Γ Λυκείου Θεωρία - Ορισμοί - Προτάσεις 2021 - Γ Λυκείου
Θεωρία - Ορισμοί - Προτάσεις 2021 - Γ Λυκείου
 
Διδακτικό σενάριο Α΄ Λυκείου [2021]
Διδακτικό σενάριο Α΄ Λυκείου [2021]Διδακτικό σενάριο Α΄ Λυκείου [2021]
Διδακτικό σενάριο Α΄ Λυκείου [2021]
 
Διαγώνισμα Γ Λυκείου ( 2.6 έως 2.10) από το Καλαμαρί
Διαγώνισμα Γ Λυκείου ( 2.6 έως 2.10) από το ΚαλαμαρίΔιαγώνισμα Γ Λυκείου ( 2.6 έως 2.10) από το Καλαμαρί
Διαγώνισμα Γ Λυκείου ( 2.6 έως 2.10) από το Καλαμαρί
 
Κεφάλαιο 7ο - Α΄ Γυμνασίου
Κεφάλαιο 7ο - Α΄ ΓυμνασίουΚεφάλαιο 7ο - Α΄ Γυμνασίου
Κεφάλαιο 7ο - Α΄ Γυμνασίου
 
G luk eykleidhs b 118_eykleidhs_2021
G luk eykleidhs b 118_eykleidhs_2021G luk eykleidhs b 118_eykleidhs_2021
G luk eykleidhs b 118_eykleidhs_2021
 
Διαγώνισμα Γ Λυκείου από Σούρμπη
Διαγώνισμα Γ Λυκείου από ΣούρμπηΔιαγώνισμα Γ Λυκείου από Σούρμπη
Διαγώνισμα Γ Λυκείου από Σούρμπη
 

Último

7. 3. ΔΙΔΑΚΤΙΚΕΣ ΣΤΡΑΤΗΓΙΚΕΣ ΤΗΣ ΓΛΩΣΣΑΣ.pdf
7. 3. ΔΙΔΑΚΤΙΚΕΣ ΣΤΡΑΤΗΓΙΚΕΣ ΤΗΣ ΓΛΩΣΣΑΣ.pdf7. 3. ΔΙΔΑΚΤΙΚΕΣ ΣΤΡΑΤΗΓΙΚΕΣ ΤΗΣ ΓΛΩΣΣΑΣ.pdf
7. 3. ΔΙΔΑΚΤΙΚΕΣ ΣΤΡΑΤΗΓΙΚΕΣ ΤΗΣ ΓΛΩΣΣΑΣ.pdfssuser3e0dbe
 
Παρουσίαση καλλιτεχνικού θεάματος
Παρουσίαση          καλλιτεχνικού θεάματοςΠαρουσίαση          καλλιτεχνικού θεάματος
Παρουσίαση καλλιτεχνικού θεάματοςDimitra Mylonaki
 
Επίσκεψη μαθητών στην Έκθεση Η Μαγεία των Μοτίβων.pptx
Επίσκεψη μαθητών στην Έκθεση Η Μαγεία των Μοτίβων.pptxΕπίσκεψη μαθητών στην Έκθεση Η Μαγεία των Μοτίβων.pptx
Επίσκεψη μαθητών στην Έκθεση Η Μαγεία των Μοτίβων.pptx7gymnasiokavalas
 
Παρουσίαση καλλιτεχνικού θεάματος
Παρουσίαση                  καλλιτεχνικού θεάματοςΠαρουσίαση                  καλλιτεχνικού θεάματος
Παρουσίαση καλλιτεχνικού θεάματοςDimitra Mylonaki
 
Η κυρία Αλφαβήτα και τα παιδιά της. Της Σάσας Καραγιαννίδου - Πέννα
Η κυρία Αλφαβήτα και τα παιδιά της. Της Σάσας Καραγιαννίδου - ΠένναΗ κυρία Αλφαβήτα και τα παιδιά της. Της Σάσας Καραγιαννίδου - Πέννα
Η κυρία Αλφαβήτα και τα παιδιά της. Της Σάσας Καραγιαννίδου - ΠένναΣάσα Καραγιαννίδου - Πέννα
 
Κωνσταντής σημειώσεις κείμενα νεοελληνικής
Κωνσταντής σημειώσεις κείμενα νεοελληνικήςΚωνσταντής σημειώσεις κείμενα νεοελληνικής
Κωνσταντής σημειώσεις κείμενα νεοελληνικήςssuser44c0dc
 
ΣΤ2 -ΕΓΩ ΚΑΙ ΣΥ ΜΑΖΙ-ΦΙΛΟΙ ΠΑΝΤΟΤΙΝΟΙ .pdf
ΣΤ2 -ΕΓΩ ΚΑΙ ΣΥ ΜΑΖΙ-ΦΙΛΟΙ ΠΑΝΤΟΤΙΝΟΙ .pdfΣΤ2 -ΕΓΩ ΚΑΙ ΣΥ ΜΑΖΙ-ΦΙΛΟΙ ΠΑΝΤΟΤΙΝΟΙ .pdf
ΣΤ2 -ΕΓΩ ΚΑΙ ΣΥ ΜΑΖΙ-ΦΙΛΟΙ ΠΑΝΤΟΤΙΝΟΙ .pdfChrisa Kokorikou
 
Έκθεση μαθητικής Ζωγραφικής- Η μαγεία των μοτίβων.pptx
Έκθεση μαθητικής Ζωγραφικής- Η μαγεία των μοτίβων.pptxΈκθεση μαθητικής Ζωγραφικής- Η μαγεία των μοτίβων.pptx
Έκθεση μαθητικής Ζωγραφικής- Η μαγεία των μοτίβων.pptx7gymnasiokavalas
 
Εξερευνώντας τα μυστήρια του ουρανού-Παρουσίαση.pptx
Εξερευνώντας τα μυστήρια του ουρανού-Παρουσίαση.pptxΕξερευνώντας τα μυστήρια του ουρανού-Παρουσίαση.pptx
Εξερευνώντας τα μυστήρια του ουρανού-Παρουσίαση.pptxntanavara
 
Ξενάγηση στο ιστορικό κέντρο των Ιωαννίνων.pptx
Ξενάγηση στο ιστορικό κέντρο των Ιωαννίνων.pptxΞενάγηση στο ιστορικό κέντρο των Ιωαννίνων.pptx
Ξενάγηση στο ιστορικό κέντρο των Ιωαννίνων.pptxDimitraKarabali
 
Οι μικροί αρτοποιοί της Γ τάξης και το ψωμί τους.pptx
Οι μικροί αρτοποιοί της Γ τάξης και το ψωμί τους.pptxΟι μικροί αρτοποιοί της Γ τάξης και το ψωμί τους.pptx
Οι μικροί αρτοποιοί της Γ τάξης και το ψωμί τους.pptx36dimperist
 
B2 TΑΞΗ -ΜΗΝΥΜΑΤΑ ΓΙΑ ΤΑ ΑΔΕΣΠΟΤΑ.pdf-ΜΑΡΚΕΛΛΑ ΤΣΑΤΣΑΡΩΝΗ
B2 TΑΞΗ -ΜΗΝΥΜΑΤΑ ΓΙΑ ΤΑ ΑΔΕΣΠΟΤΑ.pdf-ΜΑΡΚΕΛΛΑ ΤΣΑΤΣΑΡΩΝΗB2 TΑΞΗ -ΜΗΝΥΜΑΤΑ ΓΙΑ ΤΑ ΑΔΕΣΠΟΤΑ.pdf-ΜΑΡΚΕΛΛΑ ΤΣΑΤΣΑΡΩΝΗ
B2 TΑΞΗ -ΜΗΝΥΜΑΤΑ ΓΙΑ ΤΑ ΑΔΕΣΠΟΤΑ.pdf-ΜΑΡΚΕΛΛΑ ΤΣΑΤΣΑΡΩΝΗChrisa Kokorikou
 
ΣΠΑΣΕ ΤΗ ΣΙΩΠΗ ΑΠΟ ΤΟΥΣ ΜΑΘΗΤΕΣ/ΤΡΙΕΣ ΤΟΥ Β2.pdf
ΣΠΑΣΕ ΤΗ ΣΙΩΠΗ ΑΠΟ ΤΟΥΣ ΜΑΘΗΤΕΣ/ΤΡΙΕΣ ΤΟΥ Β2.pdfΣΠΑΣΕ ΤΗ ΣΙΩΠΗ ΑΠΟ ΤΟΥΣ ΜΑΘΗΤΕΣ/ΤΡΙΕΣ ΤΟΥ Β2.pdf
ΣΠΑΣΕ ΤΗ ΣΙΩΠΗ ΑΠΟ ΤΟΥΣ ΜΑΘΗΤΕΣ/ΤΡΙΕΣ ΤΟΥ Β2.pdfChrisa Kokorikou
 
ΝΕΕΣ ΚΟΥΡΤΙΝΕΣ ΜΕ ΔΩΡΕΑ ΤΟΥ ΣΥΛΛΟΓΟΥ ΓΟΝΕΩΝ.pptx
ΝΕΕΣ ΚΟΥΡΤΙΝΕΣ ΜΕ ΔΩΡΕΑ ΤΟΥ ΣΥΛΛΟΓΟΥ ΓΟΝΕΩΝ.pptxΝΕΕΣ ΚΟΥΡΤΙΝΕΣ ΜΕ ΔΩΡΕΑ ΤΟΥ ΣΥΛΛΟΓΟΥ ΓΟΝΕΩΝ.pptx
ΝΕΕΣ ΚΟΥΡΤΙΝΕΣ ΜΕ ΔΩΡΕΑ ΤΟΥ ΣΥΛΛΟΓΟΥ ΓΟΝΕΩΝ.pptx41dimperisteriou
 
Η εποχή του Ιουστινιανού-Η ελληνοχριστιανική οικουμένη
Η εποχή του Ιουστινιανού-Η ελληνοχριστιανική οικουμένηΗ εποχή του Ιουστινιανού-Η ελληνοχριστιανική οικουμένη
Η εποχή του Ιουστινιανού-Η ελληνοχριστιανική οικουμένηEvangelia Patera
 
Παρουσίαση ομάδας ECOMOBILITY Σχολείου Δεύτερης Ευκαιρίας Άρτας
Παρουσίαση ομάδας ECOMOBILITY Σχολείου Δεύτερης Ευκαιρίας ΆρταςΠαρουσίαση ομάδας ECOMOBILITY Σχολείου Δεύτερης Ευκαιρίας Άρτας
Παρουσίαση ομάδας ECOMOBILITY Σχολείου Δεύτερης Ευκαιρίας Άρταςsdeartas
 
Ημέρα Επιστημών – Επίδειξη πειραμάτων από τους μαθητές.pptx
Ημέρα Επιστημών – Επίδειξη πειραμάτων από τους μαθητές.pptxΗμέρα Επιστημών – Επίδειξη πειραμάτων από τους μαθητές.pptx
Ημέρα Επιστημών – Επίδειξη πειραμάτων από τους μαθητές.pptx36dimperist
 
Πρόγραμμα - Πάμε μια βόλτα στο φεγγάρι.pptx
Πρόγραμμα - Πάμε μια βόλτα στο φεγγάρι.pptxΠρόγραμμα - Πάμε μια βόλτα στο φεγγάρι.pptx
Πρόγραμμα - Πάμε μια βόλτα στο φεγγάρι.pptxntanavara
 

Último (20)

7. 3. ΔΙΔΑΚΤΙΚΕΣ ΣΤΡΑΤΗΓΙΚΕΣ ΤΗΣ ΓΛΩΣΣΑΣ.pdf
7. 3. ΔΙΔΑΚΤΙΚΕΣ ΣΤΡΑΤΗΓΙΚΕΣ ΤΗΣ ΓΛΩΣΣΑΣ.pdf7. 3. ΔΙΔΑΚΤΙΚΕΣ ΣΤΡΑΤΗΓΙΚΕΣ ΤΗΣ ΓΛΩΣΣΑΣ.pdf
7. 3. ΔΙΔΑΚΤΙΚΕΣ ΣΤΡΑΤΗΓΙΚΕΣ ΤΗΣ ΓΛΩΣΣΑΣ.pdf
 
Παρουσίαση καλλιτεχνικού θεάματος
Παρουσίαση          καλλιτεχνικού θεάματοςΠαρουσίαση          καλλιτεχνικού θεάματος
Παρουσίαση καλλιτεχνικού θεάματος
 
Επίσκεψη μαθητών στην Έκθεση Η Μαγεία των Μοτίβων.pptx
Επίσκεψη μαθητών στην Έκθεση Η Μαγεία των Μοτίβων.pptxΕπίσκεψη μαθητών στην Έκθεση Η Μαγεία των Μοτίβων.pptx
Επίσκεψη μαθητών στην Έκθεση Η Μαγεία των Μοτίβων.pptx
 
Παρουσίαση καλλιτεχνικού θεάματος
Παρουσίαση                  καλλιτεχνικού θεάματοςΠαρουσίαση                  καλλιτεχνικού θεάματος
Παρουσίαση καλλιτεχνικού θεάματος
 
Η κυρία Αλφαβήτα και τα παιδιά της. Της Σάσας Καραγιαννίδου - Πέννα
Η κυρία Αλφαβήτα και τα παιδιά της. Της Σάσας Καραγιαννίδου - ΠένναΗ κυρία Αλφαβήτα και τα παιδιά της. Της Σάσας Καραγιαννίδου - Πέννα
Η κυρία Αλφαβήτα και τα παιδιά της. Της Σάσας Καραγιαννίδου - Πέννα
 
Κωνσταντής σημειώσεις κείμενα νεοελληνικής
Κωνσταντής σημειώσεις κείμενα νεοελληνικήςΚωνσταντής σημειώσεις κείμενα νεοελληνικής
Κωνσταντής σημειώσεις κείμενα νεοελληνικής
 
Λαπμπουκ .pdf
Λαπμπουκ                                                    .pdfΛαπμπουκ                                                    .pdf
Λαπμπουκ .pdf
 
ΣΤ2 -ΕΓΩ ΚΑΙ ΣΥ ΜΑΖΙ-ΦΙΛΟΙ ΠΑΝΤΟΤΙΝΟΙ .pdf
ΣΤ2 -ΕΓΩ ΚΑΙ ΣΥ ΜΑΖΙ-ΦΙΛΟΙ ΠΑΝΤΟΤΙΝΟΙ .pdfΣΤ2 -ΕΓΩ ΚΑΙ ΣΥ ΜΑΖΙ-ΦΙΛΟΙ ΠΑΝΤΟΤΙΝΟΙ .pdf
ΣΤ2 -ΕΓΩ ΚΑΙ ΣΥ ΜΑΖΙ-ΦΙΛΟΙ ΠΑΝΤΟΤΙΝΟΙ .pdf
 
Έκθεση μαθητικής Ζωγραφικής- Η μαγεία των μοτίβων.pptx
Έκθεση μαθητικής Ζωγραφικής- Η μαγεία των μοτίβων.pptxΈκθεση μαθητικής Ζωγραφικής- Η μαγεία των μοτίβων.pptx
Έκθεση μαθητικής Ζωγραφικής- Η μαγεία των μοτίβων.pptx
 
Εξερευνώντας τα μυστήρια του ουρανού-Παρουσίαση.pptx
Εξερευνώντας τα μυστήρια του ουρανού-Παρουσίαση.pptxΕξερευνώντας τα μυστήρια του ουρανού-Παρουσίαση.pptx
Εξερευνώντας τα μυστήρια του ουρανού-Παρουσίαση.pptx
 
Στο μουσείο
Στο                                        μουσείοΣτο                                        μουσείο
Στο μουσείο
 
Ξενάγηση στο ιστορικό κέντρο των Ιωαννίνων.pptx
Ξενάγηση στο ιστορικό κέντρο των Ιωαννίνων.pptxΞενάγηση στο ιστορικό κέντρο των Ιωαννίνων.pptx
Ξενάγηση στο ιστορικό κέντρο των Ιωαννίνων.pptx
 
Οι μικροί αρτοποιοί της Γ τάξης και το ψωμί τους.pptx
Οι μικροί αρτοποιοί της Γ τάξης και το ψωμί τους.pptxΟι μικροί αρτοποιοί της Γ τάξης και το ψωμί τους.pptx
Οι μικροί αρτοποιοί της Γ τάξης και το ψωμί τους.pptx
 
B2 TΑΞΗ -ΜΗΝΥΜΑΤΑ ΓΙΑ ΤΑ ΑΔΕΣΠΟΤΑ.pdf-ΜΑΡΚΕΛΛΑ ΤΣΑΤΣΑΡΩΝΗ
B2 TΑΞΗ -ΜΗΝΥΜΑΤΑ ΓΙΑ ΤΑ ΑΔΕΣΠΟΤΑ.pdf-ΜΑΡΚΕΛΛΑ ΤΣΑΤΣΑΡΩΝΗB2 TΑΞΗ -ΜΗΝΥΜΑΤΑ ΓΙΑ ΤΑ ΑΔΕΣΠΟΤΑ.pdf-ΜΑΡΚΕΛΛΑ ΤΣΑΤΣΑΡΩΝΗ
B2 TΑΞΗ -ΜΗΝΥΜΑΤΑ ΓΙΑ ΤΑ ΑΔΕΣΠΟΤΑ.pdf-ΜΑΡΚΕΛΛΑ ΤΣΑΤΣΑΡΩΝΗ
 
ΣΠΑΣΕ ΤΗ ΣΙΩΠΗ ΑΠΟ ΤΟΥΣ ΜΑΘΗΤΕΣ/ΤΡΙΕΣ ΤΟΥ Β2.pdf
ΣΠΑΣΕ ΤΗ ΣΙΩΠΗ ΑΠΟ ΤΟΥΣ ΜΑΘΗΤΕΣ/ΤΡΙΕΣ ΤΟΥ Β2.pdfΣΠΑΣΕ ΤΗ ΣΙΩΠΗ ΑΠΟ ΤΟΥΣ ΜΑΘΗΤΕΣ/ΤΡΙΕΣ ΤΟΥ Β2.pdf
ΣΠΑΣΕ ΤΗ ΣΙΩΠΗ ΑΠΟ ΤΟΥΣ ΜΑΘΗΤΕΣ/ΤΡΙΕΣ ΤΟΥ Β2.pdf
 
ΝΕΕΣ ΚΟΥΡΤΙΝΕΣ ΜΕ ΔΩΡΕΑ ΤΟΥ ΣΥΛΛΟΓΟΥ ΓΟΝΕΩΝ.pptx
ΝΕΕΣ ΚΟΥΡΤΙΝΕΣ ΜΕ ΔΩΡΕΑ ΤΟΥ ΣΥΛΛΟΓΟΥ ΓΟΝΕΩΝ.pptxΝΕΕΣ ΚΟΥΡΤΙΝΕΣ ΜΕ ΔΩΡΕΑ ΤΟΥ ΣΥΛΛΟΓΟΥ ΓΟΝΕΩΝ.pptx
ΝΕΕΣ ΚΟΥΡΤΙΝΕΣ ΜΕ ΔΩΡΕΑ ΤΟΥ ΣΥΛΛΟΓΟΥ ΓΟΝΕΩΝ.pptx
 
Η εποχή του Ιουστινιανού-Η ελληνοχριστιανική οικουμένη
Η εποχή του Ιουστινιανού-Η ελληνοχριστιανική οικουμένηΗ εποχή του Ιουστινιανού-Η ελληνοχριστιανική οικουμένη
Η εποχή του Ιουστινιανού-Η ελληνοχριστιανική οικουμένη
 
Παρουσίαση ομάδας ECOMOBILITY Σχολείου Δεύτερης Ευκαιρίας Άρτας
Παρουσίαση ομάδας ECOMOBILITY Σχολείου Δεύτερης Ευκαιρίας ΆρταςΠαρουσίαση ομάδας ECOMOBILITY Σχολείου Δεύτερης Ευκαιρίας Άρτας
Παρουσίαση ομάδας ECOMOBILITY Σχολείου Δεύτερης Ευκαιρίας Άρτας
 
Ημέρα Επιστημών – Επίδειξη πειραμάτων από τους μαθητές.pptx
Ημέρα Επιστημών – Επίδειξη πειραμάτων από τους μαθητές.pptxΗμέρα Επιστημών – Επίδειξη πειραμάτων από τους μαθητές.pptx
Ημέρα Επιστημών – Επίδειξη πειραμάτων από τους μαθητές.pptx
 
Πρόγραμμα - Πάμε μια βόλτα στο φεγγάρι.pptx
Πρόγραμμα - Πάμε μια βόλτα στο φεγγάρι.pptxΠρόγραμμα - Πάμε μια βόλτα στο φεγγάρι.pptx
Πρόγραμμα - Πάμε μια βόλτα στο φεγγάρι.pptx
 

Διαγωνισμός Αρχιμήδης (μικροί) - 56 Ασκήσεις Γεωμετρίας με λύσεις

  • 1. Μπάμπης Στεργίου Διαγωνισμός Αρχιμήδης Juniors-Μικροί Ασκήσεις στη Γεωμετρία *** Αφιερωμένο στους μαθητές και τους συναδέλφους 2017
  • 2.
  • 3. ΘΕΩΡΙΑ ΚΑΙ ΕΦΑΡΜΟΓΕΣ Α. Τρίγωνα – Τετράπλευρα 1Α . Τρίγωνα Θεωρήματα α) Οι μεσοκάθετοι των πλευρών κάθε τριγώνου διέρχονται από το ίδιο σημείο (συντρέχουν). Το σημείο αυτό λέγεται περίκεντρο του τριγώνου αυτού και είναι το κέντρο του περιγεγραμμένου κύκλου. β) Οι διχοτόμοι των γωνιών κάθε τριγώνου διέρχονται από το ίδιο σημείο (συντρέχουν). Το σημείο αυτό λέγεται έγκεντρο του τριγώνου και είναι το κέντρο του εγγεγραμμένου κύκλου. γ) Οι διχοτόμοι δύο εξωτερικών γωνιών και της τρίτης εσωτερικής γωνίας διέρχονται από το ίδιο σημείο. Το σημείο αυτό λέγεται παράκεντρο. Υπάρχουν τρία παράκεντρα τα οποία είναι κέντρα των παρεγγεγραμμένων κύκλων. Οι κύκλοι αυτοί εφάπτονται στη μια πλευρά του τριγώνου και στις προεκτάσεις των δύο άλλων. Τα παράκεντρα και οι παρεγγεγραμμένοι κύκλοι ενός τριγώνου φαίνονται στο διπλανό σχήμα.
  • 4. 2. Άθροισμα γωνιών τριγώνου – τετραπλεύρου Θεωρήματα α) Το άθροισμα των γωνιών κάθε τριγώνου είναι ο 180 . Δηλαδή σε κάθε τρίγωνο ΑΒΓ ισχύει: οˆ ˆ ˆΑ Β Γ 180   β) Η εξωτερική γωνία τριγώνου είναι ίση με το άθροισμα των δύο απέναντι εσωτερικών του. Έτσι: εξ ˆˆ ˆΓ Α Β  ή ω x y  γ)  Δύο τρίγωνα που έχουν δύο γωνίες ίσες, έχουν και τις τρίτες ίσες.  Οι οξείες γωνίες ενός ορθογωνίου τριγώνου είναι συμπληρωματικές.  Κάθε γωνία ισοπλεύρου τριγώνου είναι ίση με o 60 .  Ένα ισοσκελές τρίγωνο με μία γωνία o 60 είναι ισόπλευρο. Θεώρημα Δύο γωνίες που έχουν τις πλευρές τους κάθετες μία προς μία είναι:  ίσες, αν είναι και οι δύο οξείες ή αμβλείες (έτσι, θα είναι x y) ,  παραπληρωματικές, αν είναι η μία οξεία και η άλλη αμβλεία (έτσι, θα είναι ο x ω 180  ). Θεώρημα α) Το άθροισμα των γωνιών κάθε κυρτού ν-γώνου είναι (2ν 4) ορθές ή ο (180ν 360) . β) Το άθροισμα των εξωτερικών γωνιών ενός κυρτού ν-γώνου είναι 4 ορθές ή ο 360 .
  • 5. Θεώρημα α) Οι διχοτόμοι των γωνιών ˆΒ και ˆΓ ενός τριγώνου σχηματίζουν γωνία ίση με ο ˆΑ 90 2  , δηλαδή: ο ˆΑˆΒΙΓ 90 2   β) Οι διχοτόμοι των ˆΒ και εξ ˆΓ σχηματίζουν γωνία ίση με ˆΑ 2 . Έτσι: β ˆΑˆΙ 2  γ) Οι διχοτόμοι των εξ ˆΒ και εξ ˆΓ σχηματίζουν γωνία ίση με ο ˆΑ 90 2  . Έτσι: ο α ˆΑˆΙ 90 2   1.1 Δίνεται ισοσκελές τρίγωνο ΑΒΓ, με ΑΒ = ΑΓ και οˆΑ = 20 . Στις πλευρές ΑΓ και ΑΒ παίρνουμε σημεία Δ και Ε αντίστοιχα, ώστε οˆΔΒΓ = 50 και οˆΕΓΒ = 60 . Να αποδειχθεί ότι οˆΓΕΔ = 30 . Λύση Παίρνουμε στην ΑΒ σημείο Ζ, ώστε οˆΖΓΒ 20 . Τότε:  ΓΒ ΓΖ , αφού το τρίγωνο ΓΒΖ είναι ισοσκελές,  ΓΒ ΓΔ , αφού το τρίγωνο ΓΒΔ είναι ισοσκελές,  ΖΓ ΖΕ , αφού το τρίγωνο ΖΕΓ είναι ισοσκελές. Αλλά το τρίγωνο ΓΔΖ θα είναι τότε ισόπλευρο, διότι οˆΖΓΔ 60 . Άρα ΖΔ ΖΓ ΖΕ  , οπότε: ο ο ο ο ˆ180 ΕΖΔ 180 40ˆΖΕΔ 70 2 2      Τελικά έχουμε: ο ο οˆ ˆ ˆΓΕΔ ΖΕΔ ΖΕΓ 70 40 30     1.2 Δίνεται τρίγωνο ΑΒΓ και η διχοτόμος ΑΔ. Αν ισχύει ότι ΑΒ + ΓΔ = ΑΓ + ΒΔ , να αποδειχθεί ότι
  • 6. ΑΒ = ΑΓ . Λύση Θα χρησιμοποιήσουμε βοηθητικές γραμμές. Στις προεκτάσεις των πλευ- ρών ΑΒ και ΑΓ παίρνουμε τμήμα ΒΕ ΓΔ y  και ΓΖ ΒΔ x  αντίστοιχα. Έχουμε ότι: AB ΓΔ ΑΓ ΒΔ    ΑΒ ΒΕ ΑΓ ΓΖ ΑΕ ΑΖ      Το τρίγωνο ΑΕΖ είναι οπότε ισοσκελές και έτσι η διχοτόμος ΑΔ της γωνίας ˆΑ είναι μεσοκάθετος της ΕΖ. Άρα ΔΕ ΔΖ . Έτσι τα τρίγωνα ΒΔΕ και ΓΔΖ είναι ίσα (έχουν τις πλευρές τους μία προς μία ίσες), οπότε: ˆ ˆΕΒΔ ΖΓΔ Επομένως ˆ ˆΒ Γ , ως παραπληρωματικές των ίσων γωνιών ˆΕΒΔ και ˆΖΓΔ . Άρα το τρίγωνο ΑΒΓ είναι ισοσκελές, οπότε ΑΒ ΑΓ . 1.3 Σε ένα τρίγωνο ΑΒΓ, οι διχοτόμοι των εξωτερικών γωνιών ˆΒ και ˆΓ τέμνονται στο σημείο Ο. Στην προέκταση της ΑΒ, προς το Β, παίρνουμε τμήμα ΒΔ = ΒΓ + ΑΓ . Να αποδειχθεί ότι ΟΔ = ΟΑ . Λύση Στην προέκταση της ΑΓ παίρνουμε τμήμα ΓΕ ΓΒ α  . Φέρνουμε τις ΟΑ και ΒΕ. Το τρίγωνο ΓΒΕ είναι ισοσκελές, αφού ΓΒ ΓΕ α  . Επομένως η διχοτόμος ΓΟ της εξ ˆΓ είναι μεσοκάθετος της ΒΕ. Άρα: ΟΒ ΟΕ (1) Από την ισότητα των τριγώνων ΒΟΓ και ΕΟΓ παίρνουμε ότι ˆ ˆΓΒΟ Ε . Έτσι: ˆ ˆΕ ΔΒΟ ω  (2) Τα τρίγωνα τώρα ΟΒΔ και ΟΑΕ είναι ίσα, διότι:  ΟΒ ΟΕ , λόγω της σχέσης (1),  ΒΔ ΑΕ α β   και  ˆ ˆΔΒΟ ΟΕΑ ω  , λόγω της σχέσης (2). Επομένως θα είναι και ΟΔ ΟΑ .
  • 7. 2Α . Παραλληλόγραμμα Ορισμός α) Παραλληλόγραμμο ονομάζεται το τετράπλευρο που έχει τις απέναντι πλευρές του παράλληλες. β) Αν το ΑΒΓΔ είναι παραλληλόγραμμο, τότε ΑΒ // ΓΔ και ΑΔ // ΒΓ. Ιδιότητες Στο παραλληλόγραμμο ισχύουν οι εξής ιδιότητες:  Οι απέναντι πλευρές είναι ίσες. Έτσι ΑΒ ΓΔ και ΑΔ ΒΓ .  Οι απέναντι γωνίες είναι ίσες. Έτσι ˆ ˆΑ Γ και ˆˆΒ Δ .  Οι διαγώνιοι διχοτομούνται. Αυτό σημαίνει ότι ΟΑ ΟΓ και ΟΒ ΟΔ .  Το κέντρο του παραλληλογράμμου, δηλαδή το σημείο τομής των διαγωνίων του, είναι κέντρο συμμετρίας του παραλληλογράμμου. Κριτήρια Για να αποδείξουμε ότι ένα τετράπλευρο είναι παραλληλόγραμμο, αρκεί να αποδείξουμε ότι ισχύει ένα από τα παρακάτω (κριτήρια):  Οι απέναντι πλευρές είναι παράλληλες.  Οι απέναντι πλευρές είναι ίσες.  Οι απέναντι γωνίες είναι ίσες.  Οι διαγώνιοι διχοτομούνται.  Δύο απέναντι πλευρές είναι ίσες και παράλληλες. Σημείωση Έστω 1(ε ) και 2(ε ) δύο παράλληλες ευθείες. Πάνω σ' αυτές θεωρούμε δύο τυχαία, αλλά ίσα τμήματα ΑΒ και ΓΔ, όπως φαίνεται στο διπλανό σχήμα. Τότε το ΑΒΔΓ, δηλαδή το κυρτό τετράπλευρο που σχηματίζεται από τα άκρα των τμημάτων αυτών, είναι παραλληλόγραμμο. Πραγματικά, επειδή τα ΑΒ, ΓΔ είναι ίσα και παράλληλα, το ΑΒΔΓ είναι παραλληλόγραμμο. Η προηγούμενη παρατήρηση βρίσκει συχνή εφαρμογή σε πολλά θέματα με παραλληλόγραμμα.
  • 8. 1.4 Δίνεται τρίγωνο ΑΒΓ, το ισόπλευρο τρίγωνο ΑΒΔ εκτός αυτού, καθώς και το ισοσκελές τρίγωνο ΑΓΕ στο εξωτερικό του Δ ΑΒΓ , με οˆ ˆΕΑΓ = ΕΓΑ = 30 . Αν Μ είναι το μέσο του ΒΓ, να αποδειχθεί ότι οˆΔΜΕ = 90 . Λύση Έστω Ζ το συμμετρικό του Δ ως προς το Μ. Το ΒΔΓΖ είναι παραλληλόγραμμο, διότι οι διαγώνιες διχοτομούνται. Επομένως είναι: ΓΖ ΒΔ ΑΔ  και οˆ ˆ ˆΒΓΖ ΓΒΔ 60 Β   Τα τρίγωνα ΑΔΕ και ΓΕΖ είναι ίσα, διότι:  ΑΔ ΓΖ γ  και ΑΕ ΓΕ  ο ο ο ο ο ο ο ˆˆ ˆ ˆ ˆ ˆ ˆ ˆ ˆΕΓΖ 360 ΕΓΑ ΑΓΒ ΒΓΖ 360 30 Γ (60 Β) 270 (Β Γ) 270 (180 Α)                ο ˆ ˆ90 Α ΔΑΕ   Άρα είναι και ΕΔ ΕΖ και αφού στο ισοσκελές τρίγωνο ΕΔΖ η ΕΜ είναι διάμεσος, είναι και ύψος. Είναι δηλαδή οˆΔΜΕ 90 . 3Α . Είδη Παραλληλογράμμων Ορισμός Ορθογώνιο ονομάζεται το παραλληλόγραμμο που έχει μία (τουλάχιστον) ορθή γωνία. Προφανώς, στο ορθογώνιο όλες οι γωνίες είναι ορθές.
  • 9. Ιδιότητες Στο ορθογώνιο, εκτός από τις ιδιότητες που ισχύουν σε κάθε παραλληλόγραμμο, ισχύουν και οι εξής:  όλες οι γωνίες είναι ορθές,  οι διαγώνιοι είναι ίσες. Κριτήρια Για να αποδείξουμε ότι ένα τετράπλευρο είναι ορθογώνιο, αποδεικνύουμε ότι:  είναι παραλληλόγραμμο και έχει μία ορθή γωνία ή  είναι παραλληλόγραμμο και έχει ίσες διαγωνίους. Τονίζουμε ότι:  Αν σε ένα τετράπλευρο τρεις γωνίες είναι ορθές, τότε αυτό είναι ορθογώνιο.  Αν οι γωνίες ενός τετραπλεύρου είναι ίσες, τότε αυτό είναι ορθογώνιο. Ορισμός Ρόμβος ονομάζεται το παραλληλόγραμμο που έχει δύο διαδοχικές πλευρές ίσες. Προφανώς, στον ρόμβο όλες οι πλευρές είναι ίσες. Ιδιότητες Στον ρόμβο, εκτός από τις ιδιότητες που ισχύουν σε κάθε παραλληλόγραμμο, ισχύουν και οι εξής:  όλες οι πλευρές είναι ίσες,  οι διαγώνιοι τέμνονται κάθετα,  οι διαγώνιοι διχοτομούν τις γωνίες. Κριτήρια Για να αποδείξουμε ότι ένα τετράπλευρο είναι ρόμβος, αποδεικνύουμε ότι:  είναι παραλληλόγραμμο και έχει δύο διαδοχικές πλευρές ίσες ή  είναι παραλληλόγραμμο και οι διαγώνιοί του τέμνονται κάθετα ή  είναι παραλληλόγραμμο και μία διαγώνιός του διχοτομεί μία γωνία του. Τονίζουμε ότι αν όλες οι πλευρές ενός τετραπλεύρου είναι ίσες, τότε είναι ρόμβος.
  • 10. Ορισμός Τετράγωνο ονομάζεται το παραλληλόγραμμο που είναι συγχρόνως και ορθογώνιο και ρόμβος. Προφανώς, στο τετράγωνο όλες οι πλευρές είναι ίσες και όλες οι γωνίες είναι ορθές. Ιδιότητες Το τετράγωνο, εκτός από τις ιδιότητες του τυχαίου παραλληλογράμμου, έχει επιπλέον τις ιδιότητες του ορθογωνίου και του ρόμβου. Οι ιδιότητες λοιπόν του τετραγώνου διατυπώνονται ως εξής:  Οι απέναντι πλευρές είναι ίσες και παράλληλες.  Όλες οι γωνίες του είναι ορθές.  Οι διαγώνιοί του είναι ίσες, διχοτομούνται, τέμνονται κάθετα και διχοτομούν τις γωνίες του. Κριτήρια Για να αποδείξουμε ότι ένα τετράπλευρο είναι τετράγωνο, αποδεικνύουμε ότι είναι παραλληλόγραμμο, έχει μία χαρακτηριστική ιδιότητα του ορθογωνίου (μία γωνία ορθή ή ίσες διαγωνίους) και μία χαρακτηριστική ιδιότητα του ρόμβου (δύο διαδοχικές πλευρές ίσες ή οι διαγώνιοι τέμνονται κάθετα ή μία διαγώνιος διχοτομεί μία γωνία). Είναι προφανές ότι:  Ένα ορθογώνιο με ίσες πλευρές ή κάθετες διαγωνίους είναι τετράγωνο.  Ένας ρόμβος με μια ορθή γωνία ή με ίσες διαγωνίους είναι τετράγωνο. 1.5 Στις πλευρές ΑΒ, ΒΓ ενός τετραγώνου ΑΒΓΔ θεωρούμε αντίστοιχα τα σημεία Ε και Ζ έτσι, ώστε οˆΕΔΖ = 45 . Να αποδειχθεί ότι: α) ο κύκλος με κέντρο Δ και ακτίνα ΑΒ εφάπτεται με την ευθεία ΕΖ, β) ΕΖ = ΑΕ + ΓΖ. Γερμανία – 2009 Λύση
  • 11. α) Θεωρούμε το σημείο Η έτσι, ώστε οˆ ˆΖΔΗ ΕΔΖ 45  . Επειδή oˆΖΔΗ x y 45 x ω     , είναι y ω , δηλαδή: ˆ ˆΑΔΕ ΓΔΗ και Δ Δ ΔΑΕ ΔΓΗ αφού ΔΑ ΔΓ και ˆ ˆΑΔΕ ΓΔΗ . Άρα ΔΕ ΔΗ και ΑΕ ΓΗ . Έτσι τα τρίγωνα ΔΖΕ και ΔΖΗ είναι ίσα, αφού: ΔΖ κοινή, ΔΕ ΔΗ , οˆ ˆΖΔΕ ΖΔΗ 45  Αν λοιπόν ΔΡ ΕΖ , τότε ΔΡ ΔΓ , αφού στα ίσα τρίγωνα ΔΖΕ, ΔΖΗ τα ΔΡ, ΔΓ είναι ομόλογα ύψη. Επομένως, αφού ΔΡ α , ο κύκλος (Δ,α) εφάπτεται με την ΕΖ στο Ρ. β) Επειδή ΖΗ ΖΕ και ΓΗ ΑΕ , έχουμε: ΕΖ ΖΗ ΖΓ ΓΗ ΖΓ ΑΕ     1.6 Στις πλευρές ΒΓ, ΓΔ ενός τετραγώνου ΑΒΓΔ με ΑΒ = α , παίρνουμε τα σημεία Ε, Ζ αντίστοιχα. Αν η περίμετρος του τριγώνου ΓΕΖ είναι ίση με 2α, να αποδειχθεί ότι οˆΕΑΖ = 45 . Λύση Στο ορθογώνιο τρίγωνο ΓΕΖ είναι:  ΓΖ ΖΕ ΕΓ 2α    ΓΖ ΖΕ ΕΓ ΑΔ ΑΒ α 2      Άρα το Α είναι παράκεντρο της γωνίας ˆΓ του Δ ΓΕΖ . Αν λοιπόν ΑΗ ΖΕ , τότε: ο ˆ ˆ ˆΔΑΗ ΗΑΒ ΔΑΒˆ ˆ ˆΖΑΕ ΖΑΗ ΗΑΕ 45 2 2 2       Μπορούμε επίσης απευθείας να πούμε ότι: ο ο ˆΓˆΖΑΕ 90 45 2    με βάση γνωστή ιδιότητα των διχοτόμων των γωνιών τριγώνου.
  • 12. Σχόλιο Αν αΙ είναι το παράκεντρο ορθογωνίου τριγώνου ΑΒΓ οˆ(Α 90 ) , τότε είναι: α ΑΒ ΒΓ ΓΑ ρ ΑΖ τ 2      Αντίστροφα, αν ισχύει η παραπάνω σχέση, τότε το αΙ είναι παράκεντρο του Δ ΑΒΓ . Άλλος τρόπος Στην προέκταση του ΓΒ παίρνουμε τμήμα ΒΗ ΔΖ . Είναι τότε Δ Δ ΑΒΗ ΑΔΖ , οπότε: ΑΗ ΑΖ και οˆΖΑΗ 90 Είναι οπότε:  ΕΖ 2α ΕΓ ΖΓ (α ΕΓ) (α ΖΓ) ΒΕ ΔΖ BE BH            ΕΗ  Δ Δ ΑΕΗ ΑΕΖ , διότι ΑΗ ΑΖ , ΕΗ ΕΖ και ΑΕ κοινή. Άρα παίρνουμε: ο ο ˆΖΑΗ 90ˆΖΑΕ 45 2 2    Άλλος τρόπος Έστω ΓΕ x και ΓZ y . Είναι τότε:  2 2 ΓΕ ΓΖ ΕΖ 2α x y x y 2α         2 2 2 x y [2α (x y)]      2 2 2 2 x y (α x) (α y) 2(α x)(α y)          2 2 2 2 2 2 x y (α 2αx x ) (α 2αy y ) 2(α x)(α y)            2 (α x)(α y) αx αy α      (1)
  • 13.  (1) 2 2 2 2 α x α y 2α x y εφω εφφ α(2α x y)α α αεφ(ω φ) 1 α x α y1 εφω εφφ α (α x)(α y) α αx αy α1 α α α                         . Άρα ο ω φ 45  , οπότε οˆΕΑΖ 45 . 1.7 Δίνεται τετράγωνο ΑΒΓΔ, το μέσο Μ του ΑΔ, το μέσο Ν του ΑΒ και σημείο Ρ στην προέκταση της ΜΓ τέτοιο, ώστε ΓΡ = ΓΜ . Αν η ΡΝ τέμνει τη ΒΓ στο Κ, να αποδειχθεί ότι ΚΒ = ΚΓ . Λύση
  • 14. Έστω ότι η ΜΝ τέμνει τη ΒΓ στο Σ. Επειδή Δ Δ ΝΑΜ ΝΒΣ , είναι ΝΜ ΝΣ . Αλλά ΓΜ ΓΡ , οπότε στο τρίγωνο ΡΜΣ το Κ είναι βαρύκεντρο. Αφού ΓΣ ΓΒ ΒΣ 3α   (έχουμε θέσει ΑΔ ΓΔ 2α  ), είναι: 1 1 ΓΚ ΓΣ 3α α 3 3     Άρα ΒΓ ΓΚ α 2   , οπότε το Κ είναι μέσο του ΒΓ. 1.8 Στη διαγώνιο ΑΓ ενός τετραγώνου ΑΒΓΔ παίρνουμε σημείο Ε έτσι, ώστε οˆΓΒΕ = 30 . Στην προέκταση της διαγωνίου ΑΓ, προς το Γ, παίρνουμε σημείο Ζ, ώστε ΓΖ = ΓΕ . Να αποδειχθεί ότι το τρίγωνο ΖΒΔ είναι ισόπλευρο. Λύση Θεωρούμε το ισόπλευρο τρίγωνο ΒΓΗ. Αν η ευθεία ΔΗ τέμνει την ΑΒ στο σημείο Θ, τότε:  Το σημείο Η βρίσκεται στη μεσοκάθετη του ΒΓ, άρα και του ΑΔ, και έτσι το Η είναι μέσο του ΔΘ, αφού ΗΔ ΗΑ και οˆΑ 90 .  Επειδή ΓΗ ΓΔ και η γωνία ˆΔΓΗ είναι ίση με ο 30 , η γωνία ΑΔΘ είναι ίση με ο ο ο 90 75 15  . Άρα η γωνία ΘΗΒ είναι ίση με ο 45 .  Τα τρίγωνα ΒΓΕ και ΘΒΗ είναι ίσα, διότι ΒΓ ΒΗ και οι προσκείμενες σε αυτές γωνίες είναι ίσες (οι γωνίες αυτές είναι ο 45 και ο 30 αντίστοιχα). Επομένως ΒΕ ΒΘ , ΕΓ ΗΘ και έτσι: ΖΕ 2ΕΓ 2ΗΘ ΔΘ   δηλαδή ΖΕ ΔΘ .  Τα τρίγωνα ΘΒΔ και ΕΒΖ είναι τώρα ίσα, διότι ΘΔ ΕΖ , ΘΒ ΕΒ και οι γωνίες ˆΒΘΔ , ˆΒΕΖ είναι ίσες με ο 105 η καθεμιά. Άρα ΒΔ ΒΖ . Αλλά ΒΖ ΖΔ , οπότε ΒΔ ΒΖ ΖΔ  και η απόδειξη ολοκληρώθηκε. Τη λύση οφείλω στον Ρώσο Μαθηματικό Mikhail Leptchinsky.
  • 15. 4Α . Βαρύκεντρο – Ορθόκεντρο Θεωρήματα α) Το ευθύγραμμο τμήμα που ενώνει τα μέσα δύο πλευρών ενός τριγώνου είναι παράλληλο προς την τρίτη πλευρά και ισούται με το μισό της. Έτσι, αν στο διπλανό σχήμα Μ και Ν είναι τα μέσα των πλευρών ΑΒ και ΑΓ αντίστοιχα, ισχύει ΜΝ // ΒΓ και ΒΓ ΜΝ 2  . β) Αν μια ευθεία διέρχεται από το μέσο μιας πλευράς τριγώνου και είναι παράλληλη προς κάποια πλευρά του, τότε η ευθεία αυτή διέρχεται και από το μέσο της τρίτης πλευράς. Έτσι, αν στο διπλανό σχήμα το Μ είναι μέσο της ΑΒ και ε // ΒΓ, τότε το Ν θα είναι μέσο και της πλευράς ΑΓ. γ) Το τετράπλευρο που έχει κορυφές τα μέσα των πλευρών ενός τετραπλεύρου (κυρτού ή μη κυρτού) είναι παραλληλόγραμμο. Έτσι, με βάση το διπλανό σχήμα, το ΚΛΜΝ είναι παραλληλόγραμμο. Θεώρημα α) Οι διάμεσοι κάθε τριγώνου διέρχονται από το ίδιο σημείο το οποίο λέγεται βαρύκεντρο (ή κέντρο βάρους). β) Το βαρύκεντρο έχει μια πολύ σπουδαία ιδιότητα: χωρίζει την κάθε διάμεσο σε δύο τμήματα από τα οποία το ένα είναι διπλάσιο του άλλου. Έτσι έχουμε:  ΑΘ 2ΘΜ , ΓΘ 2ΘΚ και ΒΘ 2ΘΝ  α 2 ΘΑ μ 3  και α 1 ΘΜ μ 3   β 2 ΘΒ μ 3  και β 1 ΘΝ μ 3   γ 2 ΘΓ μ 3  και γ 1 ΘΚ μ 3 
  • 16. Τονίζουμε ότι αν Θ είναι σημείο της διαμέσου ΑΜ ενός τριγώνου ΑΒΓ και ΘΑ 2ΘΜ , τότε το Θ είναι το βαρύκεντρο του τριγώνου αυτού. Θεώρημα α) Οι φορείς των υψών κάθε τριγώνου διέρχονται από το ίδιο σημείο το οποίο λέγεται ορθόκεντρο. β) Για το ορθόκεντρο σημειώνουμε τα εξής:  Αν το τρίγωνο είναι οξυγώνιο, τότε το ορθόκεντρο είναι εσωτερικό σημείο του τριγώνου.  Αν το τρίγωνο είναι ορθογώνιο, τότε το ορθόκεντρο συμπίπτει με την κορυφή της ορθής γωνίας.  Αν το τρίγωνο είναι αμβλυγώνιο, τότε το ορθόκεντρο είναι εξωτερικό σημείο του τριγώνου. 1.9 Δίνεται τρίγωνο ΑΒΓ, με οˆΑ < 90 , και εκτός αυτού τα ισοσκελή τρίγωνα ΑΒΕ, ΑΓΖ, με βάσεις ΑΒ, ΑΓ αντίστοιχα. Αν Δ είναι το μέσο του ΒΓ και οˆΕΔΖ = 90 , να αποδειχθεί ότι οι γωνίες ˆΑΕΒ και ˆΑΖΓ είναι παραπληρωματικές. (JBMO – 2008, short list) Λύση Θεωρούμε το συμμετρικό Η του Ζ ως προς το μέσο Δ του ΒΓ. Το ΒΗΓΖ είναι παραλληλόγραμμο, διότι οι διαγώνιες διχοτομούνται. Επομένως: ΒΗ ΖΓ ΖΑ  Στο τρίγωνο ΕΗΖ η ΕΔ είναι διάμεσος και ύψος, οπότε αυτό είναι ισοσκελές. Άρα ΕΗ ΕΖ και επειδή ΒΕ ΕΑ , είναι Δ Δ ΑΕΖ ΒΕΗ . Είναι λοιπόν: ˆˆΕΒΗ ΕΑΖ  ο ˆˆ ˆ360 φ Β ΓΒΗ φ Α ω        ο ˆˆ ˆ360 φ Β ΒΓΖ φ Α ω        ο οˆ ˆˆ ˆ ˆ ˆ2φ ω (Α Β) (Γ ω) 360 2φ 2ω (Α Β Γ) 360              ο ο ο ο ˆ ˆ180 ΑΕΒ 180 ΑΖΓ φ ω 90 90 2 2          οˆ ˆΑΕΒ ΑΖΓ 180  
  • 17. 1.10 Δίνεται ισοσκελές τρίγωνο ΑΒΓ (ΑΒ = ΑΓ) και δύο εσωτερικά σημεία Δ, Ε της πλευράς ΑΓ τέτοια, ώστε ΔΒ = ΔΕ και ˆ ˆΓΒΔ = ΑΒΕ . Αν Ο είναι το έγκεντρο του τριγώνου ΕΒΓ, να αποδειχθεί ότι οˆΓΟΕ =120 . (JBMO – 2006) Λύση Έστω ˆΔΕΒ 2ω . Επειδή ΔΕ ΔΒ , είναι ˆΔΒΕ 2ω . Αλλά: ο ο ˆΕΒΓ 2ω xˆΕΟΓ 90 90 2 2      (1) Στο τρίγωνο ΒΕΑ είναι: ˆ ˆˆ ˆBEΓ ΒΑΕ ΕΒΑ 2ω Α x      ο ˆ2ω (180 2Β) x     o 2ω 180 2(2x 2ω) x      ο o 6ω 180 3x 3x 6ω 180       ο x 2ω 60   (2) Έτσι η (1) δίνει: ο(2) ο ο ο ο ˆΕΒΓ x 2ω 60ˆΕΟΓ 90 90 90 120 2 2 2         Θεωρήματα α) Η διάμεσος ορθογωνίου τριγώνου που αντιστοιχεί στην υποτείνουσα είναι ίση με το μισό της υποτείνουσας. Έτσι, με βάση το διπλανό σχήμα, ισχύει ότι: ΒΓ ΑΜ ΜΒ ΜΓ 2    β) Αν η διάμεσος ενός τριγώνου ισούται με το μισό της αντίστοιχης πλευράς, τότε το τρίγωνο αυτό είναι ορθογώνιο με υποτείνουσα την πλευρά αυτή. Έτσι, αν για το τρίγωνο ΑΒΓ του διπλανού σχήματος ισχύει:
  • 18. ΑΜ ΜΒ ΜΓ  τότε οˆΑ 90 . Θεωρήματα α) Αν σε ένα ορθογώνιο τρίγωνο μια γωνία του είναι ίση με ο 30 , τότε η απέναντι πλευρά του είναι ίση με το μισό της υποτείνουσας. β) Αν σε ένα ορθογώνιο τρίγωνο μία κάθετη πλευρά του είναι ίση με το μισό της υποτείνουσας, τότε η γωνία που βρίσκεται απέναντι από την πλευρά αυτή είναι ίση με ο 30 . Πρόταση Αν Μ είναι σημείο στην υποτείνουσα ΒΓ ενός ορθογωνίου τριγώνου ΑΒΓ, ώστε ΜΑ ΜΓ , τότε το Μ είναι μέσο του ΒΓ. Απόδειξη Αφού ΜΑ ΜΓ , είναι ˆ ˆΜΑΓ ΜΓΑ φ  . Άρα:  ο οˆ ˆΒ 90 Γ 90 φ     οˆ ˆ ˆΜΑΒ Α ΜΑΓ 90 φ    Άρα οˆˆΒ ΜΑΒ 90 φ   , οπότε το τρίγωνο ΜΑΒ είναι ισοσκελές. Άρα: ΜΒ ΜΑ ΜΓ  1.11 Δίνεται τετράγωνο ΑΒΓΔ και Ρ εσωτερικό σημείο αυτού, ώστε οˆˆΡΓΔ = ΡΔΓ =15 . Να αποδειχθεί ότι το τρίγωνο ΡΑΒ είναι ισόπλευρο. Λύση Έστω ΑΜ, ΓΝ ΡΔ . Είναι Δ Δ ΑΔΜ ΓΔΝ , οπότε: ΓΝ ΔΜ Αλλά οˆΓΡΝ 30 , οπότε: ΡΓ ΡΔ ΓΝ ΔΜ 2 2    Το Μ είναι λοιπόν μέσο του ΡΔ, οπότε ΑΔ ΑΡ . Έτσι:
  • 19. ΡΒ ΡΑ ΑΔ ΑΒ   1.12 Δίνεται ορθογώνιο τρίγωνο ΑΒΓ οˆ(Α = 90 ), το ύψος ΑΗ, η διχοτόμος ΒΜ της γωνίας ˆAΒΓ , που τέμνει την ΑΗ στο Ρ, και η διχοτόμος ΓΝ της ˆΑΓΒ , που τέμνει την ΑΗ στο Ζ. Αν Κ, Λ είναι τα μέσα των ΡΜ, ΖΝ, να αποδειχθεί ότι ΚΛ // ΒΓ. (Κίνα – 2005) Λύση Προεκτείνουμε τις ΑΛ, ΑΚ μέχρι να συναντήσουν τη ΒΓ. Τα τρίγωνα Δ ΑΡΜ και Δ ΑΖΝ είναι ισοσκελή, διότι: ˆΒˆˆ ˆ ˆ ˆΑΡΜ ΡΑΒ ΡΒΑ Γ ΑΜΡ 2      και: ˆΓˆˆ ˆ ˆ ˆΑΖΝ ΑΓΡ ΗΑΓ Β ΑΝΖ 2      Άρα ΑΛ ΓΝ και ΑΚ ΒΜ . Αλλά τότε τα Λ, Κ είναι μέσα των ΑΣ, ΑΤ, οπότε ΚΛ // ΣΤ ΚΛ // ΒΓ. 5Α . Τραπέζια Ορισμός α) Τραπέζιο λέγεται το κυρτό τετράπλευρο που έχει μόνο δύο πλευρές παράλληλες. Οι παράλληλες πλευρές λέγονται βάσεις του τραπεζίου. β) Το ευθύγραμμο τμήμα που ενώνει τα μέσα των μη παράλληλων πλευρών του τραπεζίου λέγεται διάμεσος του τραπεζίου. γ) Η απόσταση των παράλληλων πλευρών του τραπεζίου λέγεται ύψος του τραπεζίου. Θεωρήματα α) Η διάμεσος του τραπεζίου είναι παράλληλη προς τις βάσεις του και ισούται με το ημιάθροισμά τους. Ισχύει δηλαδή ότι:
  • 20.  MN // AB // ΓΔ  AB ΓΔ MN 2   β) i) Το ευθύγραμμο τμήμα που ενώνει τα μέσα των διαγωνίων ενός τραπεζίου είναι παράλληλο προς τις βάσεις του και ισούται με την ημιδιαφορά τους. Ισχύει δηλαδή ότι:  ΚΛ // AB // ΓΔ  ΓΔ AB ΚΛ 2   ii) Η διάμεσος του τραπεζίου διέρχεται από τα μέσα των διαγωνίων του. Ορισμός Το τραπέζιο που έχει τις μη παράλληλες πλευρές ίσες, λέγεται ισοσκελές. Θεώρημα Στο ισοσκελές τραπέζιο ισχύουν οι εξής ιδιότητες:  οι γωνίες που πρόσκεινται στις βάσεις είναι ίσες, δηλαδή ˆˆΓ Δ και ˆ ˆA B ,  οι διαγώνιες είναι ίσες, δηλαδή ΑΓ ΒΔ . Κριτήρια Για να είναι ένα τραπέζιο ισοσκελές, αρκεί να αποδείξουμε ότι:  οι μη παράλληλες πλευρές είναι ίσες,  οι γωνίες μιας βάσης είναι ίσες,  οι διαγώνιοι είναι ίσες. Μια χρήσιμη βοηθητική γραμμή Ας υποθέσουμε ότι λύνοντας μια άσκηση εντοπίζουμε στο σχήμα ένα τραπέζιο ΑΒΓΔ, με ΑΒ // ΓΔ, του οποίου μάλιστα υπάρχει και το μέσο Μ μιας μη παράλληλης πλευράς (εδώ της ΑΔ). Αν με τα υπάρχοντα στοιχεία δεν μπορούμε να επιτύχουμε λύση, τότε κάνουμε μία από τις επόμενες ενέργειες: i) Θεωρούμε το μέσο Ν και της ΒΓ, οπότε ισχύουν:  ΜΝ // ΑΒ και ΜΝ // ΓΔ  ΑΒ + ΓΔ ΜΝ = 2 ii) Φέρνουμε ΜΝ // ΑΒ (ή ΜΝ // ΓΔ), οπότε το ΜΝ θα είναι διάμεσος του τραπεζίου και θα ισχύουν προφανώς οι προηγούμενες σχέσεις.
  • 21. iii) Θεωρούμε το συμμετρικό Σ του Β ως προς το Μ, οπότε το ΑΒΔΣ είναι παραλληλόγραμμο και τα σημεία Σ, Δ, Γ είναι συνευθειακά. 1.13 Δίνεται ορθογώνιο τρίγωνο ΑΒΓ, το μέσο Μ της ΒΓ και τα σημεία Δ, Ε των πλευρών ΑΒ, ΑΓ αντίστοιχα έτσι, ώστε οˆΔΜΕ = 90 . Από το Γ φέρουμε τμήμα ΓΝ ίσο και παράλληλο με το ΔΕ. Να αποδειχθεί ότι ΝΜ ΒΓ . Λύση Έστω Κ το μέσο του ΔΕ και Λ το μέσο του ΝΓ. Επειδή το ΔΕΓΝ είναι παραλληλόγραμμο, το ΚΛΓΕ είναι επίσης παραλληλόγραμμο. Έτσι ΚΛ // ΑΓ. Έχουμε λοιπόν: ΔΕ ΝΓ ΑΚ ΛΓ 2 2    Επομένως το τραπέζιο ΚΑΓΛ είναι ισοσκελές. Είναι όμως ΜΑ ΜΓ , οπότε το Μ βρίσκεται στη μεσοκάθετο του ΑΓ, συνεπώς και στη μεσοκάθετο του ΚΛ (αφού το ΚΛΓΑ είναι ισοσκελές τραπέζιο). Είναι λοιπόν ΜΛ ΜΚ , οπότε: ΔΕ ΝΓ ΜΛ ΜΚ 2 2    Αφού στο τρίγωνο ΜΝΓ η ΜΛ είναι διάμεσος και ΝΓ ΜΛ 2  , το τρίγωνο αυτό είναι ορθογώνιο. Άρα ΝΜ ΒΓ . 1.14 Δίνεται ισοσκελές τρίγωνο ΑΒΓ (ΑΒ = ΑΓ) και σημείο Δ της διαμέσου ΑΜ. Σχηματίζουμε το παραλληλόγραμμο ΑΒΔΕ. Να αποδειχθεί ότι ΕΓ ΒΓ . (GM – 1997) Λύση Έστω Ζ η τέταρτη κορυφή του παραλληλογράμμου ΑΒΜΖ. Επειδή: ΔΕ ΑΒ ΜΖ  και ΔΕ // ΑΒ // ΜΖ το ΔΕΖΜ είναι παραλληλόγραμμο. Άρα: ΕΖ // ΔΜ και επειδή ΔΜ ΒΓ , θα είναι και ΕΖ ΒΓ . Επίσης είναι: ΑΖ / / ΒΜ / / ΜΓ  οπότε και το τετράπλευρο ΑΖΓΜ είναι παραλληλόγραμμο. Συνεπώς:
  • 22. ΖΓ ΒΓ Τα σημεία λοιπόν Ε, Ζ, Γ είναι συνευθειακά (αφού ΕΖ, ΖΓ // ΑΜ) και έτσι: ΕΓ ΒΓ 1.15 Σε ένα ορθογώνιο τρίγωνο ΑΒΓ οˆ(Α = 90 ) είναι ΑΒ > ΑΓ . Στην πλευρά ΑΒ παίρνουμε σημείο Μ, ώστε ΒΜ = ΑΓ . Στην πλευρά ΑΓ παίρνουμε σημείο Ν, ώστε ΓΝ = ΑΜ . Να αποδειχθεί ότι η γωνία που σχηματίζουν οι ευθείες ΒΝ, ΓΜ είναι ίση με ο 45 . (Βουλγαρία) Λύση Θεωρούμε το ορθογώνιο ΓΑΒΔ. Φέρνουμε ΒΕ // ΓΜ. Έστω ΒΜ α και ΜΑ β . Είναι:  ΓΕ ΜΒ ΑΓ ΒΔ α     ΔΕ ΓΔ ΓΕ ΑΒ α ΑΜ ΓΝ β        Τα ορθογώνια τρίγωνα ΓΕΝ και ΔΕΒ είναι ίσα, οπότε: ΕΒ ΕΝ και ˆ ˆΓΕΝ ΔΒΕ φ  Επομένως: οˆ ˆ ˆ ˆΓΕΝ ΔΕΒ ΔΒΕ ΔΕΒ 90    Συνεπώς το τρίγωνο ΕΒΝ είναι ορθογώνιο και ισοσκελές, οπότε: οˆ ˆΕΝΒ ΕΒΝ 45  Άρα οˆ ˆΜΙΒ ΙΒΕ 45  , διότι ΙΜ // ΕΒ. 1.16 Σε ένα τρίγωνο ΑΒΓ, με οˆΑ = 60 , φέρουμε τις διχοτόμους ΑΔ και ΒΕ. Αν ισχύει ΑΒ + ΒΔ = ΑΕ + ΕΒ , να αποδειχθεί ότι οˆΓ = 40 . (ΙΜΟ – 2001) Λύση Στις προεκτάσεις των ΑΒ, ΑΓ παίρνουμε τμήματα ΒΗ ΒΔ και ΕΖ ΕΒ . Επειδή ΒΗ ΒΔ , είναι:
  • 23. ˆΑΒΔˆΒΗΔ φ 2   Είναι επίσης: ΑΒ ΒΔ ΑΕ ΕΒ    ΑΒ ΒΗ ΑΕ ΕΖ     ΑΗ ΑΖ  Επομένως το τρίγωνο ΑΗΖ είναι ισόπλευρο και επειδή η ΑΔ είναι διχοτόμος της γωνίας ˆΑ , είναι: ΔΗ ΔΖ και ˆ ˆΔΖΑ ΔΗΑ φ  Θα αποδείξουμε ότι το Ζ ταυτίζεται με το Γ. Αν το Ζ δεν συμπίπτει με το Γ, τότε στο τρίγωνο ΔΒΖ είναι: ˆ ˆ ˆ ˆ ˆ ˆ ˆ ˆΔΒΖ ΕΒΖ ΕΒΔ ΕΖΒ ΒΗΔ ΕΖΒ ΔΖΑ ΔΖΒ       οπότε ΔΒ ΔΖ , άρα και το τρίγωνο ΔΒΖ είναι ισοσκελές. Είναι λοιπόν ΔΗ ΔΖ ΔΒ  και επειδή ΒΗ ΒΔ , το τρίγωνο ΒΔΗ είναι ισόπλευρο. Άρα: ο ο οˆˆ ˆΑΒΔ ΒΗΔ ΒΔΗ 60 60 120     άτοπο, διότι οˆΑ 60 . Άρα το Ζ συμπίπτει με το Γ (ανάλογα εργαζόμαστε αν το Ζ είναι ανάμεσα στα Ε, Γ). Επειδή λοιπόν τα Γ, Ζ συμπίπτουν, είναι:  ˆΒ ˆΕΖ ΕΒ ΕΓ ΕΒ Γ 2       ο οˆˆ ˆ ˆ ˆΒ Γ Α 180 Β Γ 120      Έχουμε επομένως: ο ο οˆ ˆ ˆ ˆ ˆΒ Γ 120 2Γ Γ 120 Γ 40      
  • 24. B. Κύκλος και εγγράψιμα τετράπλευρα 1Α . Εγγεγραμμένες Γωνίες Ορισμός Μια γωνία που έχει την κορυφή της πάνω σε έναν κύκλο και οι πλευρές της γωνίας τέμνουν τον κύκλο, λέγεται εγγεγραμμένη γωνία. Έτσι, στο διπλανό σχήμα έχουμε τα εξής:  Η γωνία ˆΣ είναι εγγεγραμμένη και βαίνει στο τόξο ΑΒ .  Η γωνία ˆΑΟΒ είναι η αντίστοιχη επίκεντρη της γωνίας ˆΣ . Θεωρήματα α) Κάθε εγγεγραμμένη γωνία είναι ίση με το μισό της αντίστοιχης επίκεντρης γωνίας. Έτσι στο παραπάνω σχήμα έχουμε:  1 ˆˆΣ ΑΟΒ 2  ή ˆ ˆΑΟΒ 2Σ ,  1ˆΣ ΑΒ 2  , δηλαδή η εγγεγραμμένη γωνία ισούται με το μισό του αντίστοιχου τόξου. β) Η εγγεγραμμένη γωνία που βαίνει σε ημικύκλιο (βλέπει διάμετρο) είναι ορθή. Έτσι στο διπλανό σχήμα είναι: οˆ ˆΜ Σ 90  γ) Οι εγγεγραμμένες γωνίες του ίδιου ή ίσων κύκλων που βαίνουν στο ίδιο ή σε ίσα τόξα είναι ίσες. Έτσι στο διπλανό σχήμα είναι: ˆ ˆ ˆ ˆΜ Ν Σ P   Θεώρημα
  • 25. Η γωνία που σχηματίζεται από μια χορδή και την εφαπτομένη σε ένα άκρο της ισούται με κάθε εγγεγραμμένη γωνία που βαίνει στο (κυρτό) τόξο της χορδής αυτής. Έτσι στο διπλανό σχήμα ισχύουν οι σχέσεις:  ˆ ˆM ABΔ φ  ,  ˆ ˆΝ ΑΒΔ φ  ,  ˆ ˆ ˆΜ Ν ΑΒΔ φ   . Θεώρημα Όταν δύο χορδές ενός κύκλου ή οι προεκτάσεις τους τέμνονται σε ένα σημείο, τότε ισχύουν:   ΑΓ ΒΔ φ 2   ,   ΑΓ ΒΔ ω 2   . Σημείωση Ο γεωμετρικός τόπος των σημείων του επιπέδου από τα οποία ένα τμήμα ΑΒ φαίνεται υπό γωνία φ είναι δύο τόξα κύκλων, χορδής ΑΒ, χωρίς τα άκρα τους, συμμετρικά ως προς την ευθεία ΑΒ. Τα τόξα αυτά είναι:  μείζονα (μη κυρτά), αν η φ είναι οξεία,  ελάσσονα (κυρτά), αν η φ είναι αμβλεία. 1.1 Δίνεται ισόπλευρο τρίγωνο ΑΒΓ και τα σημεία Δ, Ε των ΑΒ, ΑΓ αντίστοιχα, ώστε ΔΕ // ΒΓ. Αν Μ είναι το μέσο του τμήματος ΒΕ και Κ το περίκεντρο του τριγώνου ΑΔΕ, να αποδειχθεί ότι οˆΚΜΓ = 90 . (Ρωσία) Λύση
  • 26. Θεωρούμε το παραλληλόγραμμο ΕΓΒΖ. Τα Ζ, Δ, Ε βρίσκονται στην ίδια ευθεία και το τρίγωνο ΖΔΒ είναι ισόπλευρο, διότι: ο ˆˆΖΒΔ 60 ΖΔΒ  Τα τρίγωνα ΚΔΖ και ΚΕΓ είναι ίσα, διότι:  ΚΔ ΚΕ , αφού το Κ είναι περίκεντρο,  ΖΔ ΕΓ , διότι ΖΔ ZΒ ΕΓ  ,  οˆ ˆΚΔΖ ΚΕΓ 150  , διότι οˆ ˆΚΔΕ ΚΕΑ 30  . Είναι επομένως ΚΖ ΚΓ και αφού το Μ είναι μέσο του ΓΖ, είναι ΚΜ ΓΖ . Άρα οˆΚΜΓ 90 . 2Α . Εγγεγραμμένα – Εγγράψιμα Τετράπλευρα Ορισμός Ένα τετράπλευρο λέγεται εγγεγραμμένο σε κύκλο, αν οι κορυφές του είναι σημεία ενός κύκλου. Ο κύκλος αυτός λέγεται περιγεγραμμένος κύκλος του τετραπλεύρου. Θεώρημα Σε κάθε εγγεγραμμένο τετράπλευρο ισχύουν οι παρακάτω ιδιότητες: α) Οι απέναντι γωνίες είναι παραπληρωματικές. Έτσι: οˆ ˆΑ Γ 180  , οˆˆΒ Δ 180  β) Κάθε πλευρά του φαίνεται από τις απέναντι κορυφές υπό ίσες γωνίες, δηλαδή: ˆ ˆΔΑΓ ΔΒΓ γ) Κάθε εσωτερική γωνία ισούται με την απέναντι εξωτερική. Έτσι εξ ˆ ˆΑ Γ , εξ ˆˆΒ Δ . Ορισμός Ένα τετράπλευρο λέγεται εγγράψιμο όταν μπορεί να γραφεί κύκλος που να διέρχεται από τις τέσσερις κορυφές του. Κριτήρια
  • 27. εγγραψιμότητας Ένα τετράπλευρο είναι εγγράψιμο σε κύκλο, αν ισχύει μία από τις παρακάτω προτάσεις: α) Δύο απέναντι γωνίες του είναι παραπληρωματικές. Έτσι, αν οˆ ˆΑ Γ 180  , τότε τα σημεία Α, Β, Γ και Δ είναι ομοκυκλικά (το ΑΒΓΔ είναι εγγράψιμο). β) Μία πλευρά φαίνεται από τις απέναντι κορυφές υπό ίσες γωνίες. Έτσι, αν ˆ ˆΔΑΓ ΔΒΓ , τότε το ΑΒΓΔ είναι εγγράψιμο. γ) Μια εξωτερική του γωνία είναι ίση με την απέναντι εσωτερική. Έτσι, αν εξ ˆ ˆΑ Γ , τότε το ΑΒΓΔ είναι εγγράψιμο. Ορισμός Ένα τετράπλευρο του οποίου οι πλευρές εφάπτονται στον ίδιο κύκλο λέγεται περιγεγραμμένο στον κύκλο αυτό, ενώ ο κύκλος λέγεται εγγεγραμμένος. Θεώρημα Σε κάθε περιγεγραμμένο τετράπλευρο ισχύουν οι εξής ιδιότητες:  Οι διχοτόμοι των γωνιών του διέρχονται από το ίδιο σημείο, που είναι το κέντρο του εγγεγραμμένου κύκλου.  Τα αθροίσματα των απέναντι πλευρών είναι ίσα, δηλαδή: ΑΒ ΓΔ ΑΔ ΒΓ   Ορισμός – Κριτήρια Αν ένα τετράπλευρο μπορεί να περιγραφεί σε κύκλο, τότε λέγεται περιγράψιμο σε κύκλο. Για να είναι ένα τετράπλευρο περιγράψιμο, αρκεί να ισχύει μία από τις ακόλουθες προτάσεις:  Οι διχοτόμοι των γωνιών του διέρχονται από το ίδιο σημείο.  Τα αθροίσματα των απέναντι πλευρών του είναι ίσα. 1.2 Θεωρούμε ορθογώνιο και ισοσκελές τρίγωνο ΑΒΓ, με οˆΑ = 90 , και το ύψος του ΑΔ. Η διχοτόμος ΒΕ του τριγώνου ΑΒΓ τέμνει την ΑΔ στο Κ. Αν Ι είναι το έγκεντρο του τριγώνου ΑΔΓ, να αποδειχθεί ότι ΙΚ = ΙΕ . Λύση
  • 28. Το Κ είναι έγκεντρο του τριγώνου ΑΒΓ, οπότε το Ι βρίσκεται πάνω στην ΚΓ. Αν ΚΛ ΑΓ , τότε: ΚΔ ΚΛ και Δ Δ ΚΔΙ ΚΛΙ αφού ˆ ˆΔΚΓ ΛΚΓ . Επομένως: οˆ ˆΚΛΙ ΚΔΙ 45  δηλαδή οˆ ˆΙΛΕ 45 ΙΚΕ  . Άρα το ΙΚΛΕ είναι εγγράψιμο και έτσι: οˆˆ ˆΙΕΚ ΙΛΚ 45 ΙΚΕ   Άρα το τρίγωνο ΙΚΕ είναι ισοσκελές, οπότε ΙΚ ΙΕ . Σχόλιο Ας παρατηρήσουμε ακόμα ότι οˆˆΚΙΕ = ΚΛΑ = 90 , δηλαδή ΙΚ ΙΕ και οˆΙΕΒ = 45 . Άλλος τρόπος Αν η ΓΚ τέμνει την ΑΒ στο Ζ, τότε το ΒΖΕΓ είναι ισοσκελές τραπέζιο. Έτσι: ο 45ˆ ˆ ˆΕΑΙ ΖΓΒ ΕΖΙ φ 2     Άρα το ΑΖΙΕ είναι εγγράψιμο, οπότε: οˆˆΕΙΓ Α 90  Όμως: οˆ ˆ ˆΕΚΙ ΚΒΓ ΚΓΒ 45   οπότε το ορθογώνιο τρίγωνο ΙΚΕ είναι και ισοσκελές. Επομένως ΙΚ ΙΕ και οˆΙΕΚ 45 . 1.3 Δίνεται τρίγωνο ΑΒΓ, το ύψος ΑΔ και η διάμεσος ΑΜ. Αν ˆ ˆΒΑΔ = ΜΑΓ , να αποδειχθεί ότι το τρίγωνο ΑΒΓ είναι ορθογώνιο. Λύση Έστω Ε, Ζ τα συμμετρικά του Α ως προς τα σημεία Δ, Μ αντίστοιχα. Τότε:
  • 29. ˆ ˆΒΑΔ ΒΕΑ και ˆ ˆΜΑΓ ΑΖΒ διότι το τετράπλευρο ΑΒΖΓ είναι παραλληλόγραμμο, μια και οι διαγώνιες διχοτομούνται. Επειδή από την υπόθεση είναι ˆ ˆΒΑΔ ΜΑΓ , θα είναι και ˆ ˆΒΕΑ ΑΖΒ . Αυτό εξασφαλίζει ότι το τετράπλευρο ΑΒΕΖ είναι εγγράψιμο, οπότε: ˆ ˆΑΒΖ ΑΕΖ Στο τρίγωνο ΑΕΖ το τμήμα ΔΜ ενώνει μέσα, οπότε ΔΜ // ΕΖ. Επειδή ΜΔ ΑΕ , θα είναι και ΕΖ ΑΕ . Άρα οˆΑΕΖ 90 , δηλαδή οˆΑΒΖ 90 . Το παραλληλόγραμμο λοιπόν ΑΒΖΓ είναι ορθογώνιο, οπότε οˆΑ 90 . 1.4 Στο εσωτερικό ενός παραλληλογράμμου ΑΒΓΔ υπάρχει σημείο Κ τέτοιο, ώστε οˆΑΚΔ =120 και οˆΒΚΓ = 60 . Να αποδειχθεί ότι οι περιγεγραμμένοι κύκλοι των τριγώνων ΚΑΔ και ΚΒΓ είναι ίσοι. Λύση Θεωρούμε το σημείο Ε, ώστε ΔΕ // ΑΚ και ΔΕ ΑΚ , όπως στο σχήμα. Είναι τότε:  ΑΔ ΚΕ και Δ Δ ΚΑΔ ΚΔΕ  ΚΕ ΑΔ ΒΓ  και Δ Δ ΚΒΓ ΚΓΕ , διότι το τετράπλευρο ΚΕΓΒ είναι παραλληλόγραμμο.  οˆ ˆΚΔΕ ΔΚΑ 120  και οˆ ˆΚΓΕ ΓΚΒ 60  Το τετράπλευρο ΚΔΕΓ είναι εγγράψιμο, διότι: ο ο οˆ ˆΚΔΕ ΚΓΕ 120 60 180    Αφού λοιπόν τα τρίγωνα ΚΔΕ και ΚΕΓ έχουν τον ίδιο περιγεγραμμένο κύκλο, συμπεραίνουμε ότι τα τρίγωνα ΚΑΔ, ΚΒΓ έχουν ίσους περιγεγραμμένους κύκλους. Σχόλιο Επειδή Δ Δ ΚΑΒ = ΕΔΓ , προκύπτει ότι και τα τρίγωνα ΚΑΒ, ΚΓΔ έχουν ίσους περιγεγραμμένους κύκλους με τα τρίγωνα ΚΑΔ, ΚΒΓ. 1.5 Στο εσωτερικό ενός τριγώνου ΑΒΓ υπάρχει σημείο Δ, ώστε οˆ ˆΔΑΓ = ΔΓΑ = 30 και οˆΔΒΑ = 60 . Στο τμήμα ΑΓ παίρνουμε σημείο Ζ τέτοιο, ώστε ΑΖ = 2ΖΓ . Αν Ε είναι το μέσο του ΒΓ, να
  • 30. αποδειχθεί ότι ΔΕ ΕΖ . (Κίνα – 2007, Girls Olympiad) Λύση Έστω ΒΗ η διχοτόμος της γωνίας ˆΑΒΔ . Επειδή οˆ ˆΔΑΗ 30 ΔΒΗ  , το τετράπλευρο ΑΒΔΗ είναι εγγράψιμο. Επομένως:  οˆ ˆˆΗΔΑ ΗΒΑ ΗΑΔ 30   , οπότε ΗΔ ΗΑ  οˆ ˆΔΗΖ ΑΒΔ 60  , οπότε το τρίγωνο ΗΔΓ είναι ορθογώνιο  2ΑΗ 2ΔΗ ΗΓ  και επειδή ΑΖ 2ΖΓ , είναι ΑΗ ΗΖ ΖΓ  Πραγματικά, έχουμε: ΗΓ 2ΑΗ , ΑΓ 3ΑΗ , ΑΓ 3ΖΓ οπότε ΑΓ ΑΗ ΖΓ 3   και έτσι: ΑΓ ΗΖ ΑΗ ΖΓ 3    . Αν Θ είναι το μέσο του ΑΓ, τότε ΕΘ // ΑΒ και ΕΖ // ΒΗ. Άρα: ο ˆˆ ˆΘΕΖ ΑΒΗ 30 ΘΔΖ   Επομένως το ΔΘΖΕ είναι εγγράψιμο και αφού οˆΔΘΖ 90 , είναι οˆΔΕΖ 90 . Τονίζουμε ότι δεν γνωρίζουμε εξαρχής ότι τα Β, Δ, Ζ είναι συνευθειακά, ούτε χρησιμοποιούμε πουθενά στη λύση αυτόν τον ισχυρισμό. Τελικά όμως, επειδή ΗΖ ΖΓ ΗΔ  και οˆΗΔΖ 60 , τα σημεία Β, Δ, Ζ είναι όντως συνευθειακά. Σημείωση Δύο άλλες λύσεις υπάρχουν στο βιβλίο Mathematical Olympiad in China, 2007 – 2008, σελ. 116. 3Α . Θεωρήματα και Εφαρμογές στον Κύκλο Θεώρημα 1ο Αν η διχοτόμος της γωνίας ˆΑ τριγώνου ΑΒΓ και η μεσοκάθετη της πλευράς ΒΓ τέμνονται στο σημείο Ε, τότε το Ε είναι σημείο του περιγεγραμμένου κύκλου του τριγώνου ΑΒΓ. (Θεώρημα του Νότιου Πόλου)
  • 31. Απόδειξη Έστω ότι η μεσοκάθετος της ΒΓ τέμνει τον περιγεγραμμένο κύκλο C του τριγώνου ΑΒΓ στο Ε. Τότε το Ε είναι μέσο του τόξου ΒΓ , οπότε η ΑΕ είναι διχοτόμος της γωνίας ˆΑ του Δ ΑΒΓ . Επειδή το Ε είναι το μοναδικό μέσο του τόξου ΒΓ , η απόδειξη έχει ολοκληρωθεί. 1.6 Σε ένα μη ισοσκελές τρίγωνο ΑΒΓ φέρουμε τις διχοτόμους ΑΛ και ΒΚ. Η μεσοκάθετος του ΒΚ τέμνει την ΑΛ στο σημείο Μ. Η παράλληλη από το Λ προς τη ΜΚ τέμνει τη ΒΚ στο σημείο Ν. Να αποδειχθεί ότι NA = NΛ . (JBMO – 2010) Λύση Στο τρίγωνο ΑΒΚ η ΑΜ είναι διχοτόμος της γωνίας ˆA . Επειδή η ΜΡ είναι μεσοκάθετος του ΒΚ, σύμφωνα με το θεώρημα του Νότιου Πόλου, το Μ βρίσκεται στον περιγεγραμμένο κύκλο του τριγώνου Δ ΑΒΚ . Το τετράπλευρο λοιπόν ΑΒΜΚ είναι εγγράψιμο, οπότε: ˆ ˆ ˆ ˆΑΛN AΜΚ ΑΒΚ ΑΒN   Επειδή ˆ ˆΑΛN ΑΒN , το τετράπλευρο ΑΒΛΝ είναι επίσης εγγράψιμο και επειδή η ΒΝ είναι διχοτόμος της γωνίας ˆΒ , θα είναι  ΝΑ ΝΛ (στον κύκλο (Α, Β, Λ, Ν)). Άρα NA NΛ . Μπορούμε βέβαια, να εργαστούμε και ως εξής: ˆ ˆ ˆˆ ˆΛΑΝ ΛΒΝ ΝΒΑ ΝΛΑ ΑΛΝ    Άρα ˆ ˆΛΑN ΑΛN , οπότε το τρίγωνο ΝΑΛ είναι ισοσκελές. Επομένως NA NΛ . Θεώρημα 2ο Έστω Ι το έγκεντρο ενός τριγώνου ΑΒΓ. Αν η ευθεία ΑΙ τέμνει τον περιγεγραμμένο κύκλο του τριγώνου ΑΒΓ στο σημείο Μ, τότε ΜΒ = ΜΙ = ΜΓ . Απόδειξη
  • 32. Η ΑΙ είναι διχοτόμος της γωνίας ˆΑ του τριγώνου ΑΒΓ, οπότε το Μ είναι το μέσο του τόξου ΒΓ . Επομένως ΜΒ ΜΓ . Θα αποδείξουμε λοιπόν ότι ΜΒ ΜΙ . Είναι:  ˆΑˆ ˆΙΑΒ ΙΑΓ φ 2    ,  ˆΒˆ ˆΙΒΑ ΙΒΓ ω 2    ,  ˆˆ ˆΒΙΜ ΙΑΒ ΙΒΑ φ ω    , ˆ ˆ ˆΙΒΜ ΙΒΓ ΓΒΜ ω φ    . Είναι λοιπόν ˆ ˆΒΙΜ ΙΒΜ φ ω   , οπότε ΜΒ ΜΙ . Άρα ΜΒ ΜΙ ΜΓ  . Σχόλιο Αν Μ είναι το μέσο του τόξου ΒΓ και Ι σημείο του τμήματος ΑΜ, ώστε ΜΙ = ΜΒ , τότε το Ι είναι το έγκεντρο του τριγώνου ΑΒΓ. Πρόταση 3η Έστω Ι το έγκεντρο ενός τριγώνου ΑΒΓ και σημείο Ε στην προέκταση του ΑΙ, προς το Ι, ώστε ΕΙ = ΕΒ. Να αποδειχθεί ότι το Ε είναι σημείο του περιγεγραμμένου κύκλου του τριγώνου ΑΒΓ. Απόδειξη Έχουμε ΕΙ ΕΒ , οπότε το τρίγωνο ΕΒΙ είναι ισοσκελές. Άρα: φ ω ρ ω ˆˆ ˆ ˆ ˆ ˆΕΙΒ ΕΒΙ Α Β Β Β      ˆ ˆφ ω ρ ω φ ρ ΕΑΓ ΕΒΓ        Άρα το τετράπλευρο ΑΒΕΓ είναι εγγράψιμο, δηλαδή το Ε είναι σημείο του περιγεγραμμένου κύκλου του τριγώνου ΑΒΓ. Θεώρημα 4ο Ο εγγεγραμμένος κύκλος ενός τριγώνου ΑΒΓ και ο παρεγγεγραμμένος κύκλος, που αντιστοιχεί στη γωνία ˆA , εφάπτονται με την πλευρά ΒΓ στα σημεία Δ και Ε. Να αποδειχθεί ότι BΔ = ΓΕ .
  • 33. Απόδειξη Γνωρίζουμε ότι: ΒΔ τ β  και ΓΕ ΓΖ τ β   Άρα: ΒΔ ΓΕ τ β   Σχόλιο Το μέσο Μ της πλευράς ΒΓ είναι και μέσο του τμήματος ΔΕ, διότι ΜΒ = ΜΓ και ΒΔ = ΓΕ . ΒΑΣΙΚΟ ΣΧΗΜΑ Στο παρακάτω σχήμα βλέπουμε το έγκεντρο και τα παράκεντρα ενός τριγώνου ΑΒΓ, καθώς και τα σημεία επαφής των παρεγγεγραμμένων κύκλων (Κ), (Λ), (Μ) με τις πλευρές του τριγώνου ΑΒΓ. Στο σχήμα αυτό ισχύουν οι παρακάτω βασικές ιδιότητες: α) ΑΝ ΑΗ τ α   , ΒΝ ΒΔ τ β   , ΓΔ ΓΗ τ γ   . β)  1ΑΑ ΑΘ τ β   , 2ΑΑ ΑΖ τ γ    1ΒΒ ΒΘ τ α   , 2ΒΒ ΒΕ τ γ    1ΓΓ ΓΖ τ α   , 2ΓΓ ΓΕ τ β   γ) ΑΘ ΒΝ , ΒΔ ΓΕ , ΓΖ ΑΗ .
  • 34. δ) ΡΛ ΡΜ , ΣΛ ΣΚ , TK TM (κύκλος Euler). ε) Ο περιγεγραμμένος κύκλος του τριγώνου ΑΒΓ διχοτομεί τα τμήματα ΙΚ, ΙΛ, ΙΜ. 1.7 Σε ένα τετράπλευρο ΑΒΓΔ είναι ΑΒ = ΒΓ και οˆˆΑΒΓ + 2ΑΔΓ =180 . Αν Ε είναι το μέσο του ΑΓ, να αποδειχθεί ότι ˆ ˆΓΔΕ = ΒΔΑ . (Ρουμανία, IMAR – 2009) Λύση Στην προέκταση της ΒΕ παίρνουμε σημείο Ζ τέτοιο, ώστε: ˆˆΓΖΕ ΓΔΕ x  Το τετράπλευρο ΕΓΖΔ είναι εγγράψιμο, οπότε: ˆ ˆΔZΕ ΔΓΕ (1) Είναι όμως:  ο ο1ˆ ˆˆ ˆΑΒΓ 2ΑΔΓ 180 ΑΔΓ (180 ΑΒΓ) 2       ο ˆΑΒΓˆΑΔΓ 90 2    (2)  οˆˆ ˆΑΒΓ ΒΑΓ ΒΓΑ 180    ο ˆΑΒΓˆΒΑΓ 90 2    (3) Επομένως παίρνουμε: (1) ο ˆ ˆˆ ˆΔΖΕ ΔΓΕ 180 ΓΔΑ ΓΑΔ     (3) ο ο οˆ ˆ ˆ ˆ ˆ180 ΓΑΒ ΓΑΔ 180 (ΓΑΒ ΓΑΔ) 180 ΒΑΔ        Είναι λοιπόν οˆˆΔΖΕ ΒΑΔ 180  , δηλαδή οˆˆΔΖΒ ΔΑΒ 180  , οπότε το τετράπλευρο ΑΒΖΔ είναι εγγράψιμο. Έτσι: ˆ ˆˆ ˆΒΔΑ ΒΖΑ ΓΖΕ ΓΔΕ   διότι η ΖΕ είναι μεσοκάθετος του ΑΓ και έτσι ˆ ˆ ˆΒΖΑ ΒΖΓ ΕΖΓ  .
  • 35. Άλλος τρόπος Έστω 1C , 2C , οι περιγεγραμμένοι κύκλοι των τριγώνων ΑΒΓ και ΑΓΔ αντίστοιχα. Επειδή: οˆ ˆ ˆAΟΓ 2ΑΔΓ 180 ΑΒΓ   το ΑΒΓΟ είναι εγγράψιμο. Επειδή ΟΑ ΟΓ και ΑΒ ΑΓ , το Ο βρίσκεται στη ΒΕ και η ΒΟ είναι διάμετρος του κύκλου 1C . Άρα: οˆ ˆOAB OΓΒ 90  Αυτό σημαίνει ότι οι ΑΒ, ΓΒ είναι εφαπτομένες του 2C στα Α, Γ και έτσι η ΔΒ είναι συμμετροδιάμεσος του Δ ΔAΓ , οπότε θα είναι ˆ ˆΑΔΒ ΓΔΕ . 1.8 Δίνεται τετράπλευρο ΑΒΓΔ, με ΑΒ = ΑΔ και οˆˆΒ = Δ = 90 . Στην πλευρά ΓΔ παίρνουμε σημείο Ε και στην πλευρά ΒΓ σημείο Ζ τέτοια, ώστε ΑΕ ΔΖ . Να αποδειχθεί ότι ΑΖ ΒΕ . (Ρωσία – 1995) Λύση Έστω Η το συμμετρικό του Ε ως προς τη διαγώνιο ΑΓ. Προφανώς τα τρίγωνα ΑΒΓ και ΑΔΓ είναι ίσα και έτσι το Η ανήκει στη ΒΓ. Ας είναι Κ το σημείο τομής των ΑΕ, ΔΖ και Λ το σημείο τομής των ΑΖ και ΒΕ. Φέρνουμε τις ΚΛ, ΑΗ και ΔΗ. Είναι: x y o ˆ( 90 ΔΑΕ)  και y ω (λόγω συμμετρίας) δηλαδή x ω . Επομένως το ΑΔΗΖ είναι εγγράψιμο, οπότε φ ρ (βλέπουν το ΑΔ) και ρ ν (λόγω συμμετρίας ως προς την ΑΓ). Επειδή φ ν , το ΚΛΖΕ είναι επίσης εγγράψιμο, οπότε οˆ ˆΕΛΖ ΕΚΖ 90  . Άρα ΒΕ ΑΖ .
  • 36. Θεώρημα 5ο Αν ρ, αρ , βρ , γρ είναι οι ακτίνες του εγγεγραμμένου και των παρεγγεγραμμένων κύκλων ενός τριγώνου ΑΒΓ, τότε α β γρ + ρ + ρ = ρ + 4R , όπου R είναι η ακτίνα του περιγεγραμμένου κύκλου του τριγώνου ΑΒΓ. Απόδειξη Έστω Δ, Ε, Ζ, Η οι προβολές των Ι, Μ, Κ, Λ στην ευθεία ΒΓ. Σύμφωνα με βασικό θεώρημα είναι: BΔ ΓΖ τ β   , ΒΕ ΓΗ τ α   Αν Ρ είναι το μέσο της ΒΓ, τότε το Ρ είναι και μέσο του ΔΖ. Η μεσοκάθετος της ΒΓ τέμνει την ΑΙ στο Τ, οπότε το Τ είναι σημείο του περιγεγραμμένου κύκλου του τριγώνου ΑΒΓ (θεώρημα Νότιου Πόλου). Είναι επίσης ΡΕ ΡΗ , οπότε η μεσοκάθετος του ΕΗ, δηλαδή του ΒΓ, διέρχεται από το μέσο Σ του τμήματος ΜΛ (λόγω του τραπεζίου ΜΕΗΛ). Όμως από το μέσο Σ του ΜΛ διέρχεται ο περιγεγραμμένος κύκλος του Δ ΑΒΓ μια και αυτός είναι ο κύκλος Euler για το τρίγωνο ΚΛΜ (καθόσον το Ι είναι ορθόκεντρο στο Δ ΚΛΜ ).  Στο τραπέζιο ΙΔΚΖ είναι: αρ ρΚΖ ΙΔ ΤΡ 2 2     Στο τραπέζιο ΜΕΗΛ είναι: γ βρ ρΜΕ ΛΗ ΡΣ 2 2    Με πρόσθεση κατά μέλη παίρνουμε: α γ βρ ρ ρ ρ ΤΡ ΡΣ 2      Αλλά ΤΡ ΡΣ 2R  , αφού η μεσοκάθετος μιας χορδής ορίζει με τον κύκλο διάμετρο. Άρα: α γ β α β γ ρ ρ ρ ρ 2R ρ ρ ρ ρ 4R 2          Σχόλιο Το γεγονός ότι το μέσο Σ του ΜΛ ανήκει στον περιγεγραμμένο κύκλο του Δ ΑΒΓ προκύπτει και ως εξής: Φέρουμε τη ΓΣ. Στο ορθογώνιο τρίγωνο ΓΜΛ η ΓΣ είναι διάμεσος, οπότε ΓΣ = ΣΛ και έτσι ˆ ˆΣΛΓ = ΣΓΛ . Άρα:
  • 37. ο οˆ ˆ ˆˆΓΣΛ =180 2ΣΛΓ =180 2(ΣΛΒ + ΒΛΓ) =  ο ο ο ˆˆ ˆΓ Α 180 Β ˆ=180 2 + =180 2 = Β 2 2 2            Επομένως το τετράπλευρο ΑΒΓΣ είναι εγγράψιμο. Θυμίζουμε ότι από βασική πρόταση στις διχοτόμους τριγώνου, είναι ˆΑˆΒΛΓ = 2 και ˆΓˆΒΛΑ = 2 . 1.9 Σε ένα τρίγωνο ΑΒΓ (ΑΒ < ΑΓ) φέρουμε το ύψος ΒΔ και τη διάμεσο ΒΜ. Έστω Κ, Λ οι προβολές των σημείων Α, Γ πάνω στη διχοτόμο της γωνίας ˆΒ . Να αποδειχθεί ότι: α) τα σημεία Κ, Μ, Λ, Δ ανήκουν σε κύκλο C, β) το κέντρο Σ του C ανήκει στον περιγεγραμμένο κύκλο του τριγώνου ΡΔΜ, όπου Ρ είναι το μέσο του τμήματος ΑΒ. Λύση α) Αν η ΑΚ τέμνει τη ΒΓ στο Ν, τότε το τρίγωνο ΒΑΝ είναι ισοσκελές, διότι η διχοτόμος ΒΚ είναι και ύψος. Έτσι το Κ είναι μέσο του ΑΝ, οπότε ΚΜ // ΓΝ (από το τρίγωνο ΑΝΓ). Επομένως: ˆ ˆΒΓΑ ΡΜΑ x y   Αλλά το τετράπλευρο ΒΔΛΓ είναι εγγράψιμο, διότι: οˆ ˆBΔΓ ΒΛΓ 90  Έτσι x ω , που σημαίνει τελικά ότι y ω . Άρα τα σημεία Δ, Κ, Μ, Λ είναι ομοκυκλικά. β) Έστω Σ το κέντρο του κύκλου (Δ, Κ, Μ, Λ). Τότε: ˆ ˆˆ ˆΔΣΜ 2ΔΛΜ 2(ω φ) 2ω 2φ 2Γ 2ΚΔΜ        ˆ ˆ2Γ 2ΑΒΚ  (το ΒΑΔΚ είναι εγγράψιμο, οπότε ˆ ˆΚΔΜ ΑΒΚ ) ˆ ˆ2Γ Β  αφού η ΒΚ είναι διχοτόμος της ˆΒ . Επίσης είναι: ο ˆˆ ˆ ˆ ˆΔΡΜ ΑΡΜ ΑΡΔ Β (180 2Α)     (διότι ΡΜ // ΒΓ και ΡΔ ΡΑ) ο ο o οˆ ˆ ˆ ˆ ˆˆ ˆ ˆ ˆΒ 2Α 180 Α (Α Β) 180 A (180 Γ) 180 A Γ             Είναι λοιπόν: οˆ ˆˆ ˆ ˆ ˆ ˆ ˆ ˆΔΣΜ ΔΡΜ (2Γ Β) (Α Γ) Α Β Γ 180         οπότε το τετράπλευρο ΔΡΜΣ είναι εγγράψιμο. Αυτό σημαίνει ότι το Σ ανήκει στον περιγεγραμμένο κύκλο του τριγώνου ΡΔΜ (δηλαδή στον κύκλο Euler του τριγώνου ΑΒΓ).
  • 38. Θεώρημα 6ο Έστω Ο το περίκεντρο ενός οξυγώνιου τριγώνου ΑΒΓ και x, y, z οι αποστάσεις του Ο από τις πλευρές ΒΓ, ΑΓ, ΑΒ αντίστοιχα. Να αποδειχθεί ότι: x + y + z = R + ρ όπου R, ρ είναι οι ακτίνες του περιγεγραμμένου και του εγγεγραμμένου κύκλου αντίστοιχα στο τρίγωνο ΑΒΓ. Απόδειξη Θεωρούμε το παράκεντρο Κ του τριγώνου ΑΒΓ. Έστω Ι το έγκεντρο, ΙΔ, ΚΕ ΒΓ και Ρ το μέσο της ΒΓ. Είναι γνωστό ότι: ΒΔ ΓΕ τ β   οπότε ΡΔ ΡΕ . Αφού ΟΡ ΒΓ , η ΟΡ θα περάσει από το μέσο Ν του ΙΚ, το οποίο είναι (από βασική πρόταση) σημείο του περιγεγραμμένου κύκλου του τριγώνου ΑΒΓ. Είναι λοιπόν: KE IΔ x OP ON PN R 2        α α 1 1 R (ρ ρ) (2R ρ ρ) 2 2       Όμοια είναι: β 1 y (2R ρ ρ) 2    και γ 1 z (2R ρ ρ) 2    Άρα, με πρόσθεση κατά μέλη, παίρνουμε: α β γ 1 x y z (6R ρ ρ ρ 3ρ) 2        Γνωρίζουμε όμως ότι α β γρ ρ ρ ρ 4R    , οπότε:   1 1 x y z 6R (ρ 4R) 3ρ (2R 2ρ) R ρ 2 2           Σημείωση Πρέπει να τονίσουμε με έμφαση ότι ο τύπος αυτός ισχύει μόνο στα οξυγώνια τρίγωνα. Αν π.χ. οι γωνίες ˆΑ , ˆΒ , ˆΓ είναι αντίστοιχα αμβλείες, τότε ισχύει: -x + y + z = R + ρ , x - y + z = R + ρ , x + y - z = R + ρ
  • 39. Θεώρημα 7ο Θεωρούμε τετράπλευρο ΑΒΓΔ εγγεγραμμένο σε κύκλο. Αν 1ρ , 2ρ είναι οι ακτίνες των εγγεγραμμένων κύκλων, αντίστοιχα, στα τρίγωνα ΑΒΓ, ΑΓΔ και 3ρ , 4ρ οι ακτίνες των εγγεγραμμένων κύκλων στα τρίγωνα ΒΓΔ, ΒΑΔ, τότε 1 2 3 4ρ + ρ = ρ + ρ . (Θεώρημα των Ιαπώνων) Απόδειξη Ας σημειώσουμε με α, β, γ, δ, x, y τις αποστάσεις του Ο από τις πλευρές ΑΒ, ΒΓ, ΓΔ, ΔΑ και από τις διαγώνιες ΑΓ, ΒΔ αντίστοιχα. Με βάση το προηγούμενο θεώρημα είναι: 1α β x R ρ    (1) 2γ δ x R ρ    (2) 3β γ y R ρ    (3) 4α δ y R ρ    (4) Προσθέτουμε τις σχέσεις (1), (2) και έχουμε: 1 22R ρ ρ α β γ δ      Από τις σχέσεις (3), (4) παίρνουμε: 3 42R ρ ρ α β γ δ      Έχουμε αποδείξει λοιπόν ότι: 1 2 3 4 1 2 3 42R ρ ρ 2R ρ ρ ρ ρ ρ ρ         Πρόταση 8η Ο εγγεγραμμένος κύκλος (Ι,ρ) ενός τριγώνου ΑΒΓ εφάπτεται των πλευρών ΑΒ, ΑΓ στα σημεία Ε, Ζ αντίστοιχα. Αν η ευθεία ΓΙ τέμνει την ευθεία ΕΖ στο σημείο Ρ, τότε η γωνία ˆΒΡΓ είναι ορθή. Απόδειξη  Στο τρίγωνο ΙΒΓ είναι: ο ˆˆ ˆΒ Γ Αˆ ˆ ˆΡΙΒ ΙΒΓ ΙΓΒ 90 2 2 2       (1)  Στο ισοσκελές τρίγωνο ΑΕΖ (ΑΕ ΑΖ) είναι: ο οˆ ˆˆ ˆΑΕΖ ΑΖΕ 180 Α 2φ 180 Α       ο ˆΑ φ 90 2    (2)
  • 40. Επειδή ο ˆΑˆ ˆΑΕΡ ΡΙΒ 90 2    , το τετράπλευρο ΡΕΒΙ είναι εγγράψιμο. Άρα oˆ ˆΒΡΙ BEI 90  . Θεώρημα 9ο Δίνεται ισόπλευρο τρίγωνο ΑΒΓ, εγγεγραμμένο σε κύκλο, και Μ τυχαίο σημείο του μικρού τόξου ΒΓ . Να αποδειχθεί ότι ΜΒ + ΜΓ = ΜΑ . (Θεώρημα van Schooten) Απόδειξη Έστω σημείο Ν στη ΜΑ, ώστε ΜΝ ΜΒ . Το Δ MBN είναι ισόπλευρο, διότι οˆ ˆΒΜΑ ΒΓΑ 60  . Άρα: BN BM και οˆ ˆ ˆABN MBΓ 60 ΝΒΓ   Αλλά τώρα είναι Δ Δ ΑΝΒ ΜΒΓ , αφού: ΑΒ ΒΓ , ΒΝ ΒΜ και οˆ ˆ ˆΑΒΝ ΜΒΓ 60 ΝΒΓ   Άρα ΑΝ ΜΓ , δηλαδή: ΜΑ ΜΝ ΝΑ ΜΒ ΜΓ    Άλλος τρόπος Το θεώρημα Πτολεμαίου στο εγγεγραμμένο τετράπλευρο ΑΒΜΓ δίνει: ΑΒ ΜΓ ΑΓ ΜΒ ΑΜ ΒΓ α ΜΓ α ΜΒ ΑΜ α            ΜΓ ΜΒ ΜΑ ΜΑ ΜΒ ΜΓ      Άλλος τρόπος Στην προέκταση της ΒΜ παίρνουμε τμήμα ΜΕ ΜΓ . Αλλά τότε τα τρίγωνα ΑΜΓ και ΕΒΓ είναι ίσα, οπότε: ΒΕ ΑΜ ΒΜ ΜΕ ΑΜ ΑΜ ΜΒ ΜΓ       Θεώρημα 10ο Σε τρίγωνο ΑΒΓ, που είναι εγγεγραμμένο σε κύκλο (Κ,R), φέρνουμε το ύψος ΑΔ. Αν ΑΒ < ΑΓ , να αποδειχθεί ότι: α) ˆ ˆΔΑΒ = ΚΑΓ , β) ˆ ˆ ˆΚΑΔ = Β - Γ . Απόδειξη
  • 41. α) Φέρνουμε τη διάμετρο ΑΚΕ και τη ΓΕ. Το τρίγωνο ΑΓΕ είναι ορθογώνιο, οπότε: o oˆ ˆy 90 E 90 B x     διότι και το τρίγωνο ΑΔΒ είναι ορθογώνιο. Άρα x y , δηλαδή ˆ ˆΔΑΒ ΚΑΓ . β) Είναι: οˆ ˆ ˆ ˆΚΑΔ ΓΑΔ ΓΑΕ (90 Γ) y      o οˆ ˆ ˆ ˆ(90 y) Γ (90 x) Γ Β Γ        Σχόλια α) Επειδή ˆ ˆΔΑΒ = ΚΑΓ , η διχοτόμος της γωνίας ˆΒΑΓ διχοτομεί τη γωνία ˆΚΑΔ . Επομένως, αν Μ είναι το μέσο του τόξου ΒΓ , τότε ˆ ˆΜΑΔ = ΜΑΚ . β) Από την παραπάνω πρόταση προκύπτει το θεώρημα: "Το ορθόκεντρο και το περίκεντρο ενός τριγώνου είναι ισογώνια σημεία". Υπενθυμίζουμε ότι δύο ημιευθείες λέγονται ισογώνιες ως προς τις πλευρές μιας γωνίας, όταν είναι συμμετρικές ως προς τη διχοτόμο της γωνίας αυτής. Θεώρημα 11ο Να αποδειχθεί ότι οι ακτίνες της περιγεγραμμένης περιφέρειας ενός τριγώνου, που αντιστοιχούν στις κορυφές του είναι αντίστοιχα κάθετες στις πλευρές του ορθικού τριγώνου. (Θεώρημα Nagel) Απόδειξη Έστω O το περίκεντρο, Η το ορθόκεντρο και ΒΔ, ΓΕ ύψη του τριγώνου ΑΒΓ. Θα αποδειχθεί ότι OA ΔΕ . Έστω ΚΑΛ η κοινή εφαπτομένη στο Α. Τότε:  ˆˆΑΓΒ ΒΑΚ (x y) , διότι η ˆΓ είναι εγγεγραμμένη και η ˆΒΑΚ είναι αντίστοιχη γωνία χορδής και εφαπτομένης.  ˆ ˆΑΓΒ ΔΕΑ (x ω) , διότι το τετράπλευρο ΒΕΔΓ είναι εγγράψιμο οˆˆ(ΒΕΓ ΒΔΓ 90 )  . Επομένως y ω , που σημαίνει ότι ΚΛ // ΔΕ. Όμως ΟΑ ΚΛ , οπότε ΟΑ ΔΕ . Όμοια, αν ΑΖ είναι το τρίτο ύψος του Δ ΑΒΓ , τότε ΟΒ ΕΖ και ΟΓ ΔΖ . Με απλά λόγια, οι ακτίνες ΟΑ, ΟΒ, ΟΓ είναι αντίστοιχα κάθετες στις πλευρές ΔΕ, ΕΖ, ΖΔ του ορθικού τριγώνου ΔΕΖ, του τριγώνου ΑΒΓ.
  • 42. Θεώρημα 12ο Στο εξωτερικό ενός τριγώνου ΑΒΓ θεωρούμε τα ισόπλευρα τρίγωνα ΑΒΔ, ΒΓΕ και ΓΑΖ. Να αποδειχθεί ότι: α) ΑΕ = ΒΖ = ΓΔ , β) οι ευθείες ΑΕ, ΒΖ, ΓΔ συντρέχουν σε σημείο Σ και ισχύει ΣΑ + ΣΒ + ΣΓ = ΑΕ = ΒΖ = ΓΔ . γ) τα κέντρα Κ, Λ, Μ των τριγώνων ΑΒΔ, ΒΓΕ, ΓΑΖ σχηματίζουν ισόπλευρο τρίγωνο. (Θεώρημα Steiner) Απόδειξη α) Τα τρίγωνα ΑΔΓ και ΑΒΖ έχουν:  ΑΔ ΑΒ γ  και ΑΓ ΑΖ β  ,  οˆ ˆ ˆ ˆ ˆ ˆΔΑΓ ΔΑΒ ΒΑΓ 60 ΒΑΓ ΒΑΓ ΓΑΖ       ˆΒΑΖ . Επομένως Δ Δ ΑΔΓ ΑΒΖ , οπότε ΔΓ ΒΖ . Όμοια είναι Δ Δ ΒΔΓ ΑΒΕ , οπότε ΔΓ ΑΕ . Άρα ΑΕ ΒΖ ΓΔ  . β) Έστω ότι οι ΒΖ, ΓΔ τέμνονται στο Σ. Επειδή Δ Δ ΑΔΓ ΑΒΖ , είναι: ˆ ˆΑΔΓ ΑΒΖ x  και ˆ ˆAZB AΓΔ y  Επειδή ˆ ˆAΔΣ ΑΒΣ x  και ˆ ˆAZΣ ΑΓΣ y  , τα τετράπλευρα ΑΣΒΔ, ΑΣΓΖ είναι εγγράψιμα. Επομένως: οˆ ˆΔΣΑ ΔΒΑ 60  και οˆˆΔΣΒ ΔΑΒ 60  Επειδή οˆ ˆΒΣΔ 60 ΒΕΓ  , το ΒΣΓΕ είναι εγγράψιμο. Άρα οˆ ˆΒΣΕ ΒΓΕ 60  . Είναι λοιπόν: ο ο ο οˆˆ ˆ ˆ ˆ ˆ ˆΑΣΕ ΑΣΔ ΔΣΒ ΒΣΕ ΑΒΔ ΔΑΒ ΒΓΕ 60 60 60 180          οπότε η ΑΣΕ είναι ευθεία. Επομένως και η ΑΕ διέρχεται από το Σ. Από το θεώρημα van Schooten είναι όμως ΣΒ ΣΓ ΣΕ  , οπότε (α) ΣΑ ΣΒ ΣΓ ΣΑ ΣΕ ΑΕ ΓΔ ΒΖ       .
  • 43. γ) Έχουμε αποδείξει ότι: οˆ ˆ ˆΑΣΒ ΒΣΓ ΓΣΑ 120   Τα Κ, Λ είναι κέντρα των κύκλων (Σ,Α,Δ,Β) και (Σ,Β,Ε,Γ). Άρα η ΚΛ είναι κάθετη στην κοινή χορδή τους ΣΒ. Όμοια, η ΛΜ είναι κάθετη στην κοινή χορδή ΣΓ των κύκλων (Λ), (Μ) και η ΚΜ είναι κάθετη στην κοινή χορδή ΑΣ των κύκλων (Κ) και (Μ). Επειδή: οˆ ˆ ˆΑΣΒ ΒΣΓ ΓΣΑ 120   είναι: οˆˆ ˆΚ Λ Μ 60   και έτσι το τρίγωνο ΚΛΜ είναι ισόπλευρο. Σχόλια i) Αν όλες οι γωνίες του τριγώνου ΑΒΓ είναι μικρότερες από ο 120 , τότε το Σ είναι εσωτερικό σημείο του τριγώνου ΑΒΓ. Στην περίπτωση αυτή το σημείο Σ λέγεται σημείο Fermat – Torricelli και είναι το μοναδικό σημείο του τριγώνου για το οποίο το άθροισμα ΣΑ + ΣΒ + ΣΓ γίνεται ελάχιστο. ii) Στο ερώτημα (γ) είδαμε ότι τα κέντρα των τριγώνων ΔΑΒ, ΕΒΓ, ΖΓΑ σχηματίζουν ισόπλευρο τρίγωνο. Το συμπέρασμα αυτό είναι γνωστό ως θεώρημα του Ναπολέοντα. Η απόδειξη μπορεί να γίνει επίσης με χρήση μιγαδικών αριθμών, με τριγωνομετρία, αλλά και με χρήση ενός γεωμετρικού μετασχηματισμού που λέγεται στροφή. Θεώρημα 13ο Αν Η είναι το ορθόκεντρο και Ο το περίκεντρο ενός τριγώνου ΑΒΓ, να αποδειχθεί ότι ΑΗ = 2ΟΜ , όπου Μ είναι το μέσο του ΒΓ. Απόδειξη Θεωρούμε το αντιδιαμετρικό Ν του Β ως προς το Ο, δηλαδή τη διάμετρο ΒΟΝ. Φέρουμε τις ΝΑ, ΝΓ. Επειδή ΝΓ ΓΒ και ΑΗ ΒΓ , είναι ΑΗ // ΝΓ. Είναι ΝΑ ΑΒ και ΓΗ ΑΒ , οπότε ΓΗ // ΝΑ. Το ΝΑΗΓ είναι λοιπόν παραλληλόγραμμο, οπότε ΑΗ ΝΓ . Στο τρίγωνο ΒΓΝ το ΟΜ ενώνει τα μέσα δύο πλευρών. Άρα ΓΝ ΟΜ 2  , δηλαδή ΓΝ 2ΟΜ . Επομένως: ΑΗ ΝΓ 2ΟΜ 
  • 44. Θεώρημα 14ο Το συμμετρικό του ορθόκεντρου ενός τριγώνου ΑΒΓ, ως προς τυχαία πλευρά του, είναι σημείο του περιγεγραμμένου κύκλου του τριγώνου αυτού. Απόδειξη Θεωρούμε το ύψος ΑΔ, το ορθόκεντρο Η του Δ ΑΒΓ και το συμμετρικό Ν του Η ως προς το ΒΓ. Για να ανήκει το Ν στον περιγεγραμμένο κύκλο του Δ ΑΒΓ , αρκεί να αποδείξουμε ότι το τετράπλευρο ΑΒΝΓ είναι εγγράψιμο. Επειδή όμως ˆ ˆΒΝΓ ΒΗΓ , έχουμε: οˆ ˆ ˆˆ ˆ ˆΒΝΓ ΒΑΓ ΒΗΓ ΒΑΓ ΖΗΕ ΖΑΕ 180      διότι το ΑΖΗΕ είναι εγγράψιμο οˆ ˆ(ΑΖΗ ΑΕΗ 90 )  . Άρα το ΑΒΝΓ είναι εγγράψιμο, δηλαδή το Ν ανήκει πράγματι στον περιγραμμένο κύκλο του τριγώνου ΑΒΓ. Θεώρημα 15ο Το συμμετρικό του ορθόκεντρου Η ενός τριγώνου ως προς το μέσο μιας τυχαίας πλευράς του, είναι σημείο του περιγεγραμμένου κύκλου του τριγώνου αυτού. Απόδειξη Αρκεί να αποδείξουμε ότι το τετράπλευρο ΑΒΝΓ είναι εγγράψιμο σε κύκλο. Το τετράπλευρο ΗΒΝΓ είναι παραλληλόγραμμο, διότι οι διαγώνιες ΒΓ και ΗΝ διχοτομούνται. Επομένως ˆ ˆΒΝΓ ΒΗΓ . Άρα: οˆ ˆ ˆˆ ˆ ˆΒΝΓ ΒΑΓ ΒΗΓ ΒΑΓ ΕΗΖ ΖΑΕ 180      διότι το τετράπλευρο ΑΖΗΕ είναι εγγράψιμο. Άρα το Ν είναι σημείο του περιγεγραμμένου κύκλου του τριγώνου ΑΒΓ. Σημείωση Επειδή, όπως έχουμε δει στο προηγούμενο θεώρημα, το συμμετρικό του ορθόκεντρου ως προς τυχαία πλευρά είναι επίσης σημείο του περιγεγραμμένου κύκλου, μπορούμε να διατυπώσουμε το εξής θεώρημα: «Τα συμμετρικά του ορθόκεντρου ενός τριγώνου ως προς τις πλευρές του και ως προς τα μέσα των πλευρών του είναι σημεία του περιγεγραμμένου κύκλου του τριγώνου.»
  • 45. Θεώρημα 16ο Να αποδειχθεί ότι σε κάθε τρίγωνο, το ορθόκεντρο, το βαρύκεντρο και το περίκεντρο είναι συνευθειακά σημεία. (Ευθεία Euler) Απόδειξη Έστω Η το ορθόκεντρο και Ο το περίκεντρο του Δ ΑΒΓ . Έστω Μ το μέσο της ΒΓ, Θ το σημείο τομής των ΑΜ και ΟΗ, Ρ το μέσο του ΑΘ και Ν το μέσο του ΘΗ. Θα αποδείξουμε ότι το Θ είναι το βαρύκεντρο του τριγώνου ΑΒΓ, δηλαδή ότι: ΘΑ 2ΘΜ Στο τρίγωνο ΘΗΑ το ΡΝ ενώνει τα μέσα δύο πλευρών, οπότε: ΑΗ 2ΟΜ ΡΝ ΟΜ 2 2    διότι ΑΗ 2ΟΜ . Είναι επίσης ΡΝ // ΑΗ και αφού ΑΗ ΒΓ , είναι ΡΝ ΒΓ . Αφού ΟΜ BΓ , είναι ΡΝ // ΟΜ και επειδή ΟΜ ΡΝ , το ΟΜΝΡ είναι παραλληλόγραμμο. Επομένως οι διαγώνιες ΟΝ και ΡΜ διχοτομούνται, δηλαδή ΘΜ ΘΡ ΡΑ  . Άρα: ΑΘ 2ΘΜ και έτσι το Θ είναι το βαρύκεντρο του τριγώνου ΑΒΓ. Σημείωση Επειδή ΘΟ = ΘΝ = ΝΗ , το βαρύκεντρο διαιρεί το τμήμα ΟΗ σε δύο τμήματα με λόγο 2. Πιο συγκεκριμένα είναι ΘΗ = 2ΘΟ . Θεώρημα 17ο Να αποδειχθεί ότι το ορθόκεντρο ενός τριγώνου είναι το έγκεντρο του τριγώνου που έχει κορυφές τα ίχνη των υψών του. (Θεώρημα του ορθικού τριγώνου) Απόδειξη Φέρνουμε τα ύψη ΑΔ, ΒΕ, ΓΖ του τριγώνου ΑΒΓ και έστω Η το ορθόκεντρο. Θα αποδείξουμε ότι η ΔΑ είναι διχοτόμος της γωνίας ˆΕΔΖ του ορθικού τριγώνου ΔΕΖ.  Το τετράπλευρο ΗΔΒΖ είναι εγγράψιμο, διότι: ο ο οˆ ˆΗΔΒ ΗΖΒ 90 90 180    Επομένως x φ ˆ ˆ(ΗΔΖ ΗΒΖ) .
  • 46.  Το τετράπλευρο ΗΔΓΕ είναι ομοίως εγγράψιμο, οπότε y ω ˆ ˆ(ΗΔΕ ΗΓΕ) . Όμως και το τετράπλευρο ΒΖΕΓ είναι εγγράψιμο, διότι: οˆ ˆΒΖΓ ΒΕΓ 90  Επομένως φ ω ˆ ˆ(ΖΒΕ ΖΓΕ) . Αφού λοιπόν x φ , y ω και φ ω , είναι και x y . Άρα η ΔΑ διχοτομεί τη γωνία ˆEΔΖ . Όμοια, η ΕΒ διχοτομεί τη γωνία ˆΔΕΖ και η ΖΓ διχοτομεί τη γωνία ˆΕΖΔ . Βλέπουμε λοιπόν ότι το Η είναι το έγκεντρο του τριγώνου ΔΕΖ. Σχόλιο Το τρίγωνο ΔΕΖ λέγεται ορθικό τρίγωνο του τριγώνου ΑΒΓ. Θεώρημα 18ο Σε κάθε τρίγωνο, τα μέσα των πλευρών του, τα ίχνη των υψών του και τα μέσα των τμημάτων που συνδέουν το ορθόκεντρο με τις κορυφές του τριγώνου είναι ομοκυκλικά σημεία. (Κύκλος του Euler) Απόδειξη Θα αποδείξουμε ότι ο περιγεγραμμένος κύκλος του τριγώνου με κορυφές τα μέσα Κ, Λ, Μ των πλευρών ΒΓ, ΓΑ, ΑΒ του τριγώνου ΑΒΓ διέρχεται: i) από τα ίχνη των υψών του τριγώνου ΑΒΓ, ii) από τα μέσα των αποστάσεων του ορθοκέντρου από τις κορυφές του τριγώνου ΑΒΓ. i) Φέρνουμε το ύψος ΑΔ. Επειδή ΜΛ // ΒΓ, το ΔΚΛΜ είναι τραπέζιο. Είναι επίσης: ΑΒ ΚΛ ΔΜ 2   διότι στο ορθογώνιο τρίγωνο ΑΔΒ το ΔΜ είναι διάμεσος. Το ΔΚΛΜ είναι λοιπόν ισοσκελές τραπέζιο, οπότε: οˆˆ ˆ ˆΔΜΛ ΔΚΛ ΜΛΚ ΔΚΛ 180    Άρα το ΔΚΛΜ είναι εγγράψιμο σε κύκλο (να τονίσουμε εδώ ότι κάθε ισοσκελές τραπέζιο είναι εγγράψιμο σε κύκλο). Επομένως ο κύκλος (Κ,Λ,Μ), που διέρχεται από τις κορυφές του Δ ΚΛΜ , διέρχεται επίσης από το Δ. Με άλλα λόγια, ο κύκλος (Κ, Λ, Μ) διέρχεται από τα ίχνη των υψών του τριγώνου ΑΒΓ.
  • 47. ii) Θα αποδείξουμε τώρα ότι ο κύκλος (Κ,Λ,Μ) διέρχεται και από το μέσο Ρ του τμήματος ΗΑ, όπου Η το ορθόκεντρο του τριγώνου ΑΒΓ. Αρκεί λοιπόν να αποδειχθεί ότι το τετράπλευρο ΚΛΡΜ είναι εγγράψιμο. Επειδή ΜΡ // ΗΒ και ΡΛ // ΗΓ, είναι:  ˆ ˆ ˆΜΡΛ ΒΗΓ ΖΗΕ   οˆˆ ˆ ˆΜΡΛ ΜΚΛ ΖΗΕ ΖΑΛ 180    διότι το ΑΖΗΕ είναι εγγράψιμο και το ΑΜΚΛ είναι παραλληλόγραμμο, οπότε ˆ ˆˆΜΚΛ Α ΖΑΕ  . Άρα το τετράπλευρο ΜΡΛΚ είναι εγγράψιμο, οπότε ο κύκλος (Κ,Λ,Μ) διέρχεται και από το μέσο Ρ του ΗΑ. Όμοια, ο κύκλος αυτός διέρχεται και από τα μέσα των ΗΒ, ΗΓ. Άρα τελικά τα εννέα παραπάνω σημεία είναι ομοκυκλικά. Σχόλια Από τις παραγράφους (i) και (ii) συμπεραίνουμε ότι και τα 9 σημεία βρίσκονται στον ίδιο κύκλο. Ο κύκλος αυτός λέγεται κύκλος του Euler.  Το κέντρο του κύκλου του Euler είναι το μέσο Ι του τμήματος που συνδέει το ορθόκεντρο με το περίκεντρο του τριγώνου ΑΒΓ. Αυτό συμβαίνει διότι π.χ. η μεσοκάθετος της χορδής ΔΚ του κύκλου του Euler διέρχεται από το μέσο Ι της μη παράλληλης πλευράς ΟΗ του τραπεζίου ΗΔΚΟ (ΗΔ // ΟΚ, διότι ΗΔ, ΟΚ ΒΓ) .  Η ακτίνα του κύκλου του Euler είναι ίση με το μισό της ακτίνας R του περιγεγραμμένου κύκλου του τριγώνου ΑΒΓ. Πραγματικά, από το τρίγωνο ΗΟΑ προκύπτει ότι: 9 ΟΑ R R = ΙΡ = ΙΡ = 2 2 
  • 48. Αλλά η ΙΡ είναι ακτίνα του κύκλου του Euler και έτσι ο ισχυρισμός μας αποδείχθηκε. Άλλος τρόπος Γνωρίζουμε από τα θεωρήματα 15 και 16 ότι το συμμετρικό του ορθόκεντρου Η ως προς την πλευρά ΒΓ καθώς και ως προς το μέσο Μ της πλευράς ΒΓ είναι σημεία του περιγεγραμμένου κύκλου του Δ ΑΒΓ . Αν λοιπόν Θ είναι το μέσο του ΗΒ, Δ το ίχνος του ύψους από το Α και Μ το μέσο του ΒΓ, αρκεί να αποδειχθεί ότι TΘ ΤΔ ΤΜ  , όπου Τ είναι το μέσο του ΟΗ.  Φέρουμε ΤΣ ΒΓ . Στο τραπέζιο ΟΜΔΗ το ΤΣ είναι διάμεσος (αφού ΤΣ // ΟΜ και το Τ είναι μέσο του ΟΗ), οπότε η ΤΣ είναι μεσοκάθετος του ΜΔ. Άρα: ΟΝ R ΤΔ ΤΜ 2 2    (1) Σημειώνουμε ότι στο τρίγωνο ΗΟΝ το ΤΜ ενώνει τα μέσα των πλευρών ΗΟ, ΗΝ.  Στο τρίγωνο ΗΟΒ το ΤΘ ενώνει τα μέσα Τ, Θ των πλευρών ΗΟ, ΗΒ. Επομένως: ΟΒ R TΘ 2 2   (2) Από τις (1) και (2) συμπεραίνουμε ότι: R TΘ ΤΔ ΤΜ 2    Άρα ο κύκλος με κέντρο το μέσο Τ του ΟΗ και ακτίνα R 2 διέρχεται από το Θ, το Δ και το Μ, δηλαδή τελικά από τα μέσα των τμημάτων ΗΒ, ΗΓ, ΗΑ, από τα ίχνη των υψών και από τα μέσα των πλευρών του τριγώνου ΑΒΓ. Το Τ είναι λοιπόν το κέντρο του κύκλου Euler, η δε ακτίνα του κύκλου αυτού είναι R 2 . Θεώρημα 19ο
  • 49. Δίνεται τρίγωνο ΑΒΓ και το τρίγωνο ΚΛΜ που σχηματίζουν τα παράκεντρα του τριγώνου αυτού. Τότε ο περιγεγραμμένος κύκλος του τριγώνου ΑΒΓ διέρχεται από τα μέσα των τμημάτων ΚΛ, ΛΜ, ΚΜ. Απόδειξη Επειδή oˆIAM 90 , είναι: KA MΛ Ανάλογα είναι ΛΒ ΚΜ , ΜΓ ΚΛ . Άρα στο τρίγωνο ΚΛΜ το έγκεντρο Ι του Δ ΑΒΓ είναι ορθόκεντρο. Στο τρίγωνο ΚΛΜ τα Α, Β, Γ είναι ίχνη των υψών, οπότε από τα Α, Β, Γ περνάει ο κύκλος Euler του τριγώνου ΚΛΜ. Αλλά ο κύκλος Euler περνάει και από τα μέσα των πλευρών ΚΛ, ΛΜ, ΚΜ. Με άλλα λόγια, ο περιγεγραμμένος κύκλος του τριγώνου ΑΒΓ δέρχεται από τα μέσα των τμημάτων ΚΛ, ΛΜ, ΚΜ του τριγώνου ΚΛΜ που σχηματίζουν τα παράκεντρα του Δ ΑΒΓ . Σημείωση Ο κύκλος (Α,Β,Γ) διέρχεται επίσης από τα μέσα των ΙΜ, ΙΚ, ΙΛ, αφού στο τρίγωνο ΚΛΜ ο κύκλος αυτός είναι ο κύκλος Euler και το Ι είναι το ορθόκεντρο του Δ ΚΛΜ . Θεώρημα 20ο Οι προβολές τυχαίου σημείου του περιγεγραμμένου κύκλου ενός τριγώνου πάνω στις πλευρές του τριγώνου αυτού, είναι συνευθειακά σημεία. (Ευθεία Simson) Απόδειξη Έστω ΜΔ ΒΓ , ΜΕ ΑΓ , ΜΖ ΑΒ . Επειδή η ΑΕΓ είναι ευθεία, για να είναι τα σημεία Δ, Ε, Ζ συνευθειακά, αρκεί να αποδείξουμε ότι ˆ ˆΑΕΖ ΔΕΓ .
  • 50.  Το τετράπλευρο ΓΔΕΜ είναι εγγράψιμο, οπότε: ˆ ˆΔΕΓ ΔΜΓ x   Το τετράπλευρο ΑΕΜΖ είναι εγγράψιμο, οπότε: ˆ ˆAEZ AMZ y   Τα τρίγωνα ΜΔΓ, ΜΖΑ είναι ορθογώνια, οπότε: ο oˆ ˆ ˆΔΜΓ 90 ΜΓΔ x 90 MΓΔ     (1) ο oˆ ˆˆΑΜΖ 90 ΖΑΜ y 90 ZAM     (2)  Από το εγγεγραμμένο τετράπλευρο ΑΒΓΜ παίρνουμε όμως ότι: ˆ ˆˆ ˆMΓΒ ΜΑΖ ΜΓΔ ΖΑΜ   (3) Οι σχέσεις (1) και (2), λόγω της (3), δίνουν x y . Άρα τα σημεία Δ, Ε, Ζ είναι συνευθειακά. Σχόλια α) Είναι σημαντικό να παρατηρήσουμε ότι ισχύει και το αντίστροφο του θεωρήματος αυτού. Έτσι: "Αν οι προβολές ενός σημείου του επιπέδου εκτός τριγώνου πάνω στις πλευρές του είναι συνευθειακά σημεία, τότε το σημείο αυτό ανήκει στον περιγεγραμμένο κύκλο του τριγώνου." β) Η ευθεία που σχηματίζουν οι προβολές των σημείων Μ πάνω στις πλευρές του τριγώνου λέγεται ευθεία Simson του Μ για το τρίγωνο αυτό ή ακόμα ευθεία Simson – Wallace. γ) Το γενικευμένο θεώρημα Simson – Wallace έχει την εξής μορφή: Αν Μ είναι σημείο του περιγεγραμμένου κύκλου ενός τριγώνου ΑΒΓ και Δ, Ε, Ζ είναι σημεία των ΒΓ, ΓΑ, ΑΒ αντίστοιχα, ώστε οι προσανατολισμένες γωνίες ˆMΔΓ , ˆΜΕΓ , ˆΜΖΑ να είναι ίσες, τότε τα σημεία Δ, Ε, Ζ είναι συνευθειακά. Η απόδειξη προκύπτει, όπως και πριν, από τα εγγράψιμα τετράπλευρα ΜΕΔΓ, ΜΕΑΖ και το εγγεγραμμένο τετράπλευρο ΑΒΓΜ, οπότε τελικά είναι ˆ ˆΔΕΓ = ΑΕΖ .
  • 51. Σελίδα 1 από 5 Μπάμπης Στεργίου – Μαθηματικός 25/02/2017 Οι εκφωνήσεις των ασκήσεων
  • 52. Σελίδα 2 από 5 Μπάμπης Στεργίου – Μαθηματικός 25/02/2017 A. Τρίγωνα – τετράπλευρα ArJun.1 Δίνεται ισόπλευρο τρίγωνο ΑΒΓ και σημείο Δ στην πλευρά ΒΓ. Στην προέκταση της ΑΓ παίρνουμε τμήμα ΓΕ ΒΔ . Να αποδείξετε ότι ΔΑ ΔΕ . ArJun.2 Δίνεται τετράγωνο ΑΒΓΔ, σημείο Ε στην πλευρά ΑΒ και σημείο Ζ στην πλευρά ΒΓ, ώστε ΒΕ ΒΖ . Η κάθετη ΒΚ από το Β προς την ΓΕ τέμνει την ΑΔ στο σημείο Η. Να αποδείξετε ότι: α) ΓΕ ΒΗ και ΓΕ ΒΗ . β) Το ΔΗΖΓ είναι ορθογώνιο. γ) Η γωνία ˆΔΚΖ είναι ορθή. ArJun.3 Δίνεται τρίγωνο ΑΒΓ, το μέσο Μ της ΒΓ, μια ημιευθεία Αx στο εσωτερικό της γωνίας ˆΑ που δεν διέρχεται από το Μ και οι προβολές Δ, Ε των σημείων Β και Γ πάνω στην Αx. Να αποδείξετε ότι: ΜΔ ΜΕ . ArJun.4 Δίνεται τετράγωνο ΑΒΓΔ και το μέσο Μ της πλευράς ΑΒ. Η κάθετη προς την ΑΓ στο σημείο Α τέμνει την ευθεία ΓΒ στο σημείο Ε. Να αποδείξετε ότι τα σημεία Γ, Β, Ε βρίσκονται στην ίδια ευθεία. ArJun.5 Έστω Μ το μέσο της πλευράς ΑΒ ενός τετραγώνου ΑΒΓΔ. Η ευθεία ΔΜ τέμνει την ευθεία ΓΒ στο σημείο Ε. Να αποδείξετε ότι το τρίγωνο ΑΓΕ είναι ορθογώνιο και ισοσκελές. ArJun.6 Στη βάση ΒΓ ισοσκελούς τριγώνου ΑΒΓ παίρνουμε σημείο Δ. Στις πλευρές ΑΓ, ΑΒ παίρνουμε αντίστοιχα τα σημεία Ε και Ζ έτσι, ώστε: ˆ ˆΔΑΒ 2ΓΔΕ και ˆ ˆΔΑΓ 2ΒΔΖ . Να αποδείξετε ότι το τρίγωνο ΑΖΕ είναι ισοσκελές. ArJun.7 Στο εσωτερικό ενός ισοσκελούς τριγώνου ΑΒΓ (ΑΒ ΑΓ) υπάρχει ένα σημείο Μ, ώστε: οˆΜΒΓ 30 και 3ˆ ˆΜΑΒ ΒΑΓ 4  . Να αποδείξετε ότι οˆΑΜΓ 150 . ArJun.8 Σε ένα τρίγωνο ΑΒΓ είναι: οˆΒ 30 και οˆΓ 45 Αν Ε είναι το συμμετρικό του Β ως προς το σημείο Γ, να αποδειχθεί ότι οˆΕ 15 . (Vrenceanu – 2006) ArJun.9 Έστω ΑΒΓ ισοσκελές τρίγωνο με ΑΒ ΑΓ και οˆΑ 40 . Στο εσωτερικό του ΑΒΓ παίρνουμε σημείο Δ τέτοιο, ώστε οˆΔΑΓ 10 και ΑΔ ΒΓ . Να αποδείξετε ότι οˆΔΓΑ 20 . ArJun.10 Δίνεται ισόπλευρο τρίγωνο ΑΒΓ και σημείο Δ στην πλευρά ΑΓ. Η διχοτόμος της γωνίας ΑΒΔ και η παράλληλη από το Α προς τη ΒΓ τέμνονται στο σημείο Ε. Να αποδείξετε ότι ΒΔ ΑΕ ΓΔ  . (Sperante Olympia – 2010) ArJun.11 Στις πλευρές ΑΒ και ΑΓ ενός τριγώνου ΑΒΓ παίρνουμε τα σημεία Δ, Ε αντίστοιχα έτσι, ώστε ΒΔ ΓΕ . Αν Μ, Ν είναι τα μέσα των τμημάτων ΒΕ, ΓΔ και η ευθεία ΜΝ τέμνει τις ευθείες ΑΒ, ΑΓ στα σημεία Ζ και Η, να αποδείξετε ότι το τρίγωνο ΑΖΗ είναι ισοσκελές. (Βουλγαρία – 2002) ArJun.12 Σε ένα ισοσκελές τρίγωνο ΑΒΓ είναι οˆ ˆΒ Γ 80  . Στην πλευρά ΑΓ παίρνουμε τμήμα ΑΔ ΒΓ . Να αποδειχθεί ότι οˆΑΒΔ 10 . ArJun.13 Θεωρούμε το παραλληλόγραμμο ΑΒΓΔ, το μέσο Μ του ΑΒ, το μέσο Ν του ΑΓ και η προβολή Ρ του Δ στην ευθεία ΓΜ. Να αποδείξετε ότι ˆ ˆΑΡΝ ΑΒΓ . ArJun.14 Δίνεται ορθογώνιο τρίγωνο ΑΒΓ οˆ(Α 90 ) , το ύψος ΑΔ, τυχαίο σημείο Ρ του τμήματος ΑΔ, η κάθετη προς την ΒΡ στο Ρ που τέμνει την ευθεία ΓΑ στο Ε και η παράλληλη από το Ρ προς τη ΒΓ που τέμνει την ΑΓ στο Ζ. Να αποδείξετε ότι ΑΕ ΓΖ . ArJun.15 Δίνεται τετράγωνο ΑΒΓΔ και τα σημεία Μ, Ν των πλευρών ΓΔ, ΓΒ αντίστοιχα έτσι, ώστε ˆ ˆΜΑΔ ΜΑΝ . Στην προέκταση της ΝΒ παίρνουμε σημείο Ε τέτοιο, ώστε ΑΕ ΑΜ . Αν η ευθεία ΑΜ τέμνει τη ΒΓ στο Ρ, να αποδείξετε ότι: α) Το τρίγωνο ΑΕΡ είναι ορθογώνιο. β) ΡΕ ΔΜ ΒΝ 2   . γ) ΑΝ ΜΔ ΝΒ  . (Arad – 2008) ArJun.16 Σε ένα τραπέζιο ΑΒΓΔ με ΑΒ // ΓΔ είναι ΓΔ 2ΑΒ . Έστω Ν το μέσο του ΓΔ και Μ το μέσο του ΒΓ. Εξωτερικά του ΑΒΓΔ θεωρούμε τα ισόπλευρα τρίγωνα ΑΔΕ και ΓΝΖ. Να αποδειχθεί ότι: οˆΕΜΖ 90 .